SlideShare ist ein Scribd-Unternehmen logo
1 von 267
Glomeruli and associated blood vessels in the kidney (colorized scanning electron micrograph). From
Widmaier EP, Raff H , Strang KT. Vander’s Human Physiology : The Mechanisms of Body Function, 14th ed.
New York, NY: McGraw-Hill, 2016.
Pathology of Renal System
Marc Imhotep Cray, M.D.
Marc Imhotep Cray, MD
Overall Goal
2
The overall goal of this lecture series is to provide the learner with a
basic scientific understanding of the patterns, causes, mechanisms
and effects of the most clinically common renal disorders; including:
 Acute renal failure (AKI) and chronic renal failure (CKD)
 Glomerular diseases
 Urinary Tract Infections
 Tubulointerstitial diseases
 Obstructive uropathy, hydronephrosis & urolithiasis
 Vascular diseases
 Renal neoplasms
 Congenital and cystic diseases of kidney
 Volume regulation & electrolyte disorders
 Acid-base disorders
Covered elsewhere
within the sequence.
Marc Imhotep Cray, MD
Learning objectives (abridged*)
3
1.Describe the causes, mechanisms and effects of acute kidney
injury.
2.Describe the causes, mechanisms and effects of chronic kidney
disease.
3. Classify glomerular diseases based on acute vs chronic,
histopathologic features and clinical presentation.
4. Explain the pathogenesis (ie, immune-mediated Injury) and
pathophysiology of the most common glomerulonephritidies.
5. Describe the clinicopathologic features of the nephrotic vs
nephritic syndrome
*See Renal Pathology Learning Objectives, unabridged version.pdf
Marc Imhotep Cray, MD
Learning objectives cont’d.
4
6. Describe the characteristic light microscopy (LM),
immunofluorescence (IF), and electron microscopy (EM) findings
glomerular diseases.
7. Describe the renal features of systemic diseases (eg., HTN, DM,
SLE, amyloidosis, vasculitides) that involve the kidney.
8. Describe the major causes and pathologic mechanisms of
tubulointerstitial diseases.
9. Explain in the classification, etiology and pathogenesis of
urinary tract infections.
Marc Imhotep Cray, MD
Learning objectives cont’d.
5
10. Describe the gross and microscopic changes in acute
pyelonephritis (PN) and chronic PN.
11. List the causes of renal papillary necrosis.
Explain the causes and complications of obstructive uropathy,
and hydronephrosis.
12. Describe the composition of the most common forms,
favored sites for formation and complications of urolithiasis.
13. Describe the gross and microscopic features and typical
pathologic and clinical findings of commonest benign and
malignant tumors of the kidney.
14. List the congenital and cystic anomalies of the kidney.
Marc Imhotep Cray, MD
Topics Discussion Outline
6
 Scope of Problem
 Anatomy, Histology and Cell Biology
 Functions of kidney and lower urinary tract
 Overview of Renal Pathology
 Clinical Manifestations of Renal Diseases
 Glomerular Diseases
 Tubulointerstitial Diseases
 Vascular Diseases
 Obstructive uropathy, hydronephrosis & urolithiasis
 Renal Neoplasia
 Congenital and Cystic Diseases of Kidney
Marc Imhotep Cray, MD
Scope of Problem
7
Centers for Disease Control and Prevention estimates in U.S.
more than 10% of people 20 years and older (or more than 20
million individuals) have chronic kidney disease
 many more suffer from acute kidney injury annually
thus,
Clinicians of all specialties will encounter patients w renal
disorders behooves all to be aware of various risk factors and
causes of kidney disease (eg. hypertension and DM)
 Important b/c w early detection and appropriate management may
be able to prevent or at least slow rate of progression to kidney
failure or other complications
NB: Diagnosing kidney disease is particularly challenging, as patients are
typically asymptomatic until relatively advanced kidney failure is present.
Marc Imhotep Cray, MD
Lecture 1:
Scope of Problem
Anatomy, Histology and Cell Biology
Functions of kidney and lower urinary tract
Overview of Renal Pathology
Clinical Manifestations of Renal Diseases
8
Marc Imhotep Cray, MD
Anatomy, Histology and Cell Biology
9
Basic structural and functional unit of kidney function is nephron
 A structure consisting of a tuft of capillaries termed glomerulus and a
tubule
Each human kidney has approx. 1 million nephrons
 Each nephron is composed of an initial filtering component (renal
corpuscle [glomerulus& Bowman's capsule]) and a tubule specialized for
reabsorption and secretion (renal tubule)
NB: Understanding the complex functional organization of glomerulus is
crucial for understanding both normal renal function and characteristics
of different glomerular diseases.
Marc Imhotep Cray, MD
Schematic of Gross Anatomy
10
Mohan H. Textbook of Pathology, 7th Ed. New Delhi: Jaypee Brothers Medical Publishers, 2015.
Cross-section of kidney showing gross structures. Cross-section of kidney showing arterial blood supply
Renal blood flow: renal artery  segmental artery interlobar artery 
arcuate artery interlobular artery  afferent arteriole  glomerulus 
efferent arteriole  vasa recta/ peritubular capillaries  venous outflow
Marc Imhotep Cray, MD
Kidney and lower urinary tract
11
Kidney (upper urinary tract) &
lower urinary tract are
functionally connected
abnormalities in one component
can affect function of other
components of system
Bladder, urethra, ureters, & renal
pelvis (lower urinary tract) are
lined by transitional epithelium
(urothelium) subject to similar
types of disease processes
Widmaier EP, Raff H , Strang KT. Vander’s Human Physiology : The
Mechanisms of Body Function, 14th ed. New York, NY: McGraw-Hill, 2016.
12
Normal Kidney, Gross
Klatt EC. Robbins and Cotran Atlas of Pathology, 3rd Ed. Philadelphia: Saunders, 2015.
13
Normal Kidney, Gross cross section
Klatt EC. Robbins and Cotran Atlas of Pathology, 3rd Ed. Philadelphia: Saunders, 2015.
14
Normal Kidney, CT image
Klatt EC. Robbins and Cotran Atlas of Pathology, 3rd Ed. Philadelphia: Saunders, 2015.
15
Normal Kidney, microscopic
Klatt EC. Robbins and Cotran Atlas of Pathology, 3rd Ed. Philadelphia: Saunders, 2015.
16
Normal Kidney, microscopic
Klatt EC. Robbins and Cotran Atlas of Pathology, 3rd Ed. Philadelphia: Saunders, 2015.
17
Normal Kidney, microscopic HP
Klatt EC. Robbins and Cotran Atlas of Pathology, 3rd Ed. Philadelphia: Saunders, 2015.
Marc Imhotep Cray, MD
Structure & histology of renal corpuscle
18
Kelly CR, Landman J (Eds.). The Netter Collection of Medical Illustrations, 2Ed.
Vol 5- Urinary System. Philadelphia: Saunders- Elsevier, 2012;21.
Four layers make up filtering
apparatus of glomerulus each
of these can be affected in renal
disease with effects on filtration:
• basement membrane
• blood vessel lining (endothelial
cells)
• podocytes (epithelial cells)
• mesangium (smooth muscle-like
cells with phagocytes)
See: Histology of the glomerular filtration
barrier.pdf
Marc Imhotep Cray, MD
Glomerulus and glomerular capillary
19
Hammer GD & McPhee SJ (Eds.). Pathophysiology of Disease: An Introduction to Clinical Medicine, 7th Ed. New York, NY:
McGraw-Hill Education, 2014; 457.
A glomerulus consists of an afferent and an efferent arteriole and an intervening tuft of capillaries lined by
endothelial cells and covered by epithelial cells that form a continuous layer with those of Bowman capsule
and renal tubule. Space between capillaries in glomerulus is called mesangium. Material comprising a
basement membrane is located between capillary endothelial cells and epithelial cells.
Marc Imhotep Cray, MD
Detailed structure of glomerulus and glomerular filtration membrane
composed of endothelial cell, basement membrane, and podocyte.
20
Hammer GD & McPhee SJ (Eds.). Pathophysiology of Disease: An Introduction to Clinical Medicine, 7th Ed.
New York, NY: McGraw-Hill Education, 2014; 457.
Glomerular capillaries have unique features not found in most peripheral capillaries. First, glomerular capillary
endothelium is fenestrated. However, because endothelial cells have a coat of negatively charged glycoproteins and
glycosaminoglycans, they normally exclude plasma proteins such as albumin. On other side of glomerular basement
membrane are epithelial cells. Termed “podocytes” because of their numerous extensions or foot processes, these
cells are connected to one another by modified desmosomes. Mesangium is an extension of GBM.
21
(Shown on left) Anatomy of a normal glomerular capillary.
Note fenestrated endothelium (EN), glomerular
basement membrane (GBM), and epithelium with its foot
processes (EP). Mesangium is composed of mesangial cells
(MC) surrounded by extracellular matrix (MM) in direct
contact with endothelium. Ultrafiltration occurs across
glomerular wall and through channels in mesangial matrix
into urinary space (US).
(Shown on right) Typical localization of immune deposits
and other pathologic changes.
(1) Uniform subepithelial deposits as in membranous
nephropathy. (2) Large, irregular subepithelial deposits or
“humps” seen in acute postinfectious glomerulonephritis.
(3) Subendothelial deposits as in diffuse proliferative lupus
glomerulonephritis. (4) Mesangial deposits characteristic of
immunoglobulin A nephropathy. (5) Antibody binding to
glomerular basement membrane (as in Goodpasture
syndrome) does not produce visible deposits, but a smooth
linear pattern is seen on immunofluorescence.
(6) Effacement of epithelial foot processes is common in all
forms of glomerular injury with proteinuria.
Hammer GD & McPhee SJ (Eds.). Pathophysiology of Disease: An Introduction to
Clinical Medicine, 7th Ed. New York, NY: McGraw-Hill Education, 2014; 462.
22
Normal glomerular capillary electron micrograph
Rubin R, Strayer D (eds.) Rubin’s Pathology. Clinicopathologic Foundations of Medicine,
6th ed. Baltimore: Wolters Kluwer Health, 2012.
 This EM depicts a single capillary loop and
adjacent mesangium. Capillary wall portion of
lumen (L) is lined by a thin layer of fenestrated
endothelial cytoplasm (high magnification
next slide) that extends out from endothelial
cell body (E)
 Endothelial cell body is in direct contact with
mesangium, which includes the mesangial cell
(M) and adjacent matrix
 Outer aspect of basement membrane (B) is
covered by foot processes (F) from podocyte
(P) that line urinary space (U)
23
Electron micrograph of glomerular filter
Molecules that pass from capillary lumen
(CL) to urinary space (US) traverse
fenestrations (F) of endothelial cell (E)
trilaminar basement membrane (BM) (lamina
rara interna [LRI], lamina densa [LD] and
lamina rara externa [LRE]) and slit pore
diaphragm (D) that connects podocyte foot
processes (FP)
Rubin R, Strayer D (eds.) Rubin’s Pathology. Clinicopathologic Foundations of
Medicine, 6th ed. Baltimore: Wolters Kluwer Health, 2012.
REMEMBER: Endothelial cells have a coat of negatively
charged glycoproteins and glycosaminoglycans, they
normally exclude plasma proteins such as albumin.
24
Photo: Glomerulus in a human kidney scanning electron micrograph. From: Widmaier EP, Raff H & Strang
KT. Vander’s Human Physiology: The Mechanisms Of Body Function, 13th Ed. New York, NY: McGraw-Hill
Companies, 2014; 490. [Labeling mine]
Foot processes
Cell body of podocyte
Filtration slits
Bowman’s capsule podocytes with foot
processes and filtration slits.
Note the filtration slits between adjacent
foot processes. The podocytes and their foot
processes encircle the glomerular capillaries.
Marc Imhotep Cray, MD
Functions of kidney and urinary tract
25
1. Excretion:
 of waste products and drugs this involves
selective reabsorption and excretion of substances
as they pass through nephron
2. Regulation:
 of body fluid volume and ionic composition
kidneys have a major role in homeostasis and
 are also involved in maintaining acid–base balance
 Kidney participates in whole-body homeostasis via 4
main functions:
Marc Imhotep Cray, MD
Functions of kidney cont’d.
26
3. Endocrine:
 kidneys are involved in synthesis of renin (which generates
angiotensin I from angiotensinogen) thus has a role in
blood pressure and sodium balance)
 Erythropoietin  controls erythrocyte production; and
 Prostaglandins  involved in regulation of renal function
 ADH retain water in body and constrict blood vessels
4. Metabolism:
 Vitamin D is metabolized to its active form
 kidney is a major site for the catabolism of low-molecular-
weight proteins including several hormones such as insulin,
PTH and calcitonin
 has ability to produce glucose through gluconeogenesis
Marc Imhotep Cray, MD
Overview of Renal Pathology
27
 Kidney consists of glomerular, vascular, tubular &
interstitial anatomic compartments
 Renal diseases are best understood in relation to
compartments affected & assoc. functional impairment
 Pathology of kidney can be organized into four
anatomic categories= diseases of
1. glomeruli
2. tubules and
3. interstitium
4. vessels
thus,
Marc Imhotep Cray, MD
Overview (2)
28
 Diseases that affect glomeruli most often have an
immunologic etiology
 Glomerular function disrupted by diseases that alter
glomerular structural arrangements seen w structural
damage to basement membrane, endothelium, epithelium
or mesangium
whereas,
 Diseases that affect tubules and interstitium usually
have an infectious or toxic (drugs) etiology
 Tubular function disrupted by metabolic insult to tubular
cells  hypoxia or toxins
Marc Imhotep Cray, MD
Overview (3)
29
 Early in disease process, predominantly affect one of
four anatomic structures over time, however, entire
kidney becomes diseased b/c of close structural &
functional relationships in nephron
 tubules & interstitium usually affected together
 As both glomerular and tubular functions are highly
dependent on adequate perfusion by blood if bld
perfusion disrupted, both functions are impaired
Marc Imhotep Cray, MD
Overview (4)
30
 b/c of large physiologic reserve of kidneys, many diseases do
not become clinically apparent until majority of organ is
affected subtle abnormalities in laboratory findings* are
only early indication of renal disease
 Recognition of patterns of abnormalities, pathologic findings,
and clinical presentation are arguably more important to renal
pathology than in any other organ system
N.B. *Chronic kidney disease can be detected at an early
stage by measuring GFR (nml 90–140 mL/ min men, and
80–125 mL/ min for women).
Marc Imhotep Cray, MD
Overview(5)Types of renal failure syndromes
31
Partial renal failure syndromes affect only some elements of
renal function
 There are four main types of partial renal failure:
1. Asymptomatic hematuria
2. Persistent proteinuria
3. Nephrotic syndrome
4. Nephritic syndrome
Total renal failure syndromes all functions of nephron are
impaired
 There are two types of total renal failure syndromes:
1. Acute renal failure (=AKI) sudden cessation of nephron function,
affecting all nephrons at same time
2. Chronic renal failure (=CKD) nephrons are destroyed one by one
over a long period
Marc Imhotep Cray, MD
QUESTION: How does someone with a partial renal failure syndrome,
such as a nephrotic syndrome, eventually develops total renal failure
syndrome(CKD)? Steps in pathological progression follow…
32
In most glomerular diseases damage causing nephritic or nephrotic
syndrome also eventually causes glomeruli to become completely
scarred (hyalinized) leading to loss of individual nephrons
↓
Glomerular hyalinization is  result of excessive production of mesangial
matrix by mesangial cells over a long period of time
↓
Expanding mesangial matrix mass slowly but progressively crushes
glomerular architecture out of existence until no blood flows through
glomerular capillaries and no oxygenated blood passes into efferent
arterioles and peritubular capillary systems
↓
Tubules are deprived of oxygenated blood, and tubular epithelial cells die
irrevocably and become atrophic…cont’d. on next slide
Marc Imhotep Cray, MD
Partial renal failure to complete renal failure explained cont’d.
33
Thus destruction of glomerulus  leads to destruction of
entire nephron unit
↓
As more and more nephrons are destroyed partial renal
failure syndrome (nephritic or nephrotic)  develops into
total renal failure syndrome of chronic renal failure (CKD)
↓
This is associated with progressive shrinkage of kidney to form
a small, scarred organ termed end-stage kidney
↓
34Kibble J , Cannarozzi ML. Pathophysiology Flash Cards. New York: McGraw-Hill, 2013.
Marc Imhotep Cray, MD
Overview (5) Renal Function Tests (RFT)
35
 To assess renal function, tests are available to give information
regarding following parameters:
 Renal blood flow
 Glomerular filtration (next slide)
 Renal tubular function
 Urinary outflow unhindered by any obstruction
NB: Urinalysis and measurement of serum creatinine are
initial steps in evaluation of renal disorders.
 RFT divided into 4 groups:
1. Urinalysis
2. Concentration & dilution tests
3. Blood chemistry
4. Renal clearance tests
Marc Imhotep Cray, MD
Overview (6) Glomerular Filtration
36
Filtration Barrier
 Composed of 1) fenestrated capillary endothelium, 2) fused
glomerular basement membrane, and 3) podocyte foot
processes epithelial layer
o capillary endothelium serves as a size barrier, while basement
membrane contains heparan sulfate leads to a negative charge
barrier preventing protein (albumin) filtration (=primary barrier)
Glomerular Filtration Rate and Filtration Fraction
 GFR can be estimated by clearance of creatinine (CCr)
 Effective renal plasma flow (RPF) can be estimated by clearance of para-
aminohippuric acid (CPAH)
 Filtration fraction(FF) = fraction of RPF filtered across glomerular
capillaries  GFR/RPF=FF
Marc Imhotep Cray, MD
GFR estimated by CCr
37
True picture of status of renal function can be obtained by
measuring GFR achieved clinically by measuring CCr, using a
calculation based on Cr content of a 24-hour collection of urine
and a single bld Cr estimation
 GFR nml range: 90–140 mL/ min men & 80–125 mL/ min women
Blood levels of urea and creatinine do not rise above nml until
GFR (CCr) has fallen below 50% nml
 thus , for many years of a chronic progressive kidney disease pt. may be
asymptomatic or show only minor Sx or Sn
o important to detect these early stages and to investigate cause and
monitor rate of progression
Marc Imhotep Cray, MD
Overview (7) Urinalysis & Urine Microscopy
38
…Urine microscopy:
 Presence of casts indicates that hematuria/pyuria is of
glomerular or renal tubular origin
 Bladder cancer and kidney stones→hematuria, no casts
 Acute cystitis → pyuria, no casts
 RBC casts → glomerulonephritis, malignant hypertension
 Understanding basic urinalysis (UA) and urine microscopy
helps interpret pathology of kidney, for instance…
(See Urinalysis & Urine Microscopy. Pdf notes)
Marc Imhotep Cray, MD
Overview (9) Renal Biopsy
39
 In addition to RFT, renal biopsy is performed on selected pts w
kidney disease to confirm Dx Indications include: hematuria,
proteinuria, renal failure, transplant kidney surveillance…more
 Renal biopsy tissue sample is fixed in alcoholic solution and examined
morphologically supported by special stains…
1. Light microscopy (LM)
o Periodic acid-Schiff (PAS) stain for highlighting GBM
o Silver impregnation to outline glomerular & tubular BM
2. Immunofluorescence (IF) to localize antigens, complements and
immunoglobulins type of immunologic injury is assessed
3. Electron microscopy (EM) to see ultrastructure of glomerular
changes exact site of immune-complex deposition is visualized
Marc Imhotep Cray, MD
Overview (10) Definitions of key terms
40
Nephrotic syndrome: A kidney disorder affecting
glomerulus, leading to proteinuria >3.5 g/day,
hypoalbuminemia, generalized edema, and
hyperlipidemia
 Nephrosis: nephropathy without inflammation or neoplasia
Nephritic syndrome: A kidney disorder of oliguria,
hematuria, edema, and hypertension, resulting from
glomerulonephritis (also sub-nephrotic proteinuria)
 Nephritis: nephropathy with inflammation
Marc Imhotep Cray, MD
Definitions of key terms cont’d.
41
Primary glomerular disease: A condition of kidney that
leads to
 nephrosis, such as membranous glomerulopathy, minimal
change disease, and focal segmental glomerulosclerosis or
 nephritis, such as postinfectious glomerulonephritis, rapidly
progressive glomerulonephritis and IgA nephropathy (Berger
disease)
Secondary glomerular disease: Conditions that are
systemic, infectious, and (or) toxic & affect kidney,
leading to nephrosis or nephritis
 Some of causes are DM, SLE, amyloidosis, penicillamine,
chronic hepatitis B infection, HIV, SBE…
Marc Imhotep Cray, MD
Definitions of key terms cont’d.
42
 Azotemia is a biochemical abnormality that refers to
an elevation of blood urea nitrogen (BUN) and
creatinine (Cr) levels, and is related to a ↓ glomerular
filtration rate (GFR)
 Uremia is when azotemia becomes assoc. w a
constellation of clinical signs (Sn) and symptoms (Sx)
and biochemical abnormalities
Marc Imhotep Cray, MD
Definitions of key terms cont’d.
43
Glomerulosclerosis versus glomerulonephritis:
 In glomerulosclerosis (sclerosis means hardening) there are
sclerotic, scarred areas that lose ability to filter, secondary to
capillary collapse
 In glomerulonephritis (-itis means inflammation), there is
ongoing glomerular inflammation (=hypercellularity and
leukocytosis)
Crescentic: Used to describe appearance when inflammatory
cells fill Bowman’s space leading to a crescent appearance
 NB: always indicative of a rapidly progressive
glomerulonephritis (RPGN)=bad disease
Marc Imhotep Cray, MD
Definitions of key terms cont’d.
44
Glomerulopathy: can include processes that are inflammatory
or noninflammatory, however
 b/c term glomerulitis exists for inflammatory conditions,
glomerulopathy sometimes carries a noninflammatory
implication
 Regarding location of glomerular lesions:
 Subepithelial: between podocyte and GBM (on epithelial
side of GBM)
 Subendothelial: between endothelium and GBM (on
endothelial side of GBM)
Marc Imhotep Cray, MD
Clinical Manifestations of Renal Diseases
45
 Following is a discussion of clinical manifestations and syndromes of renal
diseases with their defining features two most common syndromes assoc.
w glomerular diseases, nephrotic and nephritic, are given most attention
 Azotemia and uremia (defined above)
 Asymptomatic hematuria or proteinuria
 Acute kidney injury
 Chronic kidney disease
 End-stage renal disease (ESRD)
 Renal tubular defects
 Urinary tract obstruction and renal tumors
 Nephrolithiasis (renal stones)
 Nephrotic syndrome
 Nephritic syndrome
Marc Imhotep Cray, MD
Clinical Manifestations (2)
46
 Azotemia =↑of BUN & Cr levels  reflects a ↓ (GFR)
 GFR may be ↓ as a consequence of intrinsic renal disease or
extrarenal causes
 Azotemia has three classifications, depending on causative origin
o Prerenal azotemia is encountered when there is hypoperfusion of
kidneys  usually due to reduced extracellular fluid volume 
usually reversible if hypoperfusion is corrected in time
o Primary renal azotemia (ARF [AKI]) typically leading to uremia
intrinsic disease of kidney resultant of renal parenchymal damage
• Causes include renal failure, glomerulonephritis, acute tubular necrosis, or
any other kind of renal disease
o Postrenal azotemia results when urine outflow is obstructed
relief of obstruction is followed by correction of azotemia
Marc Imhotep Cray, MD
Clinical Manifestations (3)
47
 When azotemia gives rise to clinical manifestations and
systemic biochemical abnormalities it becomes uremia=
 failure of renal excretory function + a host of metabolic & endocrine
alterations
 In addition, uremia is characterized by secondary GI (e.g.,
uremic gastroenteritis), neuromuscular (e.g., peripheral
neuropathy), and cardiovascular (e.g., uremic pericarditis)
involvement
48
Renal Failure and
Uremia Manifestations
Buja LM, Krueger GR. Netter’s Illustrated Human Pathology 2nd Ed. Philadelphia:
Saunders-Elsevier, 2014.
Inability to make urine & excrete nitrogenous
wastes:
Consequences (MAD HUNGER):
Metabolic Acidosis
Dyslipidemia (especially ↑triglycerides)
Hyperkalemia
Uremia—clinical syndrome marked by ↑ BUN:
 Nausea and anorexia
 Pericarditis
 Asterixis
 Encephalopathy
 Platelet dysfunction
Na+/H2O retention (HF, pulmonary edema,
hypertension)
Growth retardation and developmental delay
Erythropoietin failure (anemia)
Renal osteodystrophy
Marc Imhotep Cray, MD
Clinical Manifestations (5)
49
 Asymptomatic hematuria or proteinuria, or a combination
of these two represents a manifestation of subtle or mild
glomerular abnormalities
 Acute kidney injury is characterized by rapid decline in
GFR (within hours to days), w concurrent dysregulation of fluid
& electrolyte balance, and retention of metabolic waste
products normally excreted by kidney including urea and
creatinine
 In its most severe forms it is manifested by oliguria or anuria
(reduced or no urine flow)
 AKI can result from glomerular, interstitial, vascular or acute tubular
injury
Marc Imhotep Cray, MD
Clinical Manifestations (6)
50
 Chronic kidney disease (chronic renal failure) is defined as:
presence of a diminished GFR that is persistently <60 mL /minute
for at least 3 months, from any cause, and/or persistent
albuminuria
 CKD may present w clinically silent decline in renal excretory
function in milder forms, and
 in more severe cases, by prolonged Sx and Sn of uremia
 It is end result of all chronic renal parenchymal diseases
Etiologies of CKD
Buja LM, Krueger GR. Netter’s Illustrated Human Pathology 2nd Ed.
Philadelphia: Saunders-Elsevier, 2014.
Main groups of causes are:
 Chronic vascular disease (e.g. long-standing
hypertension
 Disease of glomeruli, e.g. glomerulonephritis
and diabetic glomerular disease
 Disease of tubules and interstitium
infective, toxic and obstructive damage to
tubules and renal papillae
 Some congenital kidney diseases, e.g.
autosomal dominant polycystic kidney
disease (ADPKD)
Note: A kidney in which virtually all nephrons
have been destroyed is called an end-stage
kidney
46
Marc Imhotep Cray, MD
Clinical Manifestations (8)
52
 End-stage renal disease (ESRD)  GFR is less than 5%
of normal this is terminal stage of uremia
NDD-CKD vs. ESRD
 The term "non-dialysis-dependent chronic kidney disease" (NDD-CKD)
is used to encompass status of those persons w established chronic
kidney disease (CKD) who do not yet require life-supporting treatments
for kidney failure known as renal replacement therapy RRT, including
maintenance dialysis or kidney transplantation
 Condition of individuals w CKD, who require either of two types of RRT
(dialysis or transplant) referred to as the end-stage kidney disease
(ESKD)
53
Staging of chronic kidney disease
Stevens A, Lowe J, Scott I. Core Pathology, 3rd Ed. St. Louis: Mosby-Elsevier, 2009.
NB: Patients with CKD should be prescribed medication with care,
as renal disease impairs excretion of certain drugs.
Note: Stages 4 and 5 are considered to equate to irreversible CKD.
54
End-stage renal disease, microscopic
Klatt EC. Robbins and Cotran Atlas of Pathology, 3rd Ed. Philadelphia: Saunders, 2015.
Marc Imhotep Cray, MD
Clinical Manifestations (11)
55
 Renal tubular defects are dominated by nocturia,
and electrolyte disorders (e.g., metabolic acidosis)
 A result of diseases that either directly affect tubular
structures (e.g., nephronophthisis-medullary cystic disease
complex) or cause defects in specific tubular functions
o Latter can be inherited (e.g., familial nephrogenic diabetes,
cystinuria, renal tubular acidosis [RTA]) or
o acquired (e.g., lead nephropathy)
Marc Imhotep Cray, MD
Clinical Manifestations (12)
56
 Urinary tract obstruction and renal tumors have varied
clinical manifestations based on specific anatomic
location and nature of lesion
 Urinary tract infection (UTI) is characterized by bacteriuria
and pyuria (bacteria and leukocytes in urine)
 Infection may be symptomatic or asymptomatic, and
 it may affect kidney (pyelonephritis) or bladder (cystitis)
 Nephrolithiasis (renal stones) is manifested by spasms of
severe pain (renal colic) and hematuria, often w recurrent
stone formation
Marc Imhotep Cray, MD
Question
57
A patient with acute renal failure is referred for dialysis.
The following are all indications for dialysis EXCEPT?
A. Severe metabolic acidosis
B. Uncontrollable hyperkalemia
C. Pulmonary edema
D. Pericarditis
E. Anemias
Marc Imhotep Cray, MD
Answer
58
Indications for dialysis include:
 Uncontrollable hyperkalemia
 Severe metabolic acidosis
 Pulmonary edema
 Overload of fluid not expected to respond to Tx w diuretics
 Uremic complications (pericarditis, encephalopathy,
GI bleeding, platelet dysfunction…) and
 Intoxication, that is, acute poisoning w a dialyzable substances
(eg. SLIME: salicylic acid, lithium, isopropanol, magnesium-
containing laxatives, and ethylene glycol)
Anemia in a pt. w ARF is not an indication for dialysis.
Marc Imhotep Cray, MD
Nephrotic vs Nephritic Syndrome Capsule
59
 Nephrotic Syndrome
 Proteinuria: b/c of disruption of glomerular charge barrier
 Hypoalbuminemia: b/c of proteinuria
 Edema: b/c of decreased plasma oncotic pressure from
proteinuria
 Hyperlipidemia & hypercholesterolemia: b/c of ↑ in
lipoprotein synthesis
 Nephritic Syndrome
 Oliguria & Azotemia: b/c of renal inflammation
 Hypertension: results from ↓clearance of sodium & water
 Hematuria: b/c of leakage of blood into Bowman capsule
Marc Imhotep Cray, MD
Nephrotic syndrome pathophysiology
60
Heavy proteinuria, exceeds 3.5 g/day in nephrotic syndrome
Nephrotic syndrome has diverse causes that share a common
pathophysiology a derangement in capillary walls of glomeruli
results in ↑ permeability to plasma proteins
↑ permeability of GBM may result from structural or physicochemical
alterations
With long-standing or heavy proteinuria, serum albumin is ↓ giving rise
to hypoalbuminemia  a drop in plasma colloid osmotic pressure
which in turn leads to leakage of fluid from bld into extravascular
spaces…cont’d next slide
Marc Imhotep Cray, MD
Nephrotic syndrome cont’d.
61
 … resulting ↓in intravascular volume and renal blood flow triggers ↑
release of renin from renal juxtaglomerular cells renin in turn
stimulates angiotensin-aldosterone axis which promotes retention of
salt and water by kidney
 This tendency is exacerbated by reductions in cardiac secretion of
natriuretic factors ( ANP & BNP) attributed to ↓ intravascular volume
 In face of continuing proteinuria, salt and water retention further
aggravates edema  if unchecked may lead to development of
generalized edema (termed anasarca)
 At onset, there is little or no azotemia, hematuria, or hypertension occurs
Marc Imhotep Cray, MD
Nephrotic syndrome cont’d.
62
 Genesis of hyperlipidemia is more murky
 presumably, hypoalbuminemia triggers ↑ synthesis of lipoproteins in
liver, or
 massive proteinuria causes loss of an inhibitor of lipoprotein
synthesis
 There is also is abnormal transport of circulating lipid particles
and impairment of peripheral breakdown of lipoproteins
 associated lipiduria reflects ↑ permeability of GBM to lipoproteins
Marc Imhotep Cray, MD
Nephrotic syndrome cont’d.
63
 Hypercoagulability is a clinically significant manifestation of
nephrotic syndrome  caused by renal losses of proteins C &
S and antithrombin III , as well as elevated serum fibrinogen
and lipid levels
Note: Thyroid-binding globulin is also lost, however, this does not
appear to be clinically significant, as pts are euthyroid when free T4
is measured
Marc Imhotep Cray, MD
Pathophysiology of
nephrotic syndrome
64
Rubin R, Strayer D (eds.) Rubin’s Pathology. Clinicopathologic Foundations
of Medicine, 6th ed. Baltimore: Wolters Kluwer Health, 2012.
In nephrotic syndrome, RAS
is activated, leading to salt
and water retention and
edema
Marc Imhotep Cray, MD
Nephrotic syndrome cont’d.
65
Most important of primary glomerular lesions that lead to
nephrotic syndrome are focal segmental glomerulosclerosis
and minimal-change disease
 minimal-change disease is more important in children
 segmental glomerulosclerosis is more important in adults
Nephrotic syndrome is also commonly seen in two other
primary kidney diseases membranous nephropathy and
membranoproliferative glomerulonephritis, and as a
complication of systemic disease diabetes mellitus
Marc Imhotep Cray, MD
Nephritic syndrome
66
 Nephritic syndrome is a clinical entity caused by glomerular
disease and is dominated by acute onset
 either grossly visible hematuria (RBCs in urine) or microscopic
hematuria w dysmorphic red cells and red cell casts on urinalysis
 diminished GFR
 mild to moderate proteinuria (< 3.0 g/day), and
 Mild hypertension
 It is classic presentation of acute poststreptococcal
glomerulonephritis
 Rapidly progressive glomerulonephritis (RPGN) is
characterized as a nephritic syndrome with rapid decline in
GFR (within hours to days)
Marc Imhotep Cray, MD
Nephritic syndrome cont’d
67
 Nephritic syndrome usually has an acute onset and is
caused by inflammatory lesions of glomeruli
 lesions that cause nephritic syndrome have in common
proliferation of cells within glomeruli, often accompanied
by an infiltrate of leukocytes
 inflammatory reaction injures capillary walls permitting
blood to pass into urine, and induces hemodynamic
changes that lead to a reduction in GFR
Marc Imhotep Cray, MD
Nephritic syndrome cont’d
68
 Reduced GFR is manifested clinically by oliguria (or anuria),
fluid retention, and azotemia
 Hypertension is a result of both fluid retention and
augmented renin release from ischemic kidneys
 acute nephritic syndrome may be caused by primary
glomerular diseases, such as
 postinfectious glomerulonephritis GN and
 various forms of crescentic GN, or
 as a result of systemic disorders such as systemic lupus
erythematosus, amyloidosis, diabetes, hypertension
69
Etiologies of most common two renal syndromes
Syndrome Common Etiologies Uncommon Etiologies
Nephrotic
syndrome
Minimal change disease, focal
segmental glomerulosclerosis,
diabetic nephropathy
Membranous glomerulonephritis
(GN)75% of cases are idiopathic, but
2° causes include SLE, penicillamine,
gold, NSAIDs, HBV, HCV, syphilis, and
malignancy
Renal amyloidosis, SLE WHO Class V
(membranous form)
Nephritic
syndrome
Postinfectious GN; IgA nephropathy;
RPGN, ANCA-associated, pauci-
immune GN; SLE
Membranoproliferative GN, HCV,
cryoglobulinemia, Goodpasture’s
syndrome, vasculitides, TTP, HUS,
hereditary nephritis (Alport’s syndrome)
Redrawn after: Le T and Bhushan V. First Aid for the Wards, 5th Ed. New York: McGraw-Hill, 2013.
Marc Imhotep Cray, MD
Lecture 2: Glomerular Diseases
70
Marc Imhotep Cray, MD
Glomerular Diseases
71
Terminology use to describe syndromes is explained in next 4 slides.
 Many renal disorders are caused by injury to glomerulus
 Glomeruli may be only major site of disease (1° glomerular disease;
e.g., immunoglobulin [Ig]A nephropathy) or
 Part of a disease affecting several organs (2° glomerular disease; e.g.,
lupus glomerulonephritis)
 Signs (Sn) & symptoms (Sx) of fall into one of following categories:
 Asymptomatic proteinuria
 Asymptomatic hematuria
 Nephrotic syndrome
 Acute nephritic syndrome
 Rapidly progressive nephritic syndrome
 Chronic kidney injury
 ESRD
Marc Imhotep Cray, MD
Terminology describing glomerular syndromes
72
All are histologic determinations after a renal biopsy:
Focal vs diffuse: Defines number of glomeruli on biopsy affected
 If less than half glomeruli are affected it is focal
 If more than half are affected it is diffuse
Segmental vs global: Defines how much of each individual
glomerulus is affected, for each affected glomerulus
 If only part is affected then it is segmental
 If entire glomerulus is affected it is global
Therefore, focal segmental glomerulosclerosis means  less than
half of glomeruli are affected [focal] and, of those affected, only
part of each glomerulus is affected [segmental]
73
Patterns of glomerular disease
Modified from: Stevens A, Lowe J, Scott I. Core Pathology, 3rd Ed. St. Louis: Mosby-Elsevier, 2009; 367.
Global: affecting the whole of the glomerulus uniformly.
Segmental: affecting one glomerular segment, leaving
other segments unaffected.
Diffuse: affecting all glomeruli in both kidneys. Focal: affecting a proportion of glomeruli, others unaffected.
Marc Imhotep Cray, MD
Terminology describing glomerular syndromes
74
 Membranous vs proliferative vs membranoproliferative:
Membranous, glomerular basement membrane (GBM) becomes
thickened in parts thickenings appear as spikes & domes on
microscopy b/c of bulging membrane
Proliferative indicates cells are proliferating & numerous nuclei
seen on microscopy from added cell count
Membranoproliferative indicates membranous thickening &
proliferation  leads to a so-called tram track appearance b/c
GBM is rebuilt on top of damaged deposits
Marc Imhotep Cray, MD
Summary of glomerular disorders nomenclature
TYPE CHARACTERISTICS EXAMPLE
Focal < 50% of glomeruli are involved Focal segmental glomerulosclerosis
Diffuse > 50% of glomeruli are involved Diffuse proliferative glomerulonephritis
Proliferative Hypercellular glomeruli Membranoproliferative glomerulonephritis
Membranous Thickening of glomerular basement
membrane(GBM)
Membranous nephropathy
Primary
glomerular
disease
1° disease of kidney specifically
impacting glomeruli
Minimal change disease
Secondary
glomerular
disease
Systemic disease or disease of
another organ system that also
impacts glomeruli
SLE, diabetic nephropathy
Redrawn after: Le T, Bhushan V, et al. First Aid for the USMLE Step 1 2017. McGraw-Hill Education, 2017.
 If only part is affected then it is segmental
 If entire glomerulus is affected it is global
Marc Imhotep Cray, MD
Clinical features of glomerulonephritis relate
broadly to histological findings.
76
 There are four general rules of thumb which, albeit not
absolute, explain vast majority of clinical patterns of disease
that relate to each of several types of glomerulonephritis.
1. Structural change in glomerular basement membrane
(GBM) (usually thickening) or deposition of excessive
mesangial matrix leads to increased loss of protein in urine,
which if severe leads to the nephrotic syndrome.
Marc Imhotep Cray, MD
Clinical features of glomerulonephritis relate
to histological findings cont’d.
77
2. Glomerular damage associated with proliferation of
endothelial or mesangial cells is associated with the
development of microscopic hematuria or nephritic
syndrome.
3. If there is both damage to GBM and cell proliferation, a
mixed nephritic/ nephrotic syndrome is likely.
4. If damage to glomeruli is rapid and widespread, features of
acute renal failure develop.
Marc Imhotep Cray, MD
Acute versus Chronic Glomerulonephritis
78
Acute (and Rapidly Progressive) Glomerulonephritis
 There are several ways to classify acute GN
 Light microscopy (LM) essential for establishing areas of injury
 Circulating autoantibodies and measures of complement deposition
combined w Immunofluorescence (IF) studies and
 Electron microscopy (EM)
 LM, EM & IF allow GN to be categorized into subgroups
correlating w other features of disease  three patterns emerge:
1. Antiglomerular basement membrane ( anti-GBM ) antibody disease
2. Immune complex glomerulonephritis
3. Anti-neutrophil cytoplasmic antibody (ANCA) disease or pauci-immune
GN
 A brief description of each pattern with disease examples follows…
Marc Imhotep Cray, MD
Acute versus Chronic GN cont’d.
79
1. Anti-GBM antibody disease (eg, Goodpasture syndrome):
 This disease results from development of circulating
antibodies to an antigen intrinsic to GBM
 Binding of these pathologic anti-GBM antibodies to GBM
causes a cascade of inflammation
 Light microscopy shows crescentic GN, and characteristic
linear immunoglobulin deposition in glomerular capillaries
is seen on immunofluorescence
Marc Imhotep Cray, MD
Acute versus Chronic GN cont’d.
80
2. Immune complex glomerulonephritis:
 Immune complex deposition are seen in a variety of diseases
 On renal biopsy, granular immunoglobulin deposits are suggestive
of immune complexes from underlying systemic disease
 A classic example is postinfectious GN in which there is cross-
reactivity betw. an antigen of infecting organism and a host
antigen resulting in deposition of immune complexes and
complement in glomerular capillaries and mesangium
o Resolution of glomerular disease typically occurs weeks after Tx of original
infection
 Other examples include IgA nephropathy, lupus nephritis, &
membranoproliferative GN
Marc Imhotep Cray, MD
Acute versus Chronic GN cont’d.
81
3. Anti-neutrophil cytoplasmic antibody (ANCA) disease
or pauci-immune GN:
 Characterized by a necrotizing GN but few or no immune deposits
(hence, pauci-immune) seen on immunofluorescence or electron
microscopy
 This pattern is typical of granulomatosis with angiitis, microscopic
polyangiitis, or Churg-Strauss syndrome
 ANCA-negative pauci-immune necrotizing GN occurs less frequently
but is also a well-described clinical entity
Marc Imhotep Cray, MD
Acute versus Chronic GN cont’d.
82
 Rapidly progressive glomerulonephritis (RPGN)
 A subset of acute GN in which there is a progressive and dramatic
decline (weeks to months) in renal function often leading to
complete renal failure and oliguria
o It is a nonspecific final pathway in a variety of glomerular diseases
 Early disease can be subtle, but is marked by proteinuria and
hematuria followed by ↓ GFR
 Often called “crescentic GN,” as characteristic finding on biopsy is
cellular crescents in Bowman space
 Cellular crescents, visible on light microscopy, form in response to
severe damage to glomerular capillaries
N.B. Recovery without specific treatment is rare
Marc Imhotep Cray, MD
Acute versus Chronic GN cont’d.
83
Chronic Glomerulonephritis
 Some patients with acute GN develop CKD slowly over a period
of 5–20 years pathogenesis includes:
 Cellular proliferation, in either mesangium or capillary, is a pathologic
structural hallmark in some of these cases
whereas,
 others are notable for obliteration of glomeruli = sclerosing chronic
GN includes both focal and diffuse subsets
and yet,
 others display irregular subepithelial proteinaceous deposits w
uniform involvement of individual glomeruli (membranous GN)
84
Chronic glomerulonephritis, gross
Klatt EC. Robbins and Cotran Atlas of Pathology, 3rd Ed. Philadelphia: Saunders, 2015.
Marc Imhotep Cray, MD
Diagnostic features of glomerular diseases
85
I. Light microscopic (LM) features
A. Increased cellularity
Infiltration by leukocytes (e.g., neutrophils, monocytes, macrophages)
Proliferation of “endocapillary” cells (i.e., endothelial and mesangial cells)
Proliferation of “extracapillary” cells (i.e., epithelial cells) (crescent formation)
B. Increased extracellular material
Localization of immune complexes
Thickening or replication of GBM
Increases in collagenous matrix (sclerosis)
Insudation (collection) of plasma proteins (hyalinosis)
Fibrinoid necrosis
Deposition of amyloid
Marc Imhotep Cray, MD
Dx features of glomerular diseases (2)
86
II. Immunofluorescence (IF) features
A. Linear staining of GBM
Anti-GBM antibodies
Multiple plasma proteins (e.g., in diabetic glomerulosclerosis)
Monoclonal light chains (Amyloid nephropathy )
B. Granular immune complex staining
Mesangium (e.g., IgA nephropathy)
Capillary wall (e.g., membranous glomerulopathy)
Mesangium and capillary wall (e.g., lupus glomerulonephritis)
C. Irregular (fluffy) staining
Monoclonal light chains (AL amyloidosis) AA protein (AA amyloidosis)
Marc Imhotep Cray, MD
Dx features of glomerular diseases (3)
87
III. Electron microscopic features
A. Electron-dense immune complex deposits
Mesangial (e.g., IgA nephropathy)
Subendothelial (e.g., lupus glomerulonephritis)
Subepithelial (e.g., membranous glomerulopathy)
B. GBM thickening (e.g., diabetic glomerulosclerosis)
C. GBM replication (e.g., membranoproliferative glomerulonephritis)
D. Collagenous matrix expansion (e.g., focal segmental glomerulosclerosis)
E. Fibrillary deposits (e.g., amyloidosis)
To view plates see: LM, IF and EM of Select Glomerular Diseases_pdf.
Marc Imhotep Cray, MD
Mechanisms of Glomerular Injury & Disease
88
 Immune mechanisms underlie most primary glomerular
diseases & many secondary glomerular diseases
 Two mechanisms of antibody deposition in glomerulus have
been established:
1. deposition of circulating antigen-antibody complexes in glomerular
capillary wall or mesangium, and
2. antibodies reacting in situ within glomerulus either with fixed
(intrinsic) glomerular antigens or with extrinsic molecules that are
planted in glomerulus
o Deposition of circulating immune complexes gives a granular
immunofluorescence pattern
o Anti-GBM antibody GN is characterized by a linear immunofluorescence
pattern there is no immune deposit formation in this disease
Marc Imhotep Cray, MD
Mechanisms of Glomerular Injury & Disease
cont’d.
89
Once antigen-antibody complexes are deposited or formed in
glomeruli they produce injury by activating complement and recruiting
leukocytes
 Binding of immune complexes to Fc receptors on leukocytes also may contribute to
activation of cells and injury
 Morphologically, affected glomeruli exhibit leukocytic infiltrates and
proliferation of mesangial and parietal epithelial cells
 Electron microscopy reveals electron-dense immune deposits in one or
more of three locations:
1. Betw. endothelial cells and GBM (subendothelial deposits)
2. Betw. outer surface of GBM and podocytes (subepithelial deposits),
3. In the mesangium
Marc Imhotep Cray, MD
Antibody-mediated glomerular injury. Injury can result either from deposition of
circulating immune complexes or from antibody-binding to glomerular components
followed by formation of complexes in situ
90
Kumar V, Abbas AK, Aster JC. Robbins Basic Pathology, 10th ed. Philadelphia: Elsevier, 2018.
Marc Imhotep Cray, MD
Two patterns of deposition of immune complexes as seen by
immunofluorescence microscopy.
(A) Granular, characteristic of circulating and in situ immune complex deposition.
(B) Linear, characteristic of classic anti-glomerular basement membrane (anti-GBM) antibody
glomerulonephritis.
91
Kumar V, Abbas AK, Aster JC. Robbins Basic Pathology, 10th ed. Philadelphia: Elsevier, 2018
92
Localization of immune complexes in glomerulus:
Kumar V, Abbas AK, Aster JC. Robbins and Cotran Pathologic Basis of
Disease, 9th ed. Philadelphia: Saunders-Elsevier, 2015.
1. Subepithelial humps as in acute
glomerulonephritis
2. Epimembranous deposits as in
membranous nephropathy
3. Subendothelial deposits as in lupus
nephritis & membranoproliferative
glomerulonephritis
4. Mesangial deposits as in IgA
nephropathy
EN, Endothelium; EP, epithelium; GBM, glomerular basement membrane;
LD, lamina densa; LRE, lamina rara externa; LRI, lamina rara interna; MC,
mesangial cell; MM, mesangial matrix.
Marc Imhotep Cray, MD
Clinical presentations of glomerulopathy /
glomerulonephritis(GN)
93
Clinical presentation of all various types of GN can take
one of six forms:
1. Nephritic syndrome: characterized by oliguria, hematuria,
edema, and hypertension
2. Nephrotic syndrome: characterized by massive proteinuria,
edema, hypoalbuminemia, and hyperlipidemia
3. Acute renal failure (now called acute kidney injury [AKI])
4. Chronic renal failure (now called chronic kidney disease[CKD])
5. Isolated (asymptomatic) proteinuria
6. Isolated (asymptomatic) hematuria
Asymptomatic meaning no functional
abnormalities assoc. w reduced GFR,
edema, or hypertension.
Marc Imhotep Cray, MD
Suggested approach to reading/studying the data
that follows.
94
The discussions of glomerular disorders that follow are presented in
a concise and consistent format. Data for each disease commences
by presenting a trigger case illustrating the clinical relevance of the
pathologic disorder under consideration. Then the etiology and
epidemiology, pathology and pathogenesis, clinical manifestations,
treatment options and prognosis are presented.
Learners are encouraged read the trigger case with close attention
and extract all data thought to be relevant to the diagnosis before
proceeding to the knowledge-base.
This approach should make for good practice in appreciating the
intimate relationship of basic science pathology to clinical medicine.
Marc Imhotep Cray, MD
Trigger Case 1
95
A 5-year-old boy presents to the emergency room with a 1-week
history of generalized edema and fatigue. Your history reveals that
he suffered from a viral URI 1 week before this visit. Serum and
urine studies reveal massive proteinuria, hyperlipidemia, and
hypoalbuminemia. You suspect that a renal biopsy would show
normal-appearing glomeruli on electron microscopy except for
fusion of the epithelial foot processes and you begin the child on
prednisone.
What is the Diagnosis?
Marc Imhotep Cray, MD
Minimal Change Disease (Lipoid Nephrosis)
96
Etiology & Epidemiology
 Etio. unknown, but usually occurs following a viral URI
 also assoc. w Hodgkin disease & hypersensitivity reactions
 Most often seen in young children, but can occur in older
children and adults
Pathology
 LM: Normal-appearing glomeruli can see lipid
accumulation in renal tubular cells
 EM: Fusion of epithelial foot processes
Marc Imhotep Cray, MD
MCD cont’d.
97
Clinical Manifestations Nephrotic syndrome
 Complications include infection by gram-positive organism,
thromboembolism, and protein malnutrition
Treatment (Tx) & Prognosis (Px) Prednisone;
cyclophosphamide or chlorambucil for steroid-resistant cases
 Response is excellent
Note: Minimal change disease is the prototype of nephrotic
syndrome
Marc Imhotep Cray, MD
MCD
(A) When viewed with a LM, silver methenamine–stained glomerulus appears
normal, w a delicate basement membrane.
(B) Schematic diagram illustrating diffuse effacement of foot processes of
podocytes with no immune deposits.
98
Kumar V, Abbas AK, Aster JC. Robbins and Cotran Pathologic Basis of Disease, 9th ed. Philadelphia: Saunders-Elsevier, 2015.
99
MCD, Electron micrograph
Klatt EC. Robbins and Cotran Atlas of Pathology, 3rd Ed. Philadelphia: Saunders, 2015.
Marc Imhotep Cray, MD
Trigger Case 2
100
A 40-year-old woman with a history of SLE presents to your office
with a chief complaint of increased swelling in her legs. She had
been referred by her primary care physician, who suspected a
secondary illness to her lupus. Recent laboratory studies show
proteinuria, hypoalbuminemia, hyperlipidemia, and
hypercholesterolemia. You suspect that a renal biopsy would
demonstrate immune complex deposition on electron
microscopy as well as a “spike and dome” appearance on silver
methenamine stain.
What is the Diagnosis?
101
Membranous Glomerulonephritis (also called
M. Glomerulopathy or M. Nephropathy)
Etio. & Epidem.
 An immune complex disease of unknown etiology
 Secondary disease seen in 10% of SLE patients (type V Lupus Nephritis)
 is sometimes assoc. w infections (eg, hepatitis B and C, syphilis, malaria), drugs
(eg, gold salts, penicillamine, NSAIDs), or malignancy
 Incidence is highest in adults
Pathology
 LM: Diffuse capillary wall thickening & basement membrane (BM) thickening
 IF: Granular pattern of IgG or C3 deposits (lumpy-bumpy)
 EM: Electron dense immune complex deposition in subepithelial locations
within BM of glomerular capillary walls
 Silver methenamine stain: A spike-and-dome appearance resulting from
extension of BM betw. and around immune deposits (spikes = basement
membrane, domes = immune complex deposits)
Marc Imhotep Cray, MD
Membranous Glomerulopathy cont’d.
102
Clinical Manifestations Nephrotic syndrome accompanied
by azotemia
 Complications include renal vein thrombosis and higher incidence of
occult neoplasms of lung, stomach, and colon
Tx Cyclophosphamide or steroids
 ACE inhibitors (reduce urinary protein loss)
 renal transplantation for severe cases
Note: Membranous glomerulopathy is second most common cause of
nephrotic syndrome in adults, w focal segmental glomerulosclerosis
(FSGS) recently becoming most common
103
Membranous nephropathy
A. Silver methenamine stain. Note marked diffuse
thickening of capillary walls without an increase in
number of cells. There are prominent “spikes” of
silver-staining matrix (arrow) projecting from
basement membrane lamina densa toward urinary
space, which separate and surround deposited
immune complexes that lack affinity for silver stain.
B. EM showing electron-dense deposits(arrow)
along epithelial side of basement membrane (B).
Note effacement of foot processes overlying
deposits. CL, Capillary lumen; End, endothelium; Ep,
epithelium; US, urinary space.
C. Characteristic granular immunofluorescent
deposits of IgG along glomerular basement
membrane.
D. Diagrammatic representation of membranous
nephropathy.
Kumar V, Abbas AK, Aster JC. Robbins and Cotran Pathologic Basis of
Disease, 9th ed. Philadelphia: Saunders-Elsevier, 2015.
Marc Imhotep Cray, MD
Trigger Case 3
104
A 40-year-old HIV-positive man is admitted to the hospital
complaining of generalized edema and fatigue. A complete
history reveals that he is a habitual IV drug user. Laboratory
studies show hypoalbuminemia, hyperlipidemia, proteinuria,
and microscopic hematuria. You suspect that his current
presentation is related to his HIV and you prepare the patient
for a renal biopsy to determine the exact diagnosis.
What is the Diagnosis?
Marc Imhotep Cray, MD
Focal Segmental Glomerulosclerosis (FSGS)
105
Etio. & Epidem.
 Often idiopathic; has been assoc. w heroin use, HTN, prior GN
and HIV infection (=collapsing glomerulopathy)
 Most often occurs in older patients
Pathology
 LM: Sclerosis within capillary tufts of deep juxtaglomerular
glomeruli w focal and segmental distribution
 hyalinosis (deposition of hyaline masses) also seen
 IF: granular mesangial fluorescence for IgM and C3
 EM: Fusion of epithelial foot processes
Marc Imhotep Cray, MD
FSGS cont’d.
106
Clinical Manifestations nephrotic syndrome
 more severe disease in HIV and IV drug users
 Lab findings: 80% have microscopic hematuria at presentation
Tx & Px Prednisone
 Most patients progress to ESRD in 5–10 years
Notes:
 FSGS most common cause of nephrotic syndrome in adults in U.S.
 b/c of focal nature of FSGS, early cases can be difficult to distinguish from
MCD (How might one differentiate the two clinically?)
 FSGS, w no cellular proliferation, is different from focal segmental
glomerulonephritis (FSGN), which involves cellular proliferation
Marc Imhotep Cray, MD
FSGS, microscopic (PAS stain)
107
A Low-power view showing segmental sclerosis in one of three glomeruli
(at 3 o’clock)
B High-power view showing hyaline insudation (arrow) and lipid (small
vacuoles) in sclerotic area
Kumar V, Abbas AK, Aster JC. Robbins and Cotran Pathologic Basis of Disease, 9th ed. Philadelphia: Saunders-Elsevier, 2015.
Marc Imhotep Cray, MD
Collapsing Glomerulopathy
108
Visible are retraction of glomerular tuft (arrows), narrowing of
capillary lumens, proliferation and swelling of visceral epithelial cells
(double arrows), and prominent accumulation of intracellular protein
absorption droplets in the visceral epithelial cells (arrowheads).
Kumar V, Abbas AK, Aster JC. Robbins and Cotran Pathologic Basis of
Disease, 9th ed. Philadelphia: Saunders-Elsevier, 2015.
 A morphologic variant of FSGS,
characterized by retraction
and/or collapse of entire
glomerular tuft
 May be idiopathic, but it also has
been assoc. w some drug
toxicities (e.g., pamidronate)
 Most characteristic lesion of
HIV-associated nephropathy
 Typically assoc. w prominent
tubular injury
 It has a very poor prognosis
Marc Imhotep Cray, MD
Trigger Case 4
109
A 60-year-old African Americans man with a 20-year history of
type II DM presents for a nephrology consult after his primary
care physician found a progressive increase of proteinuria in
recent laboratory studies. These laboratory studies also showed
hyperlipidemia and hypercholesterolemia. On physical
examination, the patient has bilateral diabetic retinopathy and 2+
edema in both legs. You start the patient on an ACE inhibitor and
you suspect that a renal biopsy would show Kimmelstiel-Wilson
nodules.
What is the Diagnosis?
Marc Imhotep Cray, MD
Diabetic Nephropathy
110
Etio. & Epidem. Assoc. w long-standing diabetes
 T1DM carries 30%–40% chance of diabetic nephropathy after 20 years
 T2DM carries 15%–20% chance after 20 years however, b/c there
are more pts w type II diabetes ESRD is more prevalent among
type II diabetics
 higher risk of developing diabetic nephropathy among men, African
Americans and Native Americans w DM
Pathology LM: Increase in mesangial matrix, resulting in either
 diffuse glomerulosclerosis (diffusely distributed ↑ in mesangial
matrix) or
 nodular glomerulosclerosis (Kimmelstiel- Wilson nodules—nodular
accumulations of mesangial matrix material)
 EM: Striking ↑ in GBM thickening
Marc Imhotep Cray, MD
Diabetic Nephropathy cont’d.
111
Clinical Manifestations  Nephrotic syndrome
 diabetic retinopathy is invariably present
 Lab findings: Microalbuminuria (early sign), proteinuria (late sign)
Tx & Px
 Strict glycemic control
 Treatment of hypertension and microalbuminuria w ACE inhibitors
during early stages to slow progression
 Often progresses to ESRD and dialysis
Note: Diabetic nephropathy is most common cause of ESRD in U.S.
112
Nodular glomerulosclerosis, microscopic
Klatt EC. Robbins and Cotran Atlas of Pathology, 3rd Ed. Philadelphia: Saunders, 2015.
113
Diffuse glomerulosclerosis, microscopic
Klatt EC. Robbins and Cotran Atlas of Pathology, 3rd Ed. Philadelphia: Saunders, 2015.
114
A 60-year-old man complains of chronic back pain and fatigue,
excessive urination, and increased thirst. X-ray examination
reveals numerous lytic lesions in the lumbar vertebral bodies.
Laboratory studies show hypoalbuminemia, mild anemia, and
thrombocytopenia. Urinalysis displays 4+ proteinuria. A
monoclonal immunoglobulin light-chain peak is demonstrated
on serum electrophoresis. A bone marrow biopsy discloses
foci of plasma cells, which account for 20% of all
hematopoietic cells. A kidney biopsy is obtained (shown in
image). Which of the following is the most likely cause of
nephrotic syndrome in this patient?
A. Amyloid nephropathy
B. Crescentic glomerulonephritis
C. IgA nephropathy (Berger disease)
D. Membranous glomerulonephritis
E. Nodular glomerulosclerosis (Kimmelstiel-Wilson disease)
Question
Marc Imhotep Cray, MD
115
Diagnosis (A) , Amyloid nephropathy, multiple myeloma
Neoplastic plasma cells typically secrete a homogeneous immunoglobulin
chain, which can be detected in serum or urine by electrophoresis.
Amyloid nephropathy is caused by the deposition of secreted light chains in
the glomerular basement membranes and mesangial matrix. Amorphous
acellular material expands the mesangium and obstructs the glomerular
capillaries. Deposits of AL amyloid may also appear in the tubular basement
membranes and in the walls of renal vessels.
Renal amyloidosis usually presents with nephrotic syndrome.
The deposits of amyloid may take on a nodular appearance, reminiscent of
Kimmelstiel-Wilson lesion of diabetic glomerulosclerosis (choice E).
However, amyloid deposits are not PAS positive and are identifiable by Congo
red staining with characteristic apple-green birefringence.
IgA nephropathy (choice C) and membranous glomerulonephritis (choice D) are
unrelated to light-chain disease.
116
Renal Amyloidosis
 Etiology:
 Amyloidosis is complication of chronic
inflammatory disorders, such as RA,
osteomyelitis or CA= multiple myeloma etc.
 kidneys, liver, spleen, and adrenals most
common organs involved
 accumulation of proteins in form of
abnormal, insoluble fibers=amyloid fibrils
 Pathology:
 Amyloidosis leads to nephrotic syndrome
and renal failure
 LM-Congo red stain shows apple-green
birefringence under polarized light due to
amyloid deposition in mesangium
 Diagnosis: Congo red staining or tissue biopsy
gold standard methods of Dx
Amyloid nephropathy. In a section stained w Congo
red and examined under polarized light, amyloid
deposits in glomerulus and adjacent arteriole show
a characteristic apple-green birefringence
Rubin R, Strayer D (eds.) Rubin’s Pathology. Clinicopathologic Foundations of
Medicine, 6th ed. Baltimore: Wolters Kluwer Health, 2012.
117
Amyloid nephropathy
Disorder is initially assoc. w accumulation of characteristic fibrillar deposits in mesangium. These inert masses, which are
fibrillar by EM, extend along inner surface of basement membrane (BM), frequently obstructing capillary lumen. Focal
extension of amyloid through BM may elevate epithelial cell, in which case irregular spikes are seen along outer surface of BM
Amyloid nephropathy, EM
Deposits of fibrils (10 nm diameter) in a
glomerulus adjacent to podocyte
cytoplasm with effaced foot processes.
Rubin R, Strayer D (eds.) Rubin’s Pathology. Clinicopathologic Foundations of Medicine, 6th ed.
Baltimore: Wolters Kluwer Health, 2012.
Marc Imhotep Cray, MD
Trigger Case 5
118
A 45-year-old white woman with a 15-year history of SLE presents
to the nephrologist after her primary care physician found
hematuria and proteinuria on a routine urinalysis. The patient has
edema of the ankles on physical examination and she is
experiencing a SLE flare-up with an extensive malar rash visible
over her face. A renal biopsy is obtained and examination by light
microscopy reveals wire-loop abnormalities. You examine the
patient’s current immunosuppressive therapy to see what
additional therapies should be added.
What is the Diagnosis?
Marc Imhotep Cray, MD
Lupus Nephropathy
119
Etiology Renal component of systemic lupus erythematosus
Pathology Five distinct renal histologic patterns:
 (1) Type I: normal
 (2) Type II (mesangial form): focal & segmental glomerular involvement w
↑ in mesangial matrix
 (3) Type III (focal proliferative form): involves less than half of glomeruli,
causing extensive damage to individual glomeruli
 (4) Type IV (diffuse proliferative form): most severe form involving all
glomeruli w marked inflammation, mesangial proliferation, and scarring
o LM: wire-loop abnormality caused immune complex deposition and gross
thickening of GBM
o EM: endothelial cell proliferation
o IF: marked subendothelial immune complex deposition= granular pattern
 (5) Type V (membranous form): similar to membranous glomerulonephritis
Marc Imhotep Cray, MD
Lupus Nephropathy cont’d.
120
 Clinical Manifestations
 Type I: No clinical findings
 Types II and III: Mild to moderate proteinuria and hematuria
 Type IV: Combination of nephrotic and nephritic syndromes
 Type V: Nephrotic syndrome
 Tx
 Types I and II: No treatment required
 Types III, IV, and V: Immunosuppression (corticosteroids,
cyclophosphamide, and/or azathioprine)
Note: Renal lesion severity often determines overall prognosis of SLE
patients
121
A, Focal proliferative glomerulonephritis, with two focal necrotizing lesions at the 11 0'clock
and 2 0'clock positions (H&E stain). Extracapillary proliferation is not prominent in this case.
B, Diffuse proliferative glomerulonephritis. Note marked increase in cellularity throughout
glomerulus (H&E stain).
Lupus nephritis
Kumar V, Abbas AK, Aster JC. Robbins and Cotran Pathologic Basis of Disease, 9th ed. Philadelphia: Saunders-Elsevier, 2015.
122Kumar V, Abbas AK, Aster JC. Robbins and Cotran Pathologic Basis of Disease, 9th ed. Philadelphia: Saunders-Elsevier, 2015.
Lupus nephritis
C, Lupus nephritis showing a glomerulus w several 'Wire loop" lesions representing
extensive subendothelial deposits of immune complexes (PSA stain).
D, EM of a renal glomerular capillary loop from a patient w SLE nephritis. Subendothelial
dense deposits (arrowheads) correspond to "wire loops" seen by light microscopy.
B (with arrow) refers to the basement membrane.
123
E, Deposition of lgG antibody in a granular pattern, detected by immunofluorescence.
Lupus nephritis
Kumar V, Abbas AK, Aster JC. Robbins and Cotran Pathologic Basis of Disease,
9th ed. Philadelphia: Saunders-Elsevier, 2015.
Marc Imhotep Cray, MD
Trigger Case 6
124
A 10-year-old girl presents to the clinic complaining of eye
swelling. You note that the child was seen 3 weeks ago in clinic for
a chief complaint of sore throat. Upon taking a history and
performing a physical, you find that the patient has pronounced
periorbital edema, has been urinating very little despite adequate
fluid intake, and has a blood pressure of 150/90. Laboratory
findings include azotemia, hematuria, red cell casts in the urine,
and an elevated ASO antibody titer. You reassure the parents that
their child’s condition will likely resolve on its own.
What is the Diagnosis?
Marc Imhotep Cray, MD
Poststreptococcal Glomerulonephritis
(Acute Proliferative GN/ Acute GN)
125
Etio. & Epidem. Most frequently seen in children (6-10 yrs.)
following infection w nephritogenic strains of group A β-hemolytic
streptococci
 decreasing in frequency in U.S but, fairly common disorder worldwide
Pathology
 Gross : Characterized by intense inflammatory reaction involving all
glomeruli in both kidneys resulting in punctate hemorrhages on kidney
surfaces
 LM: Enlarged, hypercellular, swollen glomeruli w proliferation of
mesangial and endothelial cells; normal GBM thickness
 EM: Electron-dense humps on the epithelial side of the basement
membrane (subepithelial localization)
 IF: Coarse granular immunofluorescence for IgG or C3 (lumpy-bumpy)
Marc Imhotep Cray, MD
Poststreptococcal GN cont’d.
126
Clinical Manifestations Nephritic syndrome & periorbital
edema; pulmonary congestion not uncommon
 Lab findings: UA RBCs and/or red cell casts, decreased serum C3,
elevated ASO antibody titer (evidence of recent streptococcal infection)
Tx & Px
Resolves spontaneously
 More than 95% of affected children eventually recover renal function
w conservative therapy aimed at maintaining sodium and water balance
 In adults disease is less benign in up to 40% affected in epidemics fail
to resolve quickly
Note: Poststreptococcal glomerulonephritis is an immune complex disease w
antigen-antibody complex of streptococcal origin and is prototype of nephritic
syndrome & prototypical glomerular disease of immune complex etiology
127
Nonstreptococcal Acute Glomerulonephritis
(Postinfectious Glomerulonephritis)
A similar form of glomerulonephritis occurs sporadically in assoc. w other
infections, including those of bacterial (e.g., staphylococcal endocarditis,
pneumococcal pneumonia, and meningococcemia), viral (e.g., hepatitis B,
hepatitis C, mumps, HIV infection, varicella, and infectious mononucleosis),
and parasitic (malaria, toxoplasmosis) origin
 In these settings, granular immunofluorescent deposits and subepithelial
humps characteristic of immune complex nephritis are present
 Postinfectious glomerulonephritis due to staphylococcal infections differs
by sometimes producing immune deposits containing IgA rather than IgG
128
A Normal glomerulus.
B Glomerular hypercellularity is
due to intracapillary leukocytes and
proliferation of intrinsic glomerular
cells.
C Typical electron-dense
subepithelial “hump” and a
neutrophil in lumen.
D Immunofluorescent stain
demonstrates discrete, coarsely
granular deposits of complement
protein C3 (stain for IgG was
similar), corresponding to “humps”
illustrated in part C.
Acute proliferative
glomerulonephritis
Kumar V, Abbas AK, Aster JC. Robbins and Cotran Pathologic Basis of Disease, 9th ed. Philadelphia:
Saunders-Elsevier, 2015.
Marc Imhotep Cray, MD
Trigger Case 7
129
A 40-year-old man is admitted to the hospital with complaints of
blood in his sputum and urine. A thorough history also reveals
fever, malaise, and a 10-pound weight loss over the past month.
On physical examination, you find that his blood pressure is
160/95 and that he has several abnormal lung sounds. A urine
dipstick demonstrates hematuria. CXR reveals several nodular
lesions and blood tests show the presence of C-ANCA and an
elevated ESR. You start the patient on a high dose of
corticosteroids and you suspect that a renal biopsy would
demonstrate crescent moon shapes between the Bowman
capsule and the glomerular tuft.
What is the Diagnosis?
130
Rapidly Progressive (Crescentic) Glomerulonephritis
 RPGN is a clinical syndrome assoc. w severe glomerular injury, but does
not denote a specific etiologic form of GN
 Type 1=20% (Anti-GBM antibody-mediated disease); Idiopathic or Goodpasture
syndrome (=Ab against BM lung alveoli hemoptysis)
 Type 2=25% (immune complexes): Idiopathic; postinfectious GN; SLE; IgA
nephropathy; Henoch- Schönlein purpura
 Type 3=55% (pauci-immune type): Idiopathic; ANCA-assoc. vasculitides= Wegener
granulomatosis; microscopic polyangiitis etc.
Pathology
 LM: Formation of crescent-moon shape betw. Bowman capsule and glomerular tuft
resulting from deposition of fibrin in Bowman space and proliferation of parietal
epithelial cells of Bowman capsule
 IF: Type 1, linear pattern deposits of IgG & C3; Type 2=granular deposits IgG &C3
Marc Imhotep Cray, MD
RPGN cont’d.
131
Clinical Manifestations
 Nephritic syndrome progressing rapidly to renal failure
within months
 Sn & Sx specific to each etiology (eg, hemoptysis and anti-
GBM antibodies in Goodpasture syndrome)
Tx & Px
 Treat w diuretics and often eventual dialysis
 immunosuppression if appropriate for underlying cause
 may require transplantation
 Rapid course to renal failure
Note: RPGN refers to a syndrome assoc. w severe and progressive glomerular
injury It encompasses many different etiologies
Marc Imhotep Cray, MD
RPGN Schematic
132
 Rapidly proliferating epithelial
crescent crushes glomerular
tuft, which may show a range
of changes including
o focal proliferative GN,
o segmental tuft necrosis,
o mesangiocapillary GN, or
o acute proliferative GN
Note: An epithelial crescent is result of
breaks in GBM permitting leak of fibrin,
blood proteins and WBCs (particularly
monocyte/ macrophages) into urinary space
Stevens A, Lowe J, Scott I. Core Pathology, 3rd Ed. St. Louis: Mosby-
Elsevier, 2009.
133
Rapidly progressive glomerulonephritis, microscopic
Klatt EC. Robbins and Cotran Atlas of Pathology, 3rd Ed. Philadelphia: Saunders, 2015.
Marc Imhotep Cray, MD
RPGN, Electron
micrograph
134
EM showing characteristic wrinkling of GBM w focal
disruptions (arrows).
Kumar V, Abbas AK, Aster JC. Robbins and Cotran Pathologic Basis of
Disease, 9th ed. Philadelphia: Saunders-Elsevier, 2015.
135
RPGN, immunofluorescence
Klatt EC. Robbins and Cotran Atlas of Pathology, 3rd Ed. Philadelphia: Saunders, 2015.
Marc Imhotep Cray, MD
Trigger Case 8
136
A 10-year-old boy brought to the clinic by his mother
complaining of a red tinge to his urine. A more detailed history
reveals that he was diagnosed with mild nerve deafness 2 years
earlier and also developed posterior cataracts 1 year ago.
Laboratory studies confirm hematuria as well as the presence of
erythrocyte casts. You begin to wonder if his conditions may be
related to a genetic disorder.
What is the Diagnosis?
Marc Imhotep Cray, MD
Alport Syndrome
137
 Etiology: Genetic disorder w heterogenous inheritance (usually
X-linked dominant) results in mutation of α-5 chain of type IV
collagen
 caused by mutations in COL4A3, COL4A4, and COL4A5, three of six
genes involved in basement membrane (type IV) collagen biosynthesis
 Pathology EM: Irregular foci of thickening or attenuation in
GBM with longitudinal splitting of lamina densa
 Clinical Manifestations: Triad of nephritis, nerve deafness, and
various eye disorders (cataracts, lens dislocation, corneal
dystrophy)
 often initially presents w hematuria and RBC casts
 Treatment ACE inhibitors; renal transplantation
138
Alport syndrome, microscopic
Klatt EC. Robbins and Cotran Atlas of Pathology, 3rd Ed. Philadelphia: Saunders, 2015.
Marc Imhotep Cray, MD
Trigger Case 9
139
A 25-year-old woman with a history of SLE is admitted to the
hospital with generalized edema, malaise, and fatigue. You take a
thorough history, which reveals that she had cold symptoms
2 weeks earlier. Laboratory studies show hypoalbuminemia,
hypercholesterolemia, proteinuria, and low complement levels.
When a renal biopsy shows reduplication of the basement
membrane on electron microscopy, you adjust the patient’s
current corticosteroid dose and decide to add an antiplatelet
drug to her regimen.
What is the Diagnosis?
Marc Imhotep Cray, MD
Diffuse Membranoproliferative GN
(Mesangiocapillary GN)
140
Etio. and Epidem. Assoc. w inherited complement component
deficiency
 Type I MPGN (90%) is seen in SLE, hepatitis B and C, and involves classic
& alternative pathway activation
 Type II MPGN (10%) (dense deposit disease) dysregulation of alternative
complement pathway
 Most patients are under the age of 30
 Pathology Types I and II:
 LM: reduplication of basement membrane (splitting) and
lobular proliferation of mesangial matrix into capillary loops
(tram track appearance)
 Type I: EM: subendothelial electron-dense deposits
 Type II: EM: dense deposit of homogeneous material within GBM
Marc Imhotep Cray, MD
Membranoproliferative GN cont’d.
141
Clinical Manifestations
 Type I: Commonly presents w nephrotic syndrome
 Type II: Commonly presents w hematuria and chronic renal failure
 Lab Findings: Low complement levels (↓ C3) , ↑BUN and Cr, RBCs
and/or RBC casts in urine
 Tx & Px: Corticosteroids (poor response) and
immunosuppression if appropriate for underlying cause
 Type II, particularly, poor prognosis slowly progresses to CKD 
50% develop CKD within 10 year
 Both types have high incidence of recurrence of disease in
transplanted kidneys
Marc Imhotep Cray, MD
Schematic representation of MPGN
142
Patterns in two types of MPGN
 In type I there are subendothelial
deposits
 Type II is characterized by
intramembranous dense deposits
(=dense-deposit disease)
 In both, BM appear split when
viewed in light microscope
Kumar V, Abbas AK, Aster JC. Robbins and Cotran Pathologic Basis of
Disease, 9th ed. Philadelphia: Saunders-Elsevier, 2015.
143
MPGN, microscopic
Klatt EC. Robbins and Cotran Atlas of Pathology, 3rd Ed. Philadelphia: Saunders, 2015.
Marc Imhotep Cray, MD
A MPGN, type I. Note discrete electron-dense deposits (arrows) incorporated into glomerular capillary
wall betw. duplicated (split) basemen membranes (double arrows), and in mesangial regions (M); CL,
Capillary lumen.
B Dense-deposit disease (type II MPGN). There are dense homogeneous deposits within basement
membrane. In both, mesangial interposition gives appearance of split BM when viewed in LM
144
Kumar V, Abbas AK, Aster JC. Robbins and Cotran Pathologic Basis of Disease, 9th ed. Philadelphia: Saunders-Elsevier, 2015.
Marc Imhotep Cray, MD
Question
145
A 26-year-old woman with a history of mitral valve prolapse
comes in with 1 week of fever that started 3 days after a dental
procedure. Her urine contains red cells and her rheumatoid
factor is elevated. Which of the following serologic abnormalities
is expected to be present?
A. Anti-GBM antibody
B. Low serum complement levels
C. Antineutrophil cytoplasmic antibody
D. Elevated IgA levels
Marc Imhotep Cray, MD
Answer
146
The answer is B. (Robbins, 9th/e, p 926.) This clinical description is
classic for membranoproliferative glomerulonephritis, which is
associated with bacterial endocarditis. Low complement levels are
typically found in membranoproliferative glomerulonephritis, but
not in the other disorders listed.
Choice A. describes anti-GBM antibody disease or Goodpasture’s
syndrome when the lung is involved.
Choice C. describes pauci-immune glomerulonephritis, such as
Wegener’s.
Choice D. describes IgA nephropathy.
Marc Imhotep Cray, MD
Trigger Case 10
147
A 15-year-old Asian boy presents to the emergency room
complaining of blood in his urine. Upon taking a complete history,
you learn that he has also been suffering from fevers, myalgias,
and arthralgias for the last 2 days. Serum studies reveal increased
serum IgA levels and normal serum complement levels. You begin
him on prednisone and you suspect that he is afflicted with the
most common form of acute glomerulonephritis in the United
States.
What is the Diagnosis?
Marc Imhotep Cray, MD
IgA Nephropathy (Berger Disease)
148
Etio. & Epidem. Primary renal disease of IgA deposition in
glomerular mesangium  can manifest after infection (viral
URI, GI infection, flu-like syndrome) or can be a component of
Henoch-Schönlein purpura
 Most commonly seen in children and young adults w men
affected more often than women
Pathology
 LM: mesangial widening and focal and segmental
inflammation
 EM: Mesangial deposits of IgA
Marc Imhotep Cray, MD
IgA Nephropathy (Berger Disease) cont’d.
149
Clinical Manifestations Presents w recurrent
hematuria (red or cola-colored urine) 1–2 days after an
infection
 Lab findings: ↑ serum IgA level (50% of cases), nml serum
complement levels
Tx & Px Prednisone
 Can progress to chronic renal failure 25-50% go on to
develop CKD within 20 yrs.
Note: Berger disease is most common form of acute GN in U.S. and
worldwide
150
IgA nephropathy (Berger disease), microscopic
Klatt EC. Robbins and Cotran Atlas of Pathology, 3rd Ed. Philadelphia: Saunders, 2015.
151
IgA nephropathy, immunofluorescence
Klatt EC. Robbins and Cotran Atlas of Pathology, 3rd Ed. Philadelphia: Saunders, 2015.
152Rubin R, Strayer D (eds.) Rubin’s Pathology. Clinicopathologic Foundations of Medicine, 6th ed. Baltimore: Wolters Kluwer Health, 2012.
Algorithm demonstrating integration of pathologic findings with clinical data to
make a diagnosis of a specific form of primary or secondary glomerulonephritis.
Nephritic syndrome—due to GBM disruption.
Hypertension, ↑ BUN and creatinine, oliguria,
hematuria, RBC casts in urine. Proteinuria often
in the subnephrotic range (< 3.5 g/day) but in
severe cases may be in nephrotic range.
• Acute poststreptococcal glomerulonephritis
• Rapidly progressive glomerulonephritis
• IgA nephropathy (Berger disease)
• Alport syndrome
• Membranoproliferative glomerulonephritis
Nephritic-nephrotic syndrome—severe nephritic syndrome
with profound GBM damage that damages the glomerular
filtration charge barrier → nephrotic-range proteinuria (> 3.5
g/day) and concomitant features of nephrotic syndrome. Can
occur with any form of nephritic syndrome, but is most
commonly seen with:
• Diffuse proliferative glomerulonephritis
• Membranoproliferative glomerulonephritis
• Lupus Nephropathy Type IV
Nephrotic syndrome—podocyte disruption
→ charge barrier impaired. Massive proteinuria
(> 3.5 g/day) with hypoalbuminemia,
hyperlipidemia, edema.
May be 1° (eg, direct podocyte damage) or 2° (podocyte
damage from systemic process [eg, diabetes]).
• Focal segmental glomerulosclerosis (1° or 2°)
• Minimal change disease (1° or 2°)
• Membranous nephropathy (1° or 2°)
• Amyloidosis (2°)
• Diabetic glomerulonephropathy (2°)
Glomerular diseases capsule
0.25 3.5 > 3.5
GRAMS OF PROTEIN EXCRETED PER DAY (g/day)
Marc Imhotep Cray, MD
Lecture 3:
Urinary Tract Infections
Tubulointerstitial diseases
Obstructive uropathy, hydronephrosis &
urolithiasis
154
Marc Imhotep Cray, MD
Urinary Tract Infections
155
 A urinary tract infection (UTI) is an infection that affects
part of urinary tract
 When it affects lower urinary tract it is known as a bladder
infection (cystitis)
 When it affects upper urinary tract it is known as kidney
infection (pyelonephritis [discussed under tubulointerstitial
diseases] )
 Cystitis
 Clinical features: dysuria in absence of vaginal discharge and
significant pain, frequency, urgency, suprapubic pain
Marc Imhotep Cray, MD
UTI, Cystitis cont’d.
156
Etio-pathogenesis:
 Bacteria gain access to urinary tract via urethra
 Cystitis most frequently involves normal colonic flora
o Escherichia coli is most common cause (approx. 80%)
o Proteus, Klebsiella, and Enterobacter are also implicated
o Staphylococcus saprophyticus causes 10% to 15% of
infections in young women
o Nosocomial cystitis is frequently caused by
Pseudomonas or Staphylococcus aureus
Marc Imhotep Cray, MD
UTI, Cystitis cont’d.
157
 Epidemiology
 Women have a higher incidence of infection b/c they have
shorter urethras
 Other risk factors include sexual activity, pregnancy, urinary
obstruction, neurogenic bladder, and vesicoureteral reflux-8
Diagnostic findings
 Characteristic clinical features are present
 Pyuria (more than 8 leukocytes/high-power field)
 Bacterial culture yields >105 organisms/mL
Treatment
 antibiotics
 Recurrent cystitis may require prophylactic antibiotics
158
Acute cystitis
Patient died 2 days after surgery, and
cystitis was caused by an indwelling
catheter
A. Several foci of hemorrhage are
seen on hyperemic bladder mucosa
B. Foci of mucosal hemorrhage
C. Acute cystitis. Polymorphonuclear
leukocytes infiltrate mucosa
Rubin R , Strayer DS Eds. Rubin’s Pathology: Clinicopathologic Foundations of Medicine,
6th Ed. Baltimore: Lippincott Williams & Wilkins, 2012.
Marc Imhotep Cray, MD
Tubulointerstitial Diseases
159
Most forms of tubular injury also involve interstitium so they
are discussed together= tubulointerstitial diseases
Diseases involving tubules and interstitium may have clinical
manifestations of nephritic syndrome, or of specific defects in
tubular function, or of acute or chronic kidney disease without
more specific defining features
In this section we will discuss diseases characterized by:
1. Inflammatory involvement of tubules and interstitium (tubulointerstitial
nephritis)
 Acute and chronic pyelonephritis (Upper UTI)
2. Ischemic and toxic tubular injury leading to acute tubular injury and
clinical syndrome of acute kidney injury (ARF)
Marc Imhotep Cray, MD
Tubulointerstitial Diseases cont’d.
160
 Acute tubular necrosis (ATN)-now called acute tubular injury
(ATI)- is usually due to profound hypotension causing
ischemic damage to tubular epithelial cells
 Infections include pyelonephritis, renal abscesses & TB
 Drug toxicity usually causes tubulointerstitial nephritis due to
a hypersensitivity reaction, but
 other patterns may occur, including direct toxicity to tubular
epithelial cells giving an appearance similar to ATN
 Mechanical obstruction of ureters or bladder may lead to
hydronephrosis and recurrent infection
Marc Imhotep Cray, MD
Tubulointerstitial Nephritis
161
Tubulointerstitial nephritis (TIN) refers to a group of
inflammatory kidney diseases that primarily involve interstitium
& tubules
 glomeruli may be spared altogether or affected only late in
course
In cases of TIN caused by bacterial infection renal pelvis is
prominently involved hence a more descriptive term is
pyelonephritis (from pyelo, “pelvis”)
Marc Imhotep Cray, MD
Tubulointerstitial Nephritis cont’d.
162
Term interstitial nephritis is reserved for cases of TIN that are
nonbacterial in origin includes
 tubular injury resulting from drugs
 metabolic disorders such as hypokalemia
 irradiation
 viral infections, and
 immune reactions
o For example, acute drug-induced interstitial nephritis caused by
penicillin derivatives (eg, methicillin), NSAIDs, and diuretics
On basis of clinical manifestations & character of inflammatory
exudate TIN can be divided into acute and chronic categories
Marc Imhotep Cray, MD
Trigger Case 11
163
A 25-year-old woman presents to the emergency room with
fever, severe flank pain, and costovertebral angle tenderness.
After taking a complete history, you find that she is sexually
active and has had a 2-week history of burning pain while
urinating and increased urinary frequency. Urinalysis reveals
white cell casts in the urine and a urine sample is sent for culture.
While you await the culture results, you start her on broad
spectrum antibiotics.
What is the Diagnosis?
Marc Imhotep Cray, MD
Pyelonephritis (Acute and Chronic)
164
 Etio. & Epidem.
 Acute: Caused by infection of renal parenchyma more frequent
among women
 Chronic: Results from chronic urinary tract obstruction & recurrent UTIs
 Pathology
Acute: Affects renal cortex w sparing of glomeruli
 neutrophilic infiltration and abscess formation within renal interstitium
o abscesses may rupture introducing WBCs into tubular lumen
Chronic: Asymmetric corticomedullary scarring
 tubules contain eosinophilic, proteinaceous casts resulting in gross
appearance reminiscent of thyroid follicles (thyroidization of kidneys)
 in later stages results in tubular atrophy and interstitial fibrosis
Marc Imhotep Cray, MD
Acute pyelonephritis, gross
165
 Note in this plate that cortical surface is
studded w focal pale abscesses, more
numerous in upper pole and middle
region of kidney, lower pole is relatively
unaffected
 Betw. abscesses, there is dark
congestion of renal surface
Kumar V, Abbas AK, Aster JC. Robbins and Cotran Pathologic
Basis of Disease, 9th ed. Philadelphia: Saunders-Elsevier, 2015.
Marc Imhotep Cray, MD
Acute pyelonephritis, microscopic
166
 An extensive infiltrate of neutrophils
is present in collecting tubules and
interstitial tissue
Rubin R , Strayer DS Eds. Rubin’s Pathology: Clinicopathologic Foundations
of Medicine, 6th Ed. Baltimore: Lippincott Williams & Wilkins, 2012.
Marc Imhotep Cray, MD
Pyelonephritis cont’d.
167
Pathogenesis Principal causative organisms in acute
pyelonephritis are enteric gram-negative bacilli
 Escherichia coli is most common
 Other important organisms are Proteus, Klebsiella, Enterobacter, and
Pseudomonas these usually are assoc. w recurrent infections,
especially in individuals who undergo urinary tract manipulations or
have congenital or acquired anomalies of lower urinary tract
Bacteria can reach kidneys from lower urinary tract (ascending
infection) or through bloodstream (hematogenous infection)
 N.B. Ascending infection from lower urinary tract is most important
& frequent route by which bacteria reach kidney
Marc Imhotep Cray, MD
Pathways of renal infection
168
 Hematogenous infection results
from bacteremic spread
 ascending infection results from
a combination of
o urinary bladder infection,
o vesicoureteral reflux, and
o intrarenal reflux
Kumar V, Abbas AK, Aster JC. Robbins and Cotran Pathologic
Basis of Disease, 9th ed. Philadelphia: Saunders-Elsevier, 2015.
Marc Imhotep Cray, MD
169
Buja LM, Krueger GR. Netter’s Illustrated Human Pathology, 2nd Ed. Philadelphia: Saunders-Elsevier, 2014.
Marc Imhotep Cray, MD
170
Vesicoureteral reflux demonstrated
by a voiding
 Dye injected into bladder refluxes
into both dilated ureters, filling
pelvis and calyces
 In absence of vesicoureteral
reflux, infection remains localized
in bladder
o majority of individuals w repeated
or persistent bacterial colonization
of urinary tract suffer from cystitis
and urethritis (lower UTI) rather
than pyelonephritis
Kumar V, Abbas AK, Aster JC. Robbins and Cotran Pathologic
Basis of Disease, 9th ed. Philadelphia: Saunders-Elsevier, 2015.
Marc Imhotep Cray, MD
Pyelonephritis cont’d.
171
 Clinical Manifestations
 Acute: Fever; flank pain with CVA tenderness; polyuria
and dysuria; nausea, vomiting, and diarrhea
 Chronic: Recurrent episodes of acute pyelonephritis can
lead to renal hypertension and ESRD
 Lab findings: Leukocytosis, WBC and/or WBC casts in urine
Treatment
Acute: IV antibiotics
Chronic: Renal transplantation if progresses to ESRD
172
Chronic pyelonephritis, gross
Rubin R , Strayer DS Eds. Rubin’s Pathology: Clinicopathologic Foundations of Medicine,
6th Ed. Baltimore: Lippincott Williams & Wilkins, 2012.
 A Cortical surface contains many
irregular, depressed scars
(reddish areas)
 B marked dilation of calyces
(caliectasis) caused by
inflammatory destruction of
papillae, w atrophy and scarring
of overlying cortex
Marc Imhotep Cray, MD
Chronic pyelonephritis, microscopic
173
Rubin R , Strayer DS Eds. Rubin’s Pathology: Clinicopathologic Foundations
of Medicine, 6th Ed. Baltimore: Lippincott Williams & Wilkins, 2012.
 A LM shows tubular
dilation & atrophy, w many
tubules containing
eosinophilic hyaline casts
resembling colloid of
thyroid follicles (so-called
thyroidization)
 interstitium is scarred and
contains a chronic
inflammatory cell infiltrate
Marc Imhotep Cray, MD
Two major types of chronic pyelonephritis:
Left. Vesicoureteral reflux causes infection of peripheral papillae and, therefore,
scars in poles of kidney
Right. Obstruction of urinary tract leads to high pressure backflow of urine,
causes infection of all papillae, diffuse scarring of kidney & thinning of cortex
174
Rubin R , Strayer DS Eds. Rubin’s Pathology: Clinicopathologic Foundations of Medicine, 6th Ed.
Baltimore: Lippincott Williams & Wilkins, 2012.
Marc Imhotep Cray, MD
Reflux nephropathy [vesicoureteral reflux]
175
N.B. Most common mechanism in pathogenesis of chronic
pyelonephritis is reflux nephropathy
 Reflux nephropathy is kidney damage (nephropathy) due to urine
flowing backward (reflux) from bladder toward kidneys also called
vesicoureteral reflux (VUR)
 Longstanding VUR can result in small and scarred kidneys during first five years of
life in affected children
 End results of reflux nephropathy can include high blood pressure, excessive
protein loss in urine, and eventually kidney failure
 When reflux nephropathy is suspected as a cause of kidney disease,
other conditions to consider include chronic pyelonephritis, obstructive
uropathy, and analgesic overuse
Marc Imhotep Cray, MD
Renal papillary necrosis & Diffuse cortical
necrosis
176
Important notes:
 Renal papillary necrosis is a complication of acute
pyelonephritis in diabetics or analgesics particularly those
ingested at least 2 kg in past (=analgesic nephropathy)
 risk is higher for phenacetin (withdrawn from market in U.S.) and
acetaminophen compared to aspirin and other NSAIDs
 Diffuse cortical necrosis is an acute generalized infarction of
renal cortices (medulla is spared)usually b/c of a
combination of DIC and end-organ vasospasm in assoc. w
obstetric catastrophes or septic shock
Marc Imhotep Cray, MD
Papillary necrosis
177
 Bisected kidney shows a
dilated renal pelvis and dilated
calyces secondary to urinary
tract obstruction
 Papillae are all necrotic and
appear as sharply demarcated,
ragged, yellowish areas
Rubin R , Strayer DS Eds. Rubin’s Pathology: Clinicopathologic Foundations
of Medicine, 6th Ed. Baltimore: Lippincott Williams & Wilkins, 2012.
Mnemonic for causes of renal papillary necrosis is POSTCARDS:
pyelonephritis, obstruction of urogenital tract, sickle cell disease,
tuberculosis, cirrhosis, analgesia/alcohol abuse, renal vein
thrombosis, diabetes mellitus, and systemic vasculitis
Marc Imhotep Cray, MD
Renal cortical necrosis
178
 Cortex of kidney is pale yellow and
soft due to diffuse cortical necrosis
Rubin R , Strayer DS Eds. Rubin’s Pathology:
Clinicopathologic Foundations of Medicine, 6th Ed.
Baltimore: Lippincott Williams & Wilkins, 2012.
 Usually caused by significantly
diminished arterial perfusion of
kidneys due to spasms of feeding
arteries, microvascular injury, or DIC
 Renal cortical necrosis is the
pathological progression of acute
tubular necrosis
Marc Imhotep Cray, MD
Case Trigger 12
179
A 68-year-old man presents to the emergency room because he is
unable to urinate. After taking a detailed history, you learn that
he has had increasing urinary hesitancy and decreased force of
his urine stream for several months. He also complains of a
sensation of incomplete bladder emptying. On rectal
examination, you find a smooth, firm, elastic enlargement of the
prostate. Examination of the lower abdomen reveals signs of a
distended bladder. You order laboratory studies that show
increased urinary sodium excretion and an elevated BUN and
creatinine. You determine that the patient requires prompt
urethral catheterization to help reverse his renal failure.
What is the Diagnosis?
Marc Imhotep Cray, MD
Acute Renal Failure (Prerenal, Intrarenal, and
Postrenal Azotemia)
180
 Etiology
 Prerenal: Caused by decreased effective arterial volume (ie, CHF,
hypovolemia, systemic vasodilation [sepsis]), or renal vasoconstriction
(NSAIDs, ACE inhibitors, RAS)
 Intrarenal: Caused by acute tubular necrosis, acute interstitial nephritis,
glomerulonephritis, and thrombotic microangiopathy
 Postrenal: Caused by kidney stones, BPH, neurogenic bladder, and
neoplasia
 Pathology and Pathophysiology
 Prerenal: Renal hypoperfusion leads to decreased GFR resulting in
sodium and water retention
 Intrarenal: Characterized by patchy tubular necrosis leads to tubule
obstruction and fluid backflow across necrotic tubule and a resulting
decrease in GFR
 Postrenal: Only develops w bilateral outflow obstruction
Marc Imhotep Cray, MD
Acute Kidney Injury (AKI) [previously ARF]
181
 An acute rise in serum Cr has been called ARF
 It is classified as: prerenal, intrarenal, postrenal azotemia
Clinical Manifestations:
 oliguria; azotemia; hyperkalemia
 Intrarenal ARF is now called acute kidney injury (AKI) AKI is
categorized by portion of kidney that is primarily injured:
 glomeruli (e.g., acute glomerulonephritis)
 vessels (e.g., vasculitis),
 tubules (e.g., ischemic acute tubular injury (previously ATN) or
 interstitium (acute interstitial nephritis)
 most common cause intrarenal AKI is ischemic acute tubular injury
 most common cause of AKI overall is therapeutic drugs
Renal Pathology Lectures_Ppt Series (4 in 1)
Renal Pathology Lectures_Ppt Series (4 in 1)
Renal Pathology Lectures_Ppt Series (4 in 1)
Renal Pathology Lectures_Ppt Series (4 in 1)
Renal Pathology Lectures_Ppt Series (4 in 1)
Renal Pathology Lectures_Ppt Series (4 in 1)
Renal Pathology Lectures_Ppt Series (4 in 1)
Renal Pathology Lectures_Ppt Series (4 in 1)
Renal Pathology Lectures_Ppt Series (4 in 1)
Renal Pathology Lectures_Ppt Series (4 in 1)
Renal Pathology Lectures_Ppt Series (4 in 1)
Renal Pathology Lectures_Ppt Series (4 in 1)
Renal Pathology Lectures_Ppt Series (4 in 1)
Renal Pathology Lectures_Ppt Series (4 in 1)
Renal Pathology Lectures_Ppt Series (4 in 1)
Renal Pathology Lectures_Ppt Series (4 in 1)
Renal Pathology Lectures_Ppt Series (4 in 1)
Renal Pathology Lectures_Ppt Series (4 in 1)
Renal Pathology Lectures_Ppt Series (4 in 1)
Renal Pathology Lectures_Ppt Series (4 in 1)
Renal Pathology Lectures_Ppt Series (4 in 1)
Renal Pathology Lectures_Ppt Series (4 in 1)
Renal Pathology Lectures_Ppt Series (4 in 1)
Renal Pathology Lectures_Ppt Series (4 in 1)
Renal Pathology Lectures_Ppt Series (4 in 1)
Renal Pathology Lectures_Ppt Series (4 in 1)
Renal Pathology Lectures_Ppt Series (4 in 1)
Renal Pathology Lectures_Ppt Series (4 in 1)
Renal Pathology Lectures_Ppt Series (4 in 1)
Renal Pathology Lectures_Ppt Series (4 in 1)
Renal Pathology Lectures_Ppt Series (4 in 1)
Renal Pathology Lectures_Ppt Series (4 in 1)
Renal Pathology Lectures_Ppt Series (4 in 1)
Renal Pathology Lectures_Ppt Series (4 in 1)
Renal Pathology Lectures_Ppt Series (4 in 1)
Renal Pathology Lectures_Ppt Series (4 in 1)
Renal Pathology Lectures_Ppt Series (4 in 1)
Renal Pathology Lectures_Ppt Series (4 in 1)
Renal Pathology Lectures_Ppt Series (4 in 1)
Renal Pathology Lectures_Ppt Series (4 in 1)
Renal Pathology Lectures_Ppt Series (4 in 1)
Renal Pathology Lectures_Ppt Series (4 in 1)
Renal Pathology Lectures_Ppt Series (4 in 1)
Renal Pathology Lectures_Ppt Series (4 in 1)
Renal Pathology Lectures_Ppt Series (4 in 1)
Renal Pathology Lectures_Ppt Series (4 in 1)
Renal Pathology Lectures_Ppt Series (4 in 1)
Renal Pathology Lectures_Ppt Series (4 in 1)
Renal Pathology Lectures_Ppt Series (4 in 1)
Renal Pathology Lectures_Ppt Series (4 in 1)
Renal Pathology Lectures_Ppt Series (4 in 1)
Renal Pathology Lectures_Ppt Series (4 in 1)
Renal Pathology Lectures_Ppt Series (4 in 1)
Renal Pathology Lectures_Ppt Series (4 in 1)
Renal Pathology Lectures_Ppt Series (4 in 1)
Renal Pathology Lectures_Ppt Series (4 in 1)
Renal Pathology Lectures_Ppt Series (4 in 1)
Renal Pathology Lectures_Ppt Series (4 in 1)
Renal Pathology Lectures_Ppt Series (4 in 1)
Renal Pathology Lectures_Ppt Series (4 in 1)
Renal Pathology Lectures_Ppt Series (4 in 1)
Renal Pathology Lectures_Ppt Series (4 in 1)
Renal Pathology Lectures_Ppt Series (4 in 1)
Renal Pathology Lectures_Ppt Series (4 in 1)
Renal Pathology Lectures_Ppt Series (4 in 1)
Renal Pathology Lectures_Ppt Series (4 in 1)
Renal Pathology Lectures_Ppt Series (4 in 1)
Renal Pathology Lectures_Ppt Series (4 in 1)
Renal Pathology Lectures_Ppt Series (4 in 1)
Renal Pathology Lectures_Ppt Series (4 in 1)
Renal Pathology Lectures_Ppt Series (4 in 1)
Renal Pathology Lectures_Ppt Series (4 in 1)
Renal Pathology Lectures_Ppt Series (4 in 1)
Renal Pathology Lectures_Ppt Series (4 in 1)
Renal Pathology Lectures_Ppt Series (4 in 1)
Renal Pathology Lectures_Ppt Series (4 in 1)
Renal Pathology Lectures_Ppt Series (4 in 1)
Renal Pathology Lectures_Ppt Series (4 in 1)
Renal Pathology Lectures_Ppt Series (4 in 1)
Renal Pathology Lectures_Ppt Series (4 in 1)
Renal Pathology Lectures_Ppt Series (4 in 1)
Renal Pathology Lectures_Ppt Series (4 in 1)
Renal Pathology Lectures_Ppt Series (4 in 1)
Renal Pathology Lectures_Ppt Series (4 in 1)
Renal Pathology Lectures_Ppt Series (4 in 1)
Renal Pathology Lectures_Ppt Series (4 in 1)

Weitere ähnliche Inhalte

Was ist angesagt?

The KIDNEY - PATHOGENESIS OF GLOMERULAR DISEASES
The KIDNEY -  PATHOGENESIS OF GLOMERULAR DISEASESThe KIDNEY -  PATHOGENESIS OF GLOMERULAR DISEASES
The KIDNEY - PATHOGENESIS OF GLOMERULAR DISEASESDr. Roopam Jain
 
Renal pathology..at glance
Renal pathology..at glanceRenal pathology..at glance
Renal pathology..at glanceraj kumar
 
Renal Pathology
Renal PathologyRenal Pathology
Renal Pathologysantusan
 
Cystic diseases of kidney
Cystic diseases of kidney Cystic diseases of kidney
Cystic diseases of kidney Usman Shams
 
Renal pathology lecture5 Nephrotic & Nephritic syndrome. Sufia Husain 2020
Renal pathology lecture5 Nephrotic & Nephritic syndrome. Sufia Husain 2020Renal pathology lecture5 Nephrotic & Nephritic syndrome. Sufia Husain 2020
Renal pathology lecture5 Nephrotic & Nephritic syndrome. Sufia Husain 2020Sufia Husain
 
Glomerular diseases
Glomerular diseasesGlomerular diseases
Glomerular diseasesLeena Hafeez
 
Pathology of Lower Urinary Tract & Male Genital System
Pathology of Lower Urinary Tract & Male Genital SystemPathology of Lower Urinary Tract & Male Genital System
Pathology of Lower Urinary Tract & Male Genital SystemNilay Nishith
 
The Kidney - GLOMERULAR DISEASES
The Kidney - GLOMERULAR DISEASESThe Kidney - GLOMERULAR DISEASES
The Kidney - GLOMERULAR DISEASESDr. Roopam Jain
 
Tubulointerstitial diseases of Kidney
Tubulointerstitial diseases of KidneyTubulointerstitial diseases of Kidney
Tubulointerstitial diseases of KidneyUsman Shams
 
Renal Histo-Pathology (II) - Normal Kidney Electron Microscopy - Dr. Gawad
Renal Histo-Pathology (II) - Normal Kidney Electron Microscopy - Dr. GawadRenal Histo-Pathology (II) - Normal Kidney Electron Microscopy - Dr. Gawad
Renal Histo-Pathology (II) - Normal Kidney Electron Microscopy - Dr. GawadNephroTube - Dr.Gawad
 

Was ist angesagt? (20)

The KIDNEY - PATHOGENESIS OF GLOMERULAR DISEASES
The KIDNEY -  PATHOGENESIS OF GLOMERULAR DISEASESThe KIDNEY -  PATHOGENESIS OF GLOMERULAR DISEASES
The KIDNEY - PATHOGENESIS OF GLOMERULAR DISEASES
 
Renal pathology..at glance
Renal pathology..at glanceRenal pathology..at glance
Renal pathology..at glance
 
Renal Pathology
Renal PathologyRenal Pathology
Renal Pathology
 
Glomerular diseases
Glomerular diseases Glomerular diseases
Glomerular diseases
 
Diseases of the kidney
Diseases of the kidneyDiseases of the kidney
Diseases of the kidney
 
Renal amyloidosis
Renal amyloidosisRenal amyloidosis
Renal amyloidosis
 
Glomerular diseases
Glomerular diseasesGlomerular diseases
Glomerular diseases
 
Pathology of Glomerulonephritis
Pathology of GlomerulonephritisPathology of Glomerulonephritis
Pathology of Glomerulonephritis
 
Glomerular disease
Glomerular diseaseGlomerular disease
Glomerular disease
 
Cystic diseases of kidney
Cystic diseases of kidney Cystic diseases of kidney
Cystic diseases of kidney
 
Gn csbrp
Gn csbrpGn csbrp
Gn csbrp
 
Renal pathology lecture5 Nephrotic & Nephritic syndrome. Sufia Husain 2020
Renal pathology lecture5 Nephrotic & Nephritic syndrome. Sufia Husain 2020Renal pathology lecture5 Nephrotic & Nephritic syndrome. Sufia Husain 2020
Renal pathology lecture5 Nephrotic & Nephritic syndrome. Sufia Husain 2020
 
Glomerular diseases
Glomerular diseasesGlomerular diseases
Glomerular diseases
 
glomerular disease
glomerular diseaseglomerular disease
glomerular disease
 
Pathology of Lower Urinary Tract & Male Genital System
Pathology of Lower Urinary Tract & Male Genital SystemPathology of Lower Urinary Tract & Male Genital System
Pathology of Lower Urinary Tract & Male Genital System
 
The Kidney - GLOMERULAR DISEASES
The Kidney - GLOMERULAR DISEASESThe Kidney - GLOMERULAR DISEASES
The Kidney - GLOMERULAR DISEASES
 
Tubulointerstitial diseases of Kidney
Tubulointerstitial diseases of KidneyTubulointerstitial diseases of Kidney
Tubulointerstitial diseases of Kidney
 
Diabetic nephropathy
Diabetic nephropathyDiabetic nephropathy
Diabetic nephropathy
 
Renal Histo-Pathology (II) - Normal Kidney Electron Microscopy - Dr. Gawad
Renal Histo-Pathology (II) - Normal Kidney Electron Microscopy - Dr. GawadRenal Histo-Pathology (II) - Normal Kidney Electron Microscopy - Dr. Gawad
Renal Histo-Pathology (II) - Normal Kidney Electron Microscopy - Dr. Gawad
 
Glomerular diseases
Glomerular diseasesGlomerular diseases
Glomerular diseases
 

Ähnlich wie Renal Pathology Lectures_Ppt Series (4 in 1)

Renal Physiology and Regulation of Water and Inorganic Ions
Renal Physiology and Regulation of Water and Inorganic IonsRenal Physiology and Regulation of Water and Inorganic Ions
Renal Physiology and Regulation of Water and Inorganic IonsImhotep Virtual Medical School
 
inflammatory diseases of the kidney& urinary bladder
 inflammatory diseases of the kidney& urinary bladder    inflammatory diseases of the kidney& urinary bladder
inflammatory diseases of the kidney& urinary bladder shams atrash
 
Acute renal failure 2018
Acute renal failure 2018Acute renal failure 2018
Acute renal failure 2018BMCStudents
 
Renal biopsy workshop1,2.ppt
Renal biopsy workshop1,2.pptRenal biopsy workshop1,2.ppt
Renal biopsy workshop1,2.pptIram110
 
A mathematical model of two phase, (One phase is Newtonian and other is non-N...
A mathematical model of two phase, (One phase is Newtonian and other is non-N...A mathematical model of two phase, (One phase is Newtonian and other is non-N...
A mathematical model of two phase, (One phase is Newtonian and other is non-N...iosrjce
 
Lecture 1-8th-nov-07
Lecture 1-8th-nov-07Lecture 1-8th-nov-07
Lecture 1-8th-nov-07Waleed Rafiq
 
Histopathological Correlation of Lymph Nodes Imprints
Histopathological Correlation of Lymph Nodes ImprintsHistopathological Correlation of Lymph Nodes Imprints
Histopathological Correlation of Lymph Nodes Imprintsiosrjce
 
SPLEEN AND RETICULOENDOTHELIAL SYSTEM
SPLEEN AND RETICULOENDOTHELIAL SYSTEMSPLEEN AND RETICULOENDOTHELIAL SYSTEM
SPLEEN AND RETICULOENDOTHELIAL SYSTEMDr Nilesh Kate
 
Renal pathology lecture 1 prof wadie elmadhoun
Renal pathology lecture 1 prof wadie elmadhounRenal pathology lecture 1 prof wadie elmadhoun
Renal pathology lecture 1 prof wadie elmadhounMohamed Wadie
 
Kidney embryology & its clinical implications
Kidney embryology & its clinical implicationsKidney embryology & its clinical implications
Kidney embryology & its clinical implicationsDr. Khaled Elzorkany
 
Histology of the Kidney part 1
Histology of the Kidney part 1Histology of the Kidney part 1
Histology of the Kidney part 1Maha Hammady
 
Glomerular disease 2018
Glomerular disease 2018 Glomerular disease 2018
Glomerular disease 2018 BMCStudents
 
Ivp in urinary tract diseases
Ivp in urinary tract diseasesIvp in urinary tract diseases
Ivp in urinary tract diseasesVrishit Saraswat
 
Week 4Cardiovascular DisordersVeins and arteries are vital eleme.docx
Week 4Cardiovascular DisordersVeins and arteries are vital eleme.docxWeek 4Cardiovascular DisordersVeins and arteries are vital eleme.docx
Week 4Cardiovascular DisordersVeins and arteries are vital eleme.docxco4spmeley
 

Ähnlich wie Renal Pathology Lectures_Ppt Series (4 in 1) (20)

Renal Physiology and Regulation of Water and Inorganic Ions
Renal Physiology and Regulation of Water and Inorganic IonsRenal Physiology and Regulation of Water and Inorganic Ions
Renal Physiology and Regulation of Water and Inorganic Ions
 
inflammatory diseases of the kidney& urinary bladder
 inflammatory diseases of the kidney& urinary bladder    inflammatory diseases of the kidney& urinary bladder
inflammatory diseases of the kidney& urinary bladder
 
Acute renal failure 2018
Acute renal failure 2018Acute renal failure 2018
Acute renal failure 2018
 
Renal biopsy workshop1,2.ppt
Renal biopsy workshop1,2.pptRenal biopsy workshop1,2.ppt
Renal biopsy workshop1,2.ppt
 
A mathematical model of two phase, (One phase is Newtonian and other is non-N...
A mathematical model of two phase, (One phase is Newtonian and other is non-N...A mathematical model of two phase, (One phase is Newtonian and other is non-N...
A mathematical model of two phase, (One phase is Newtonian and other is non-N...
 
Lecture 1-8th-nov-07
Lecture 1-8th-nov-07Lecture 1-8th-nov-07
Lecture 1-8th-nov-07
 
Histopathological Correlation of Lymph Nodes Imprints
Histopathological Correlation of Lymph Nodes ImprintsHistopathological Correlation of Lymph Nodes Imprints
Histopathological Correlation of Lymph Nodes Imprints
 
SPLEEN AND RETICULOENDOTHELIAL SYSTEM
SPLEEN AND RETICULOENDOTHELIAL SYSTEMSPLEEN AND RETICULOENDOTHELIAL SYSTEM
SPLEEN AND RETICULOENDOTHELIAL SYSTEM
 
23.surgical diseases of the liver
23.surgical diseases of the liver23.surgical diseases of the liver
23.surgical diseases of the liver
 
Renal pathology lecture 1 prof wadie elmadhoun
Renal pathology lecture 1 prof wadie elmadhounRenal pathology lecture 1 prof wadie elmadhoun
Renal pathology lecture 1 prof wadie elmadhoun
 
Kidney embryology & its clinical implications
Kidney embryology & its clinical implicationsKidney embryology & its clinical implications
Kidney embryology & its clinical implications
 
Histology of the Kidney part 1
Histology of the Kidney part 1Histology of the Kidney part 1
Histology of the Kidney part 1
 
Glomerular disease 2018
Glomerular disease 2018 Glomerular disease 2018
Glomerular disease 2018
 
Thesis book
Thesis bookThesis book
Thesis book
 
Liver seminar
Liver seminarLiver seminar
Liver seminar
 
Atn
AtnAtn
Atn
 
Renal failure
Renal failure Renal failure
Renal failure
 
Ivp in urinary tract diseases
Ivp in urinary tract diseasesIvp in urinary tract diseases
Ivp in urinary tract diseases
 
Week 4Cardiovascular DisordersVeins and arteries are vital eleme.docx
Week 4Cardiovascular DisordersVeins and arteries are vital eleme.docxWeek 4Cardiovascular DisordersVeins and arteries are vital eleme.docx
Week 4Cardiovascular DisordersVeins and arteries are vital eleme.docx
 
ARRS5.pptx
ARRS5.pptxARRS5.pptx
ARRS5.pptx
 

Mehr von Imhotep Virtual Medical School

Oncologic Pathology_A Case-based Organ Systems Review (USMLE Step 1)
Oncologic Pathology_A Case-based Organ Systems Review (USMLE Step 1)Oncologic Pathology_A Case-based Organ Systems Review (USMLE Step 1)
Oncologic Pathology_A Case-based Organ Systems Review (USMLE Step 1)Imhotep Virtual Medical School
 
Reproductive System Pathology_FM Breast and FM Reproductive Systems
Reproductive System Pathology_FM Breast and FM Reproductive SystemsReproductive System Pathology_FM Breast and FM Reproductive Systems
Reproductive System Pathology_FM Breast and FM Reproductive SystemsImhotep Virtual Medical School
 
Reproductive System Pathology_Male Reproductive Systems
Reproductive System Pathology_Male Reproductive SystemsReproductive System Pathology_Male Reproductive Systems
Reproductive System Pathology_Male Reproductive SystemsImhotep Virtual Medical School
 
Nervous System Pathology_A Case-based Learning Approach
Nervous System Pathology_A Case-based Learning ApproachNervous System Pathology_A Case-based Learning Approach
Nervous System Pathology_A Case-based Learning ApproachImhotep Virtual Medical School
 
CVS Function, Regulation of the Heart and Overview of Therapeutic Goals in CV...
CVS Function, Regulation of the Heart and Overview of Therapeutic Goals in CV...CVS Function, Regulation of the Heart and Overview of Therapeutic Goals in CV...
CVS Function, Regulation of the Heart and Overview of Therapeutic Goals in CV...Imhotep Virtual Medical School
 
Cardiovascular Pathology Case-based_Gross and Microscopic
Cardiovascular Pathology Case-based_Gross and MicroscopicCardiovascular Pathology Case-based_Gross and Microscopic
Cardiovascular Pathology Case-based_Gross and MicroscopicImhotep Virtual Medical School
 
Clinical Pharmacology for Medical Students_USMLE Step 1 & 2 Review
Clinical Pharmacology for Medical Students_USMLE Step 1 & 2 ReviewClinical Pharmacology for Medical Students_USMLE Step 1 & 2 Review
Clinical Pharmacology for Medical Students_USMLE Step 1 & 2 ReviewImhotep Virtual Medical School
 
Make the Dx_ A Case-based Intro to Select Cardiovascular and Respiratory Dise...
Make the Dx_ A Case-based Intro to Select Cardiovascular and Respiratory Dise...Make the Dx_ A Case-based Intro to Select Cardiovascular and Respiratory Dise...
Make the Dx_ A Case-based Intro to Select Cardiovascular and Respiratory Dise...Imhotep Virtual Medical School
 
Myocardial infarction_ Causes, Symptoms, Diagnosis, Treatment, and Pathology
Myocardial infarction_ Causes, Symptoms, Diagnosis, Treatment, and PathologyMyocardial infarction_ Causes, Symptoms, Diagnosis, Treatment, and Pathology
Myocardial infarction_ Causes, Symptoms, Diagnosis, Treatment, and PathologyImhotep Virtual Medical School
 
Autonomic Nervous System Physiology and Pharmacology_Overview| Review of ANS
Autonomic Nervous System Physiology and Pharmacology_Overview| Review of ANSAutonomic Nervous System Physiology and Pharmacology_Overview| Review of ANS
Autonomic Nervous System Physiology and Pharmacology_Overview| Review of ANSImhotep Virtual Medical School
 

Mehr von Imhotep Virtual Medical School (20)

Oncologic Pathology_A Case-based Organ Systems Review (USMLE Step 1)
Oncologic Pathology_A Case-based Organ Systems Review (USMLE Step 1)Oncologic Pathology_A Case-based Organ Systems Review (USMLE Step 1)
Oncologic Pathology_A Case-based Organ Systems Review (USMLE Step 1)
 
Pathology and Pathophysiology of Shock
Pathology and Pathophysiology of ShockPathology and Pathophysiology of Shock
Pathology and Pathophysiology of Shock
 
Drugs Used In Disorders of the Reproductive System
Drugs Used In Disorders of the Reproductive SystemDrugs Used In Disorders of the Reproductive System
Drugs Used In Disorders of the Reproductive System
 
Reproductive System Pathology_FM Breast and FM Reproductive Systems
Reproductive System Pathology_FM Breast and FM Reproductive SystemsReproductive System Pathology_FM Breast and FM Reproductive Systems
Reproductive System Pathology_FM Breast and FM Reproductive Systems
 
Reproductive System Pathology_Male Reproductive Systems
Reproductive System Pathology_Male Reproductive SystemsReproductive System Pathology_Male Reproductive Systems
Reproductive System Pathology_Male Reproductive Systems
 
Nervous System Pathology_A Case-based Learning Approach
Nervous System Pathology_A Case-based Learning ApproachNervous System Pathology_A Case-based Learning Approach
Nervous System Pathology_A Case-based Learning Approach
 
CVS Function, Regulation of the Heart and Overview of Therapeutic Goals in CV...
CVS Function, Regulation of the Heart and Overview of Therapeutic Goals in CV...CVS Function, Regulation of the Heart and Overview of Therapeutic Goals in CV...
CVS Function, Regulation of the Heart and Overview of Therapeutic Goals in CV...
 
Cardiovascular Pathology Case-based_Gross and Microscopic
Cardiovascular Pathology Case-based_Gross and MicroscopicCardiovascular Pathology Case-based_Gross and Microscopic
Cardiovascular Pathology Case-based_Gross and Microscopic
 
HIV / AIDS Pathology
HIV / AIDS PathologyHIV / AIDS Pathology
HIV / AIDS Pathology
 
Sepsis & Septic Shock
Sepsis & Septic ShockSepsis & Septic Shock
Sepsis & Septic Shock
 
Drugs Used in infectious Disease_Antibiotics
Drugs Used in infectious Disease_AntibioticsDrugs Used in infectious Disease_Antibiotics
Drugs Used in infectious Disease_Antibiotics
 
Hematopoietic and Lymphoid Systems Pathology
Hematopoietic and Lymphoid Systems  PathologyHematopoietic and Lymphoid Systems  Pathology
Hematopoietic and Lymphoid Systems Pathology
 
Drugs Used in Neoplastic Disorders
Drugs Used in Neoplastic DisordersDrugs Used in Neoplastic Disorders
Drugs Used in Neoplastic Disorders
 
Neoplasia & Oncologic Pathology
Neoplasia & Oncologic PathologyNeoplasia & Oncologic Pathology
Neoplasia & Oncologic Pathology
 
Clinical Pharmacology for Medical Students_USMLE Step 1 & 2 Review
Clinical Pharmacology for Medical Students_USMLE Step 1 & 2 ReviewClinical Pharmacology for Medical Students_USMLE Step 1 & 2 Review
Clinical Pharmacology for Medical Students_USMLE Step 1 & 2 Review
 
Make the Dx_ A Case-based Intro to Select Cardiovascular and Respiratory Dise...
Make the Dx_ A Case-based Intro to Select Cardiovascular and Respiratory Dise...Make the Dx_ A Case-based Intro to Select Cardiovascular and Respiratory Dise...
Make the Dx_ A Case-based Intro to Select Cardiovascular and Respiratory Dise...
 
Myocardial infarction_ Causes, Symptoms, Diagnosis, Treatment, and Pathology
Myocardial infarction_ Causes, Symptoms, Diagnosis, Treatment, and PathologyMyocardial infarction_ Causes, Symptoms, Diagnosis, Treatment, and Pathology
Myocardial infarction_ Causes, Symptoms, Diagnosis, Treatment, and Pathology
 
Basic CXR Interpretation_Diagnostic Radiographs
Basic CXR Interpretation_Diagnostic RadiographsBasic CXR Interpretation_Diagnostic Radiographs
Basic CXR Interpretation_Diagnostic Radiographs
 
Electrocardiogram (ECG) Interpretation_Module 1 of 2
Electrocardiogram (ECG) Interpretation_Module 1 of 2Electrocardiogram (ECG) Interpretation_Module 1 of 2
Electrocardiogram (ECG) Interpretation_Module 1 of 2
 
Autonomic Nervous System Physiology and Pharmacology_Overview| Review of ANS
Autonomic Nervous System Physiology and Pharmacology_Overview| Review of ANSAutonomic Nervous System Physiology and Pharmacology_Overview| Review of ANS
Autonomic Nervous System Physiology and Pharmacology_Overview| Review of ANS
 

Kürzlich hochgeladen

Nutritional Needs Presentation - HLTH 104
Nutritional Needs Presentation - HLTH 104Nutritional Needs Presentation - HLTH 104
Nutritional Needs Presentation - HLTH 104misteraugie
 
Mixin Classes in Odoo 17 How to Extend Models Using Mixin Classes
Mixin Classes in Odoo 17  How to Extend Models Using Mixin ClassesMixin Classes in Odoo 17  How to Extend Models Using Mixin Classes
Mixin Classes in Odoo 17 How to Extend Models Using Mixin ClassesCeline George
 
APM Welcome, APM North West Network Conference, Synergies Across Sectors
APM Welcome, APM North West Network Conference, Synergies Across SectorsAPM Welcome, APM North West Network Conference, Synergies Across Sectors
APM Welcome, APM North West Network Conference, Synergies Across SectorsAssociation for Project Management
 
PROCESS RECORDING FORMAT.docx
PROCESS      RECORDING        FORMAT.docxPROCESS      RECORDING        FORMAT.docx
PROCESS RECORDING FORMAT.docxPoojaSen20
 
Application orientated numerical on hev.ppt
Application orientated numerical on hev.pptApplication orientated numerical on hev.ppt
Application orientated numerical on hev.pptRamjanShidvankar
 
Sports & Fitness Value Added Course FY..
Sports & Fitness Value Added Course FY..Sports & Fitness Value Added Course FY..
Sports & Fitness Value Added Course FY..Disha Kariya
 
Russian Escort Service in Delhi 11k Hotel Foreigner Russian Call Girls in Delhi
Russian Escort Service in Delhi 11k Hotel Foreigner Russian Call Girls in DelhiRussian Escort Service in Delhi 11k Hotel Foreigner Russian Call Girls in Delhi
Russian Escort Service in Delhi 11k Hotel Foreigner Russian Call Girls in Delhikauryashika82
 
ICT Role in 21st Century Education & its Challenges.pptx
ICT Role in 21st Century Education & its Challenges.pptxICT Role in 21st Century Education & its Challenges.pptx
ICT Role in 21st Century Education & its Challenges.pptxAreebaZafar22
 
fourth grading exam for kindergarten in writing
fourth grading exam for kindergarten in writingfourth grading exam for kindergarten in writing
fourth grading exam for kindergarten in writingTeacherCyreneCayanan
 
Key note speaker Neum_Admir Softic_ENG.pdf
Key note speaker Neum_Admir Softic_ENG.pdfKey note speaker Neum_Admir Softic_ENG.pdf
Key note speaker Neum_Admir Softic_ENG.pdfAdmir Softic
 
1029 - Danh muc Sach Giao Khoa 10 . pdf
1029 -  Danh muc Sach Giao Khoa 10 . pdf1029 -  Danh muc Sach Giao Khoa 10 . pdf
1029 - Danh muc Sach Giao Khoa 10 . pdfQucHHunhnh
 
Unit-V; Pricing (Pharma Marketing Management).pptx
Unit-V; Pricing (Pharma Marketing Management).pptxUnit-V; Pricing (Pharma Marketing Management).pptx
Unit-V; Pricing (Pharma Marketing Management).pptxVishalSingh1417
 
Seal of Good Local Governance (SGLG) 2024Final.pptx
Seal of Good Local Governance (SGLG) 2024Final.pptxSeal of Good Local Governance (SGLG) 2024Final.pptx
Seal of Good Local Governance (SGLG) 2024Final.pptxnegromaestrong
 
Unit-IV; Professional Sales Representative (PSR).pptx
Unit-IV; Professional Sales Representative (PSR).pptxUnit-IV; Professional Sales Representative (PSR).pptx
Unit-IV; Professional Sales Representative (PSR).pptxVishalSingh1417
 
SOCIAL AND HISTORICAL CONTEXT - LFTVD.pptx
SOCIAL AND HISTORICAL CONTEXT - LFTVD.pptxSOCIAL AND HISTORICAL CONTEXT - LFTVD.pptx
SOCIAL AND HISTORICAL CONTEXT - LFTVD.pptxiammrhaywood
 
1029-Danh muc Sach Giao Khoa khoi 6.pdf
1029-Danh muc Sach Giao Khoa khoi  6.pdf1029-Danh muc Sach Giao Khoa khoi  6.pdf
1029-Danh muc Sach Giao Khoa khoi 6.pdfQucHHunhnh
 
An Overview of Mutual Funds Bcom Project.pdf
An Overview of Mutual Funds Bcom Project.pdfAn Overview of Mutual Funds Bcom Project.pdf
An Overview of Mutual Funds Bcom Project.pdfSanaAli374401
 
Grant Readiness 101 TechSoup and Remy Consulting
Grant Readiness 101 TechSoup and Remy ConsultingGrant Readiness 101 TechSoup and Remy Consulting
Grant Readiness 101 TechSoup and Remy ConsultingTechSoup
 

Kürzlich hochgeladen (20)

Nutritional Needs Presentation - HLTH 104
Nutritional Needs Presentation - HLTH 104Nutritional Needs Presentation - HLTH 104
Nutritional Needs Presentation - HLTH 104
 
Mixin Classes in Odoo 17 How to Extend Models Using Mixin Classes
Mixin Classes in Odoo 17  How to Extend Models Using Mixin ClassesMixin Classes in Odoo 17  How to Extend Models Using Mixin Classes
Mixin Classes in Odoo 17 How to Extend Models Using Mixin Classes
 
APM Welcome, APM North West Network Conference, Synergies Across Sectors
APM Welcome, APM North West Network Conference, Synergies Across SectorsAPM Welcome, APM North West Network Conference, Synergies Across Sectors
APM Welcome, APM North West Network Conference, Synergies Across Sectors
 
INDIA QUIZ 2024 RLAC DELHI UNIVERSITY.pptx
INDIA QUIZ 2024 RLAC DELHI UNIVERSITY.pptxINDIA QUIZ 2024 RLAC DELHI UNIVERSITY.pptx
INDIA QUIZ 2024 RLAC DELHI UNIVERSITY.pptx
 
PROCESS RECORDING FORMAT.docx
PROCESS      RECORDING        FORMAT.docxPROCESS      RECORDING        FORMAT.docx
PROCESS RECORDING FORMAT.docx
 
Application orientated numerical on hev.ppt
Application orientated numerical on hev.pptApplication orientated numerical on hev.ppt
Application orientated numerical on hev.ppt
 
Sports & Fitness Value Added Course FY..
Sports & Fitness Value Added Course FY..Sports & Fitness Value Added Course FY..
Sports & Fitness Value Added Course FY..
 
Russian Escort Service in Delhi 11k Hotel Foreigner Russian Call Girls in Delhi
Russian Escort Service in Delhi 11k Hotel Foreigner Russian Call Girls in DelhiRussian Escort Service in Delhi 11k Hotel Foreigner Russian Call Girls in Delhi
Russian Escort Service in Delhi 11k Hotel Foreigner Russian Call Girls in Delhi
 
ICT Role in 21st Century Education & its Challenges.pptx
ICT Role in 21st Century Education & its Challenges.pptxICT Role in 21st Century Education & its Challenges.pptx
ICT Role in 21st Century Education & its Challenges.pptx
 
fourth grading exam for kindergarten in writing
fourth grading exam for kindergarten in writingfourth grading exam for kindergarten in writing
fourth grading exam for kindergarten in writing
 
Key note speaker Neum_Admir Softic_ENG.pdf
Key note speaker Neum_Admir Softic_ENG.pdfKey note speaker Neum_Admir Softic_ENG.pdf
Key note speaker Neum_Admir Softic_ENG.pdf
 
1029 - Danh muc Sach Giao Khoa 10 . pdf
1029 -  Danh muc Sach Giao Khoa 10 . pdf1029 -  Danh muc Sach Giao Khoa 10 . pdf
1029 - Danh muc Sach Giao Khoa 10 . pdf
 
Unit-V; Pricing (Pharma Marketing Management).pptx
Unit-V; Pricing (Pharma Marketing Management).pptxUnit-V; Pricing (Pharma Marketing Management).pptx
Unit-V; Pricing (Pharma Marketing Management).pptx
 
Seal of Good Local Governance (SGLG) 2024Final.pptx
Seal of Good Local Governance (SGLG) 2024Final.pptxSeal of Good Local Governance (SGLG) 2024Final.pptx
Seal of Good Local Governance (SGLG) 2024Final.pptx
 
Unit-IV; Professional Sales Representative (PSR).pptx
Unit-IV; Professional Sales Representative (PSR).pptxUnit-IV; Professional Sales Representative (PSR).pptx
Unit-IV; Professional Sales Representative (PSR).pptx
 
SOCIAL AND HISTORICAL CONTEXT - LFTVD.pptx
SOCIAL AND HISTORICAL CONTEXT - LFTVD.pptxSOCIAL AND HISTORICAL CONTEXT - LFTVD.pptx
SOCIAL AND HISTORICAL CONTEXT - LFTVD.pptx
 
1029-Danh muc Sach Giao Khoa khoi 6.pdf
1029-Danh muc Sach Giao Khoa khoi  6.pdf1029-Danh muc Sach Giao Khoa khoi  6.pdf
1029-Danh muc Sach Giao Khoa khoi 6.pdf
 
An Overview of Mutual Funds Bcom Project.pdf
An Overview of Mutual Funds Bcom Project.pdfAn Overview of Mutual Funds Bcom Project.pdf
An Overview of Mutual Funds Bcom Project.pdf
 
Grant Readiness 101 TechSoup and Remy Consulting
Grant Readiness 101 TechSoup and Remy ConsultingGrant Readiness 101 TechSoup and Remy Consulting
Grant Readiness 101 TechSoup and Remy Consulting
 
Mattingly "AI & Prompt Design: The Basics of Prompt Design"
Mattingly "AI & Prompt Design: The Basics of Prompt Design"Mattingly "AI & Prompt Design: The Basics of Prompt Design"
Mattingly "AI & Prompt Design: The Basics of Prompt Design"
 

Renal Pathology Lectures_Ppt Series (4 in 1)

  • 1. Glomeruli and associated blood vessels in the kidney (colorized scanning electron micrograph). From Widmaier EP, Raff H , Strang KT. Vander’s Human Physiology : The Mechanisms of Body Function, 14th ed. New York, NY: McGraw-Hill, 2016. Pathology of Renal System Marc Imhotep Cray, M.D.
  • 2. Marc Imhotep Cray, MD Overall Goal 2 The overall goal of this lecture series is to provide the learner with a basic scientific understanding of the patterns, causes, mechanisms and effects of the most clinically common renal disorders; including:  Acute renal failure (AKI) and chronic renal failure (CKD)  Glomerular diseases  Urinary Tract Infections  Tubulointerstitial diseases  Obstructive uropathy, hydronephrosis & urolithiasis  Vascular diseases  Renal neoplasms  Congenital and cystic diseases of kidney  Volume regulation & electrolyte disorders  Acid-base disorders Covered elsewhere within the sequence.
  • 3. Marc Imhotep Cray, MD Learning objectives (abridged*) 3 1.Describe the causes, mechanisms and effects of acute kidney injury. 2.Describe the causes, mechanisms and effects of chronic kidney disease. 3. Classify glomerular diseases based on acute vs chronic, histopathologic features and clinical presentation. 4. Explain the pathogenesis (ie, immune-mediated Injury) and pathophysiology of the most common glomerulonephritidies. 5. Describe the clinicopathologic features of the nephrotic vs nephritic syndrome *See Renal Pathology Learning Objectives, unabridged version.pdf
  • 4. Marc Imhotep Cray, MD Learning objectives cont’d. 4 6. Describe the characteristic light microscopy (LM), immunofluorescence (IF), and electron microscopy (EM) findings glomerular diseases. 7. Describe the renal features of systemic diseases (eg., HTN, DM, SLE, amyloidosis, vasculitides) that involve the kidney. 8. Describe the major causes and pathologic mechanisms of tubulointerstitial diseases. 9. Explain in the classification, etiology and pathogenesis of urinary tract infections.
  • 5. Marc Imhotep Cray, MD Learning objectives cont’d. 5 10. Describe the gross and microscopic changes in acute pyelonephritis (PN) and chronic PN. 11. List the causes of renal papillary necrosis. Explain the causes and complications of obstructive uropathy, and hydronephrosis. 12. Describe the composition of the most common forms, favored sites for formation and complications of urolithiasis. 13. Describe the gross and microscopic features and typical pathologic and clinical findings of commonest benign and malignant tumors of the kidney. 14. List the congenital and cystic anomalies of the kidney.
  • 6. Marc Imhotep Cray, MD Topics Discussion Outline 6  Scope of Problem  Anatomy, Histology and Cell Biology  Functions of kidney and lower urinary tract  Overview of Renal Pathology  Clinical Manifestations of Renal Diseases  Glomerular Diseases  Tubulointerstitial Diseases  Vascular Diseases  Obstructive uropathy, hydronephrosis & urolithiasis  Renal Neoplasia  Congenital and Cystic Diseases of Kidney
  • 7. Marc Imhotep Cray, MD Scope of Problem 7 Centers for Disease Control and Prevention estimates in U.S. more than 10% of people 20 years and older (or more than 20 million individuals) have chronic kidney disease  many more suffer from acute kidney injury annually thus, Clinicians of all specialties will encounter patients w renal disorders behooves all to be aware of various risk factors and causes of kidney disease (eg. hypertension and DM)  Important b/c w early detection and appropriate management may be able to prevent or at least slow rate of progression to kidney failure or other complications NB: Diagnosing kidney disease is particularly challenging, as patients are typically asymptomatic until relatively advanced kidney failure is present.
  • 8. Marc Imhotep Cray, MD Lecture 1: Scope of Problem Anatomy, Histology and Cell Biology Functions of kidney and lower urinary tract Overview of Renal Pathology Clinical Manifestations of Renal Diseases 8
  • 9. Marc Imhotep Cray, MD Anatomy, Histology and Cell Biology 9 Basic structural and functional unit of kidney function is nephron  A structure consisting of a tuft of capillaries termed glomerulus and a tubule Each human kidney has approx. 1 million nephrons  Each nephron is composed of an initial filtering component (renal corpuscle [glomerulus& Bowman's capsule]) and a tubule specialized for reabsorption and secretion (renal tubule) NB: Understanding the complex functional organization of glomerulus is crucial for understanding both normal renal function and characteristics of different glomerular diseases.
  • 10. Marc Imhotep Cray, MD Schematic of Gross Anatomy 10 Mohan H. Textbook of Pathology, 7th Ed. New Delhi: Jaypee Brothers Medical Publishers, 2015. Cross-section of kidney showing gross structures. Cross-section of kidney showing arterial blood supply Renal blood flow: renal artery  segmental artery interlobar artery  arcuate artery interlobular artery  afferent arteriole  glomerulus  efferent arteriole  vasa recta/ peritubular capillaries  venous outflow
  • 11. Marc Imhotep Cray, MD Kidney and lower urinary tract 11 Kidney (upper urinary tract) & lower urinary tract are functionally connected abnormalities in one component can affect function of other components of system Bladder, urethra, ureters, & renal pelvis (lower urinary tract) are lined by transitional epithelium (urothelium) subject to similar types of disease processes Widmaier EP, Raff H , Strang KT. Vander’s Human Physiology : The Mechanisms of Body Function, 14th ed. New York, NY: McGraw-Hill, 2016.
  • 12. 12 Normal Kidney, Gross Klatt EC. Robbins and Cotran Atlas of Pathology, 3rd Ed. Philadelphia: Saunders, 2015.
  • 13. 13 Normal Kidney, Gross cross section Klatt EC. Robbins and Cotran Atlas of Pathology, 3rd Ed. Philadelphia: Saunders, 2015.
  • 14. 14 Normal Kidney, CT image Klatt EC. Robbins and Cotran Atlas of Pathology, 3rd Ed. Philadelphia: Saunders, 2015.
  • 15. 15 Normal Kidney, microscopic Klatt EC. Robbins and Cotran Atlas of Pathology, 3rd Ed. Philadelphia: Saunders, 2015.
  • 16. 16 Normal Kidney, microscopic Klatt EC. Robbins and Cotran Atlas of Pathology, 3rd Ed. Philadelphia: Saunders, 2015.
  • 17. 17 Normal Kidney, microscopic HP Klatt EC. Robbins and Cotran Atlas of Pathology, 3rd Ed. Philadelphia: Saunders, 2015.
  • 18. Marc Imhotep Cray, MD Structure & histology of renal corpuscle 18 Kelly CR, Landman J (Eds.). The Netter Collection of Medical Illustrations, 2Ed. Vol 5- Urinary System. Philadelphia: Saunders- Elsevier, 2012;21. Four layers make up filtering apparatus of glomerulus each of these can be affected in renal disease with effects on filtration: • basement membrane • blood vessel lining (endothelial cells) • podocytes (epithelial cells) • mesangium (smooth muscle-like cells with phagocytes) See: Histology of the glomerular filtration barrier.pdf
  • 19. Marc Imhotep Cray, MD Glomerulus and glomerular capillary 19 Hammer GD & McPhee SJ (Eds.). Pathophysiology of Disease: An Introduction to Clinical Medicine, 7th Ed. New York, NY: McGraw-Hill Education, 2014; 457. A glomerulus consists of an afferent and an efferent arteriole and an intervening tuft of capillaries lined by endothelial cells and covered by epithelial cells that form a continuous layer with those of Bowman capsule and renal tubule. Space between capillaries in glomerulus is called mesangium. Material comprising a basement membrane is located between capillary endothelial cells and epithelial cells.
  • 20. Marc Imhotep Cray, MD Detailed structure of glomerulus and glomerular filtration membrane composed of endothelial cell, basement membrane, and podocyte. 20 Hammer GD & McPhee SJ (Eds.). Pathophysiology of Disease: An Introduction to Clinical Medicine, 7th Ed. New York, NY: McGraw-Hill Education, 2014; 457. Glomerular capillaries have unique features not found in most peripheral capillaries. First, glomerular capillary endothelium is fenestrated. However, because endothelial cells have a coat of negatively charged glycoproteins and glycosaminoglycans, they normally exclude plasma proteins such as albumin. On other side of glomerular basement membrane are epithelial cells. Termed “podocytes” because of their numerous extensions or foot processes, these cells are connected to one another by modified desmosomes. Mesangium is an extension of GBM.
  • 21. 21 (Shown on left) Anatomy of a normal glomerular capillary. Note fenestrated endothelium (EN), glomerular basement membrane (GBM), and epithelium with its foot processes (EP). Mesangium is composed of mesangial cells (MC) surrounded by extracellular matrix (MM) in direct contact with endothelium. Ultrafiltration occurs across glomerular wall and through channels in mesangial matrix into urinary space (US). (Shown on right) Typical localization of immune deposits and other pathologic changes. (1) Uniform subepithelial deposits as in membranous nephropathy. (2) Large, irregular subepithelial deposits or “humps” seen in acute postinfectious glomerulonephritis. (3) Subendothelial deposits as in diffuse proliferative lupus glomerulonephritis. (4) Mesangial deposits characteristic of immunoglobulin A nephropathy. (5) Antibody binding to glomerular basement membrane (as in Goodpasture syndrome) does not produce visible deposits, but a smooth linear pattern is seen on immunofluorescence. (6) Effacement of epithelial foot processes is common in all forms of glomerular injury with proteinuria. Hammer GD & McPhee SJ (Eds.). Pathophysiology of Disease: An Introduction to Clinical Medicine, 7th Ed. New York, NY: McGraw-Hill Education, 2014; 462.
  • 22. 22 Normal glomerular capillary electron micrograph Rubin R, Strayer D (eds.) Rubin’s Pathology. Clinicopathologic Foundations of Medicine, 6th ed. Baltimore: Wolters Kluwer Health, 2012.  This EM depicts a single capillary loop and adjacent mesangium. Capillary wall portion of lumen (L) is lined by a thin layer of fenestrated endothelial cytoplasm (high magnification next slide) that extends out from endothelial cell body (E)  Endothelial cell body is in direct contact with mesangium, which includes the mesangial cell (M) and adjacent matrix  Outer aspect of basement membrane (B) is covered by foot processes (F) from podocyte (P) that line urinary space (U)
  • 23. 23 Electron micrograph of glomerular filter Molecules that pass from capillary lumen (CL) to urinary space (US) traverse fenestrations (F) of endothelial cell (E) trilaminar basement membrane (BM) (lamina rara interna [LRI], lamina densa [LD] and lamina rara externa [LRE]) and slit pore diaphragm (D) that connects podocyte foot processes (FP) Rubin R, Strayer D (eds.) Rubin’s Pathology. Clinicopathologic Foundations of Medicine, 6th ed. Baltimore: Wolters Kluwer Health, 2012. REMEMBER: Endothelial cells have a coat of negatively charged glycoproteins and glycosaminoglycans, they normally exclude plasma proteins such as albumin.
  • 24. 24 Photo: Glomerulus in a human kidney scanning electron micrograph. From: Widmaier EP, Raff H & Strang KT. Vander’s Human Physiology: The Mechanisms Of Body Function, 13th Ed. New York, NY: McGraw-Hill Companies, 2014; 490. [Labeling mine] Foot processes Cell body of podocyte Filtration slits Bowman’s capsule podocytes with foot processes and filtration slits. Note the filtration slits between adjacent foot processes. The podocytes and their foot processes encircle the glomerular capillaries.
  • 25. Marc Imhotep Cray, MD Functions of kidney and urinary tract 25 1. Excretion:  of waste products and drugs this involves selective reabsorption and excretion of substances as they pass through nephron 2. Regulation:  of body fluid volume and ionic composition kidneys have a major role in homeostasis and  are also involved in maintaining acid–base balance  Kidney participates in whole-body homeostasis via 4 main functions:
  • 26. Marc Imhotep Cray, MD Functions of kidney cont’d. 26 3. Endocrine:  kidneys are involved in synthesis of renin (which generates angiotensin I from angiotensinogen) thus has a role in blood pressure and sodium balance)  Erythropoietin  controls erythrocyte production; and  Prostaglandins  involved in regulation of renal function  ADH retain water in body and constrict blood vessels 4. Metabolism:  Vitamin D is metabolized to its active form  kidney is a major site for the catabolism of low-molecular- weight proteins including several hormones such as insulin, PTH and calcitonin  has ability to produce glucose through gluconeogenesis
  • 27. Marc Imhotep Cray, MD Overview of Renal Pathology 27  Kidney consists of glomerular, vascular, tubular & interstitial anatomic compartments  Renal diseases are best understood in relation to compartments affected & assoc. functional impairment  Pathology of kidney can be organized into four anatomic categories= diseases of 1. glomeruli 2. tubules and 3. interstitium 4. vessels thus,
  • 28. Marc Imhotep Cray, MD Overview (2) 28  Diseases that affect glomeruli most often have an immunologic etiology  Glomerular function disrupted by diseases that alter glomerular structural arrangements seen w structural damage to basement membrane, endothelium, epithelium or mesangium whereas,  Diseases that affect tubules and interstitium usually have an infectious or toxic (drugs) etiology  Tubular function disrupted by metabolic insult to tubular cells  hypoxia or toxins
  • 29. Marc Imhotep Cray, MD Overview (3) 29  Early in disease process, predominantly affect one of four anatomic structures over time, however, entire kidney becomes diseased b/c of close structural & functional relationships in nephron  tubules & interstitium usually affected together  As both glomerular and tubular functions are highly dependent on adequate perfusion by blood if bld perfusion disrupted, both functions are impaired
  • 30. Marc Imhotep Cray, MD Overview (4) 30  b/c of large physiologic reserve of kidneys, many diseases do not become clinically apparent until majority of organ is affected subtle abnormalities in laboratory findings* are only early indication of renal disease  Recognition of patterns of abnormalities, pathologic findings, and clinical presentation are arguably more important to renal pathology than in any other organ system N.B. *Chronic kidney disease can be detected at an early stage by measuring GFR (nml 90–140 mL/ min men, and 80–125 mL/ min for women).
  • 31. Marc Imhotep Cray, MD Overview(5)Types of renal failure syndromes 31 Partial renal failure syndromes affect only some elements of renal function  There are four main types of partial renal failure: 1. Asymptomatic hematuria 2. Persistent proteinuria 3. Nephrotic syndrome 4. Nephritic syndrome Total renal failure syndromes all functions of nephron are impaired  There are two types of total renal failure syndromes: 1. Acute renal failure (=AKI) sudden cessation of nephron function, affecting all nephrons at same time 2. Chronic renal failure (=CKD) nephrons are destroyed one by one over a long period
  • 32. Marc Imhotep Cray, MD QUESTION: How does someone with a partial renal failure syndrome, such as a nephrotic syndrome, eventually develops total renal failure syndrome(CKD)? Steps in pathological progression follow… 32 In most glomerular diseases damage causing nephritic or nephrotic syndrome also eventually causes glomeruli to become completely scarred (hyalinized) leading to loss of individual nephrons ↓ Glomerular hyalinization is  result of excessive production of mesangial matrix by mesangial cells over a long period of time ↓ Expanding mesangial matrix mass slowly but progressively crushes glomerular architecture out of existence until no blood flows through glomerular capillaries and no oxygenated blood passes into efferent arterioles and peritubular capillary systems ↓ Tubules are deprived of oxygenated blood, and tubular epithelial cells die irrevocably and become atrophic…cont’d. on next slide
  • 33. Marc Imhotep Cray, MD Partial renal failure to complete renal failure explained cont’d. 33 Thus destruction of glomerulus  leads to destruction of entire nephron unit ↓ As more and more nephrons are destroyed partial renal failure syndrome (nephritic or nephrotic)  develops into total renal failure syndrome of chronic renal failure (CKD) ↓ This is associated with progressive shrinkage of kidney to form a small, scarred organ termed end-stage kidney ↓
  • 34. 34Kibble J , Cannarozzi ML. Pathophysiology Flash Cards. New York: McGraw-Hill, 2013.
  • 35. Marc Imhotep Cray, MD Overview (5) Renal Function Tests (RFT) 35  To assess renal function, tests are available to give information regarding following parameters:  Renal blood flow  Glomerular filtration (next slide)  Renal tubular function  Urinary outflow unhindered by any obstruction NB: Urinalysis and measurement of serum creatinine are initial steps in evaluation of renal disorders.  RFT divided into 4 groups: 1. Urinalysis 2. Concentration & dilution tests 3. Blood chemistry 4. Renal clearance tests
  • 36. Marc Imhotep Cray, MD Overview (6) Glomerular Filtration 36 Filtration Barrier  Composed of 1) fenestrated capillary endothelium, 2) fused glomerular basement membrane, and 3) podocyte foot processes epithelial layer o capillary endothelium serves as a size barrier, while basement membrane contains heparan sulfate leads to a negative charge barrier preventing protein (albumin) filtration (=primary barrier) Glomerular Filtration Rate and Filtration Fraction  GFR can be estimated by clearance of creatinine (CCr)  Effective renal plasma flow (RPF) can be estimated by clearance of para- aminohippuric acid (CPAH)  Filtration fraction(FF) = fraction of RPF filtered across glomerular capillaries  GFR/RPF=FF
  • 37. Marc Imhotep Cray, MD GFR estimated by CCr 37 True picture of status of renal function can be obtained by measuring GFR achieved clinically by measuring CCr, using a calculation based on Cr content of a 24-hour collection of urine and a single bld Cr estimation  GFR nml range: 90–140 mL/ min men & 80–125 mL/ min women Blood levels of urea and creatinine do not rise above nml until GFR (CCr) has fallen below 50% nml  thus , for many years of a chronic progressive kidney disease pt. may be asymptomatic or show only minor Sx or Sn o important to detect these early stages and to investigate cause and monitor rate of progression
  • 38. Marc Imhotep Cray, MD Overview (7) Urinalysis & Urine Microscopy 38 …Urine microscopy:  Presence of casts indicates that hematuria/pyuria is of glomerular or renal tubular origin  Bladder cancer and kidney stones→hematuria, no casts  Acute cystitis → pyuria, no casts  RBC casts → glomerulonephritis, malignant hypertension  Understanding basic urinalysis (UA) and urine microscopy helps interpret pathology of kidney, for instance… (See Urinalysis & Urine Microscopy. Pdf notes)
  • 39. Marc Imhotep Cray, MD Overview (9) Renal Biopsy 39  In addition to RFT, renal biopsy is performed on selected pts w kidney disease to confirm Dx Indications include: hematuria, proteinuria, renal failure, transplant kidney surveillance…more  Renal biopsy tissue sample is fixed in alcoholic solution and examined morphologically supported by special stains… 1. Light microscopy (LM) o Periodic acid-Schiff (PAS) stain for highlighting GBM o Silver impregnation to outline glomerular & tubular BM 2. Immunofluorescence (IF) to localize antigens, complements and immunoglobulins type of immunologic injury is assessed 3. Electron microscopy (EM) to see ultrastructure of glomerular changes exact site of immune-complex deposition is visualized
  • 40. Marc Imhotep Cray, MD Overview (10) Definitions of key terms 40 Nephrotic syndrome: A kidney disorder affecting glomerulus, leading to proteinuria >3.5 g/day, hypoalbuminemia, generalized edema, and hyperlipidemia  Nephrosis: nephropathy without inflammation or neoplasia Nephritic syndrome: A kidney disorder of oliguria, hematuria, edema, and hypertension, resulting from glomerulonephritis (also sub-nephrotic proteinuria)  Nephritis: nephropathy with inflammation
  • 41. Marc Imhotep Cray, MD Definitions of key terms cont’d. 41 Primary glomerular disease: A condition of kidney that leads to  nephrosis, such as membranous glomerulopathy, minimal change disease, and focal segmental glomerulosclerosis or  nephritis, such as postinfectious glomerulonephritis, rapidly progressive glomerulonephritis and IgA nephropathy (Berger disease) Secondary glomerular disease: Conditions that are systemic, infectious, and (or) toxic & affect kidney, leading to nephrosis or nephritis  Some of causes are DM, SLE, amyloidosis, penicillamine, chronic hepatitis B infection, HIV, SBE…
  • 42. Marc Imhotep Cray, MD Definitions of key terms cont’d. 42  Azotemia is a biochemical abnormality that refers to an elevation of blood urea nitrogen (BUN) and creatinine (Cr) levels, and is related to a ↓ glomerular filtration rate (GFR)  Uremia is when azotemia becomes assoc. w a constellation of clinical signs (Sn) and symptoms (Sx) and biochemical abnormalities
  • 43. Marc Imhotep Cray, MD Definitions of key terms cont’d. 43 Glomerulosclerosis versus glomerulonephritis:  In glomerulosclerosis (sclerosis means hardening) there are sclerotic, scarred areas that lose ability to filter, secondary to capillary collapse  In glomerulonephritis (-itis means inflammation), there is ongoing glomerular inflammation (=hypercellularity and leukocytosis) Crescentic: Used to describe appearance when inflammatory cells fill Bowman’s space leading to a crescent appearance  NB: always indicative of a rapidly progressive glomerulonephritis (RPGN)=bad disease
  • 44. Marc Imhotep Cray, MD Definitions of key terms cont’d. 44 Glomerulopathy: can include processes that are inflammatory or noninflammatory, however  b/c term glomerulitis exists for inflammatory conditions, glomerulopathy sometimes carries a noninflammatory implication  Regarding location of glomerular lesions:  Subepithelial: between podocyte and GBM (on epithelial side of GBM)  Subendothelial: between endothelium and GBM (on endothelial side of GBM)
  • 45. Marc Imhotep Cray, MD Clinical Manifestations of Renal Diseases 45  Following is a discussion of clinical manifestations and syndromes of renal diseases with their defining features two most common syndromes assoc. w glomerular diseases, nephrotic and nephritic, are given most attention  Azotemia and uremia (defined above)  Asymptomatic hematuria or proteinuria  Acute kidney injury  Chronic kidney disease  End-stage renal disease (ESRD)  Renal tubular defects  Urinary tract obstruction and renal tumors  Nephrolithiasis (renal stones)  Nephrotic syndrome  Nephritic syndrome
  • 46. Marc Imhotep Cray, MD Clinical Manifestations (2) 46  Azotemia =↑of BUN & Cr levels  reflects a ↓ (GFR)  GFR may be ↓ as a consequence of intrinsic renal disease or extrarenal causes  Azotemia has three classifications, depending on causative origin o Prerenal azotemia is encountered when there is hypoperfusion of kidneys  usually due to reduced extracellular fluid volume  usually reversible if hypoperfusion is corrected in time o Primary renal azotemia (ARF [AKI]) typically leading to uremia intrinsic disease of kidney resultant of renal parenchymal damage • Causes include renal failure, glomerulonephritis, acute tubular necrosis, or any other kind of renal disease o Postrenal azotemia results when urine outflow is obstructed relief of obstruction is followed by correction of azotemia
  • 47. Marc Imhotep Cray, MD Clinical Manifestations (3) 47  When azotemia gives rise to clinical manifestations and systemic biochemical abnormalities it becomes uremia=  failure of renal excretory function + a host of metabolic & endocrine alterations  In addition, uremia is characterized by secondary GI (e.g., uremic gastroenteritis), neuromuscular (e.g., peripheral neuropathy), and cardiovascular (e.g., uremic pericarditis) involvement
  • 48. 48 Renal Failure and Uremia Manifestations Buja LM, Krueger GR. Netter’s Illustrated Human Pathology 2nd Ed. Philadelphia: Saunders-Elsevier, 2014. Inability to make urine & excrete nitrogenous wastes: Consequences (MAD HUNGER): Metabolic Acidosis Dyslipidemia (especially ↑triglycerides) Hyperkalemia Uremia—clinical syndrome marked by ↑ BUN:  Nausea and anorexia  Pericarditis  Asterixis  Encephalopathy  Platelet dysfunction Na+/H2O retention (HF, pulmonary edema, hypertension) Growth retardation and developmental delay Erythropoietin failure (anemia) Renal osteodystrophy
  • 49. Marc Imhotep Cray, MD Clinical Manifestations (5) 49  Asymptomatic hematuria or proteinuria, or a combination of these two represents a manifestation of subtle or mild glomerular abnormalities  Acute kidney injury is characterized by rapid decline in GFR (within hours to days), w concurrent dysregulation of fluid & electrolyte balance, and retention of metabolic waste products normally excreted by kidney including urea and creatinine  In its most severe forms it is manifested by oliguria or anuria (reduced or no urine flow)  AKI can result from glomerular, interstitial, vascular or acute tubular injury
  • 50. Marc Imhotep Cray, MD Clinical Manifestations (6) 50  Chronic kidney disease (chronic renal failure) is defined as: presence of a diminished GFR that is persistently <60 mL /minute for at least 3 months, from any cause, and/or persistent albuminuria  CKD may present w clinically silent decline in renal excretory function in milder forms, and  in more severe cases, by prolonged Sx and Sn of uremia  It is end result of all chronic renal parenchymal diseases
  • 51. Etiologies of CKD Buja LM, Krueger GR. Netter’s Illustrated Human Pathology 2nd Ed. Philadelphia: Saunders-Elsevier, 2014. Main groups of causes are:  Chronic vascular disease (e.g. long-standing hypertension  Disease of glomeruli, e.g. glomerulonephritis and diabetic glomerular disease  Disease of tubules and interstitium infective, toxic and obstructive damage to tubules and renal papillae  Some congenital kidney diseases, e.g. autosomal dominant polycystic kidney disease (ADPKD) Note: A kidney in which virtually all nephrons have been destroyed is called an end-stage kidney 46
  • 52. Marc Imhotep Cray, MD Clinical Manifestations (8) 52  End-stage renal disease (ESRD)  GFR is less than 5% of normal this is terminal stage of uremia NDD-CKD vs. ESRD  The term "non-dialysis-dependent chronic kidney disease" (NDD-CKD) is used to encompass status of those persons w established chronic kidney disease (CKD) who do not yet require life-supporting treatments for kidney failure known as renal replacement therapy RRT, including maintenance dialysis or kidney transplantation  Condition of individuals w CKD, who require either of two types of RRT (dialysis or transplant) referred to as the end-stage kidney disease (ESKD)
  • 53. 53 Staging of chronic kidney disease Stevens A, Lowe J, Scott I. Core Pathology, 3rd Ed. St. Louis: Mosby-Elsevier, 2009. NB: Patients with CKD should be prescribed medication with care, as renal disease impairs excretion of certain drugs. Note: Stages 4 and 5 are considered to equate to irreversible CKD.
  • 54. 54 End-stage renal disease, microscopic Klatt EC. Robbins and Cotran Atlas of Pathology, 3rd Ed. Philadelphia: Saunders, 2015.
  • 55. Marc Imhotep Cray, MD Clinical Manifestations (11) 55  Renal tubular defects are dominated by nocturia, and electrolyte disorders (e.g., metabolic acidosis)  A result of diseases that either directly affect tubular structures (e.g., nephronophthisis-medullary cystic disease complex) or cause defects in specific tubular functions o Latter can be inherited (e.g., familial nephrogenic diabetes, cystinuria, renal tubular acidosis [RTA]) or o acquired (e.g., lead nephropathy)
  • 56. Marc Imhotep Cray, MD Clinical Manifestations (12) 56  Urinary tract obstruction and renal tumors have varied clinical manifestations based on specific anatomic location and nature of lesion  Urinary tract infection (UTI) is characterized by bacteriuria and pyuria (bacteria and leukocytes in urine)  Infection may be symptomatic or asymptomatic, and  it may affect kidney (pyelonephritis) or bladder (cystitis)  Nephrolithiasis (renal stones) is manifested by spasms of severe pain (renal colic) and hematuria, often w recurrent stone formation
  • 57. Marc Imhotep Cray, MD Question 57 A patient with acute renal failure is referred for dialysis. The following are all indications for dialysis EXCEPT? A. Severe metabolic acidosis B. Uncontrollable hyperkalemia C. Pulmonary edema D. Pericarditis E. Anemias
  • 58. Marc Imhotep Cray, MD Answer 58 Indications for dialysis include:  Uncontrollable hyperkalemia  Severe metabolic acidosis  Pulmonary edema  Overload of fluid not expected to respond to Tx w diuretics  Uremic complications (pericarditis, encephalopathy, GI bleeding, platelet dysfunction…) and  Intoxication, that is, acute poisoning w a dialyzable substances (eg. SLIME: salicylic acid, lithium, isopropanol, magnesium- containing laxatives, and ethylene glycol) Anemia in a pt. w ARF is not an indication for dialysis.
  • 59. Marc Imhotep Cray, MD Nephrotic vs Nephritic Syndrome Capsule 59  Nephrotic Syndrome  Proteinuria: b/c of disruption of glomerular charge barrier  Hypoalbuminemia: b/c of proteinuria  Edema: b/c of decreased plasma oncotic pressure from proteinuria  Hyperlipidemia & hypercholesterolemia: b/c of ↑ in lipoprotein synthesis  Nephritic Syndrome  Oliguria & Azotemia: b/c of renal inflammation  Hypertension: results from ↓clearance of sodium & water  Hematuria: b/c of leakage of blood into Bowman capsule
  • 60. Marc Imhotep Cray, MD Nephrotic syndrome pathophysiology 60 Heavy proteinuria, exceeds 3.5 g/day in nephrotic syndrome Nephrotic syndrome has diverse causes that share a common pathophysiology a derangement in capillary walls of glomeruli results in ↑ permeability to plasma proteins ↑ permeability of GBM may result from structural or physicochemical alterations With long-standing or heavy proteinuria, serum albumin is ↓ giving rise to hypoalbuminemia  a drop in plasma colloid osmotic pressure which in turn leads to leakage of fluid from bld into extravascular spaces…cont’d next slide
  • 61. Marc Imhotep Cray, MD Nephrotic syndrome cont’d. 61  … resulting ↓in intravascular volume and renal blood flow triggers ↑ release of renin from renal juxtaglomerular cells renin in turn stimulates angiotensin-aldosterone axis which promotes retention of salt and water by kidney  This tendency is exacerbated by reductions in cardiac secretion of natriuretic factors ( ANP & BNP) attributed to ↓ intravascular volume  In face of continuing proteinuria, salt and water retention further aggravates edema  if unchecked may lead to development of generalized edema (termed anasarca)  At onset, there is little or no azotemia, hematuria, or hypertension occurs
  • 62. Marc Imhotep Cray, MD Nephrotic syndrome cont’d. 62  Genesis of hyperlipidemia is more murky  presumably, hypoalbuminemia triggers ↑ synthesis of lipoproteins in liver, or  massive proteinuria causes loss of an inhibitor of lipoprotein synthesis  There is also is abnormal transport of circulating lipid particles and impairment of peripheral breakdown of lipoproteins  associated lipiduria reflects ↑ permeability of GBM to lipoproteins
  • 63. Marc Imhotep Cray, MD Nephrotic syndrome cont’d. 63  Hypercoagulability is a clinically significant manifestation of nephrotic syndrome  caused by renal losses of proteins C & S and antithrombin III , as well as elevated serum fibrinogen and lipid levels Note: Thyroid-binding globulin is also lost, however, this does not appear to be clinically significant, as pts are euthyroid when free T4 is measured
  • 64. Marc Imhotep Cray, MD Pathophysiology of nephrotic syndrome 64 Rubin R, Strayer D (eds.) Rubin’s Pathology. Clinicopathologic Foundations of Medicine, 6th ed. Baltimore: Wolters Kluwer Health, 2012. In nephrotic syndrome, RAS is activated, leading to salt and water retention and edema
  • 65. Marc Imhotep Cray, MD Nephrotic syndrome cont’d. 65 Most important of primary glomerular lesions that lead to nephrotic syndrome are focal segmental glomerulosclerosis and minimal-change disease  minimal-change disease is more important in children  segmental glomerulosclerosis is more important in adults Nephrotic syndrome is also commonly seen in two other primary kidney diseases membranous nephropathy and membranoproliferative glomerulonephritis, and as a complication of systemic disease diabetes mellitus
  • 66. Marc Imhotep Cray, MD Nephritic syndrome 66  Nephritic syndrome is a clinical entity caused by glomerular disease and is dominated by acute onset  either grossly visible hematuria (RBCs in urine) or microscopic hematuria w dysmorphic red cells and red cell casts on urinalysis  diminished GFR  mild to moderate proteinuria (< 3.0 g/day), and  Mild hypertension  It is classic presentation of acute poststreptococcal glomerulonephritis  Rapidly progressive glomerulonephritis (RPGN) is characterized as a nephritic syndrome with rapid decline in GFR (within hours to days)
  • 67. Marc Imhotep Cray, MD Nephritic syndrome cont’d 67  Nephritic syndrome usually has an acute onset and is caused by inflammatory lesions of glomeruli  lesions that cause nephritic syndrome have in common proliferation of cells within glomeruli, often accompanied by an infiltrate of leukocytes  inflammatory reaction injures capillary walls permitting blood to pass into urine, and induces hemodynamic changes that lead to a reduction in GFR
  • 68. Marc Imhotep Cray, MD Nephritic syndrome cont’d 68  Reduced GFR is manifested clinically by oliguria (or anuria), fluid retention, and azotemia  Hypertension is a result of both fluid retention and augmented renin release from ischemic kidneys  acute nephritic syndrome may be caused by primary glomerular diseases, such as  postinfectious glomerulonephritis GN and  various forms of crescentic GN, or  as a result of systemic disorders such as systemic lupus erythematosus, amyloidosis, diabetes, hypertension
  • 69. 69 Etiologies of most common two renal syndromes Syndrome Common Etiologies Uncommon Etiologies Nephrotic syndrome Minimal change disease, focal segmental glomerulosclerosis, diabetic nephropathy Membranous glomerulonephritis (GN)75% of cases are idiopathic, but 2° causes include SLE, penicillamine, gold, NSAIDs, HBV, HCV, syphilis, and malignancy Renal amyloidosis, SLE WHO Class V (membranous form) Nephritic syndrome Postinfectious GN; IgA nephropathy; RPGN, ANCA-associated, pauci- immune GN; SLE Membranoproliferative GN, HCV, cryoglobulinemia, Goodpasture’s syndrome, vasculitides, TTP, HUS, hereditary nephritis (Alport’s syndrome) Redrawn after: Le T and Bhushan V. First Aid for the Wards, 5th Ed. New York: McGraw-Hill, 2013.
  • 70. Marc Imhotep Cray, MD Lecture 2: Glomerular Diseases 70
  • 71. Marc Imhotep Cray, MD Glomerular Diseases 71 Terminology use to describe syndromes is explained in next 4 slides.  Many renal disorders are caused by injury to glomerulus  Glomeruli may be only major site of disease (1° glomerular disease; e.g., immunoglobulin [Ig]A nephropathy) or  Part of a disease affecting several organs (2° glomerular disease; e.g., lupus glomerulonephritis)  Signs (Sn) & symptoms (Sx) of fall into one of following categories:  Asymptomatic proteinuria  Asymptomatic hematuria  Nephrotic syndrome  Acute nephritic syndrome  Rapidly progressive nephritic syndrome  Chronic kidney injury  ESRD
  • 72. Marc Imhotep Cray, MD Terminology describing glomerular syndromes 72 All are histologic determinations after a renal biopsy: Focal vs diffuse: Defines number of glomeruli on biopsy affected  If less than half glomeruli are affected it is focal  If more than half are affected it is diffuse Segmental vs global: Defines how much of each individual glomerulus is affected, for each affected glomerulus  If only part is affected then it is segmental  If entire glomerulus is affected it is global Therefore, focal segmental glomerulosclerosis means  less than half of glomeruli are affected [focal] and, of those affected, only part of each glomerulus is affected [segmental]
  • 73. 73 Patterns of glomerular disease Modified from: Stevens A, Lowe J, Scott I. Core Pathology, 3rd Ed. St. Louis: Mosby-Elsevier, 2009; 367. Global: affecting the whole of the glomerulus uniformly. Segmental: affecting one glomerular segment, leaving other segments unaffected. Diffuse: affecting all glomeruli in both kidneys. Focal: affecting a proportion of glomeruli, others unaffected.
  • 74. Marc Imhotep Cray, MD Terminology describing glomerular syndromes 74  Membranous vs proliferative vs membranoproliferative: Membranous, glomerular basement membrane (GBM) becomes thickened in parts thickenings appear as spikes & domes on microscopy b/c of bulging membrane Proliferative indicates cells are proliferating & numerous nuclei seen on microscopy from added cell count Membranoproliferative indicates membranous thickening & proliferation  leads to a so-called tram track appearance b/c GBM is rebuilt on top of damaged deposits
  • 75. Marc Imhotep Cray, MD Summary of glomerular disorders nomenclature TYPE CHARACTERISTICS EXAMPLE Focal < 50% of glomeruli are involved Focal segmental glomerulosclerosis Diffuse > 50% of glomeruli are involved Diffuse proliferative glomerulonephritis Proliferative Hypercellular glomeruli Membranoproliferative glomerulonephritis Membranous Thickening of glomerular basement membrane(GBM) Membranous nephropathy Primary glomerular disease 1° disease of kidney specifically impacting glomeruli Minimal change disease Secondary glomerular disease Systemic disease or disease of another organ system that also impacts glomeruli SLE, diabetic nephropathy Redrawn after: Le T, Bhushan V, et al. First Aid for the USMLE Step 1 2017. McGraw-Hill Education, 2017.  If only part is affected then it is segmental  If entire glomerulus is affected it is global
  • 76. Marc Imhotep Cray, MD Clinical features of glomerulonephritis relate broadly to histological findings. 76  There are four general rules of thumb which, albeit not absolute, explain vast majority of clinical patterns of disease that relate to each of several types of glomerulonephritis. 1. Structural change in glomerular basement membrane (GBM) (usually thickening) or deposition of excessive mesangial matrix leads to increased loss of protein in urine, which if severe leads to the nephrotic syndrome.
  • 77. Marc Imhotep Cray, MD Clinical features of glomerulonephritis relate to histological findings cont’d. 77 2. Glomerular damage associated with proliferation of endothelial or mesangial cells is associated with the development of microscopic hematuria or nephritic syndrome. 3. If there is both damage to GBM and cell proliferation, a mixed nephritic/ nephrotic syndrome is likely. 4. If damage to glomeruli is rapid and widespread, features of acute renal failure develop.
  • 78. Marc Imhotep Cray, MD Acute versus Chronic Glomerulonephritis 78 Acute (and Rapidly Progressive) Glomerulonephritis  There are several ways to classify acute GN  Light microscopy (LM) essential for establishing areas of injury  Circulating autoantibodies and measures of complement deposition combined w Immunofluorescence (IF) studies and  Electron microscopy (EM)  LM, EM & IF allow GN to be categorized into subgroups correlating w other features of disease  three patterns emerge: 1. Antiglomerular basement membrane ( anti-GBM ) antibody disease 2. Immune complex glomerulonephritis 3. Anti-neutrophil cytoplasmic antibody (ANCA) disease or pauci-immune GN  A brief description of each pattern with disease examples follows…
  • 79. Marc Imhotep Cray, MD Acute versus Chronic GN cont’d. 79 1. Anti-GBM antibody disease (eg, Goodpasture syndrome):  This disease results from development of circulating antibodies to an antigen intrinsic to GBM  Binding of these pathologic anti-GBM antibodies to GBM causes a cascade of inflammation  Light microscopy shows crescentic GN, and characteristic linear immunoglobulin deposition in glomerular capillaries is seen on immunofluorescence
  • 80. Marc Imhotep Cray, MD Acute versus Chronic GN cont’d. 80 2. Immune complex glomerulonephritis:  Immune complex deposition are seen in a variety of diseases  On renal biopsy, granular immunoglobulin deposits are suggestive of immune complexes from underlying systemic disease  A classic example is postinfectious GN in which there is cross- reactivity betw. an antigen of infecting organism and a host antigen resulting in deposition of immune complexes and complement in glomerular capillaries and mesangium o Resolution of glomerular disease typically occurs weeks after Tx of original infection  Other examples include IgA nephropathy, lupus nephritis, & membranoproliferative GN
  • 81. Marc Imhotep Cray, MD Acute versus Chronic GN cont’d. 81 3. Anti-neutrophil cytoplasmic antibody (ANCA) disease or pauci-immune GN:  Characterized by a necrotizing GN but few or no immune deposits (hence, pauci-immune) seen on immunofluorescence or electron microscopy  This pattern is typical of granulomatosis with angiitis, microscopic polyangiitis, or Churg-Strauss syndrome  ANCA-negative pauci-immune necrotizing GN occurs less frequently but is also a well-described clinical entity
  • 82. Marc Imhotep Cray, MD Acute versus Chronic GN cont’d. 82  Rapidly progressive glomerulonephritis (RPGN)  A subset of acute GN in which there is a progressive and dramatic decline (weeks to months) in renal function often leading to complete renal failure and oliguria o It is a nonspecific final pathway in a variety of glomerular diseases  Early disease can be subtle, but is marked by proteinuria and hematuria followed by ↓ GFR  Often called “crescentic GN,” as characteristic finding on biopsy is cellular crescents in Bowman space  Cellular crescents, visible on light microscopy, form in response to severe damage to glomerular capillaries N.B. Recovery without specific treatment is rare
  • 83. Marc Imhotep Cray, MD Acute versus Chronic GN cont’d. 83 Chronic Glomerulonephritis  Some patients with acute GN develop CKD slowly over a period of 5–20 years pathogenesis includes:  Cellular proliferation, in either mesangium or capillary, is a pathologic structural hallmark in some of these cases whereas,  others are notable for obliteration of glomeruli = sclerosing chronic GN includes both focal and diffuse subsets and yet,  others display irregular subepithelial proteinaceous deposits w uniform involvement of individual glomeruli (membranous GN)
  • 84. 84 Chronic glomerulonephritis, gross Klatt EC. Robbins and Cotran Atlas of Pathology, 3rd Ed. Philadelphia: Saunders, 2015.
  • 85. Marc Imhotep Cray, MD Diagnostic features of glomerular diseases 85 I. Light microscopic (LM) features A. Increased cellularity Infiltration by leukocytes (e.g., neutrophils, monocytes, macrophages) Proliferation of “endocapillary” cells (i.e., endothelial and mesangial cells) Proliferation of “extracapillary” cells (i.e., epithelial cells) (crescent formation) B. Increased extracellular material Localization of immune complexes Thickening or replication of GBM Increases in collagenous matrix (sclerosis) Insudation (collection) of plasma proteins (hyalinosis) Fibrinoid necrosis Deposition of amyloid
  • 86. Marc Imhotep Cray, MD Dx features of glomerular diseases (2) 86 II. Immunofluorescence (IF) features A. Linear staining of GBM Anti-GBM antibodies Multiple plasma proteins (e.g., in diabetic glomerulosclerosis) Monoclonal light chains (Amyloid nephropathy ) B. Granular immune complex staining Mesangium (e.g., IgA nephropathy) Capillary wall (e.g., membranous glomerulopathy) Mesangium and capillary wall (e.g., lupus glomerulonephritis) C. Irregular (fluffy) staining Monoclonal light chains (AL amyloidosis) AA protein (AA amyloidosis)
  • 87. Marc Imhotep Cray, MD Dx features of glomerular diseases (3) 87 III. Electron microscopic features A. Electron-dense immune complex deposits Mesangial (e.g., IgA nephropathy) Subendothelial (e.g., lupus glomerulonephritis) Subepithelial (e.g., membranous glomerulopathy) B. GBM thickening (e.g., diabetic glomerulosclerosis) C. GBM replication (e.g., membranoproliferative glomerulonephritis) D. Collagenous matrix expansion (e.g., focal segmental glomerulosclerosis) E. Fibrillary deposits (e.g., amyloidosis) To view plates see: LM, IF and EM of Select Glomerular Diseases_pdf.
  • 88. Marc Imhotep Cray, MD Mechanisms of Glomerular Injury & Disease 88  Immune mechanisms underlie most primary glomerular diseases & many secondary glomerular diseases  Two mechanisms of antibody deposition in glomerulus have been established: 1. deposition of circulating antigen-antibody complexes in glomerular capillary wall or mesangium, and 2. antibodies reacting in situ within glomerulus either with fixed (intrinsic) glomerular antigens or with extrinsic molecules that are planted in glomerulus o Deposition of circulating immune complexes gives a granular immunofluorescence pattern o Anti-GBM antibody GN is characterized by a linear immunofluorescence pattern there is no immune deposit formation in this disease
  • 89. Marc Imhotep Cray, MD Mechanisms of Glomerular Injury & Disease cont’d. 89 Once antigen-antibody complexes are deposited or formed in glomeruli they produce injury by activating complement and recruiting leukocytes  Binding of immune complexes to Fc receptors on leukocytes also may contribute to activation of cells and injury  Morphologically, affected glomeruli exhibit leukocytic infiltrates and proliferation of mesangial and parietal epithelial cells  Electron microscopy reveals electron-dense immune deposits in one or more of three locations: 1. Betw. endothelial cells and GBM (subendothelial deposits) 2. Betw. outer surface of GBM and podocytes (subepithelial deposits), 3. In the mesangium
  • 90. Marc Imhotep Cray, MD Antibody-mediated glomerular injury. Injury can result either from deposition of circulating immune complexes or from antibody-binding to glomerular components followed by formation of complexes in situ 90 Kumar V, Abbas AK, Aster JC. Robbins Basic Pathology, 10th ed. Philadelphia: Elsevier, 2018.
  • 91. Marc Imhotep Cray, MD Two patterns of deposition of immune complexes as seen by immunofluorescence microscopy. (A) Granular, characteristic of circulating and in situ immune complex deposition. (B) Linear, characteristic of classic anti-glomerular basement membrane (anti-GBM) antibody glomerulonephritis. 91 Kumar V, Abbas AK, Aster JC. Robbins Basic Pathology, 10th ed. Philadelphia: Elsevier, 2018
  • 92. 92 Localization of immune complexes in glomerulus: Kumar V, Abbas AK, Aster JC. Robbins and Cotran Pathologic Basis of Disease, 9th ed. Philadelphia: Saunders-Elsevier, 2015. 1. Subepithelial humps as in acute glomerulonephritis 2. Epimembranous deposits as in membranous nephropathy 3. Subendothelial deposits as in lupus nephritis & membranoproliferative glomerulonephritis 4. Mesangial deposits as in IgA nephropathy EN, Endothelium; EP, epithelium; GBM, glomerular basement membrane; LD, lamina densa; LRE, lamina rara externa; LRI, lamina rara interna; MC, mesangial cell; MM, mesangial matrix.
  • 93. Marc Imhotep Cray, MD Clinical presentations of glomerulopathy / glomerulonephritis(GN) 93 Clinical presentation of all various types of GN can take one of six forms: 1. Nephritic syndrome: characterized by oliguria, hematuria, edema, and hypertension 2. Nephrotic syndrome: characterized by massive proteinuria, edema, hypoalbuminemia, and hyperlipidemia 3. Acute renal failure (now called acute kidney injury [AKI]) 4. Chronic renal failure (now called chronic kidney disease[CKD]) 5. Isolated (asymptomatic) proteinuria 6. Isolated (asymptomatic) hematuria Asymptomatic meaning no functional abnormalities assoc. w reduced GFR, edema, or hypertension.
  • 94. Marc Imhotep Cray, MD Suggested approach to reading/studying the data that follows. 94 The discussions of glomerular disorders that follow are presented in a concise and consistent format. Data for each disease commences by presenting a trigger case illustrating the clinical relevance of the pathologic disorder under consideration. Then the etiology and epidemiology, pathology and pathogenesis, clinical manifestations, treatment options and prognosis are presented. Learners are encouraged read the trigger case with close attention and extract all data thought to be relevant to the diagnosis before proceeding to the knowledge-base. This approach should make for good practice in appreciating the intimate relationship of basic science pathology to clinical medicine.
  • 95. Marc Imhotep Cray, MD Trigger Case 1 95 A 5-year-old boy presents to the emergency room with a 1-week history of generalized edema and fatigue. Your history reveals that he suffered from a viral URI 1 week before this visit. Serum and urine studies reveal massive proteinuria, hyperlipidemia, and hypoalbuminemia. You suspect that a renal biopsy would show normal-appearing glomeruli on electron microscopy except for fusion of the epithelial foot processes and you begin the child on prednisone. What is the Diagnosis?
  • 96. Marc Imhotep Cray, MD Minimal Change Disease (Lipoid Nephrosis) 96 Etiology & Epidemiology  Etio. unknown, but usually occurs following a viral URI  also assoc. w Hodgkin disease & hypersensitivity reactions  Most often seen in young children, but can occur in older children and adults Pathology  LM: Normal-appearing glomeruli can see lipid accumulation in renal tubular cells  EM: Fusion of epithelial foot processes
  • 97. Marc Imhotep Cray, MD MCD cont’d. 97 Clinical Manifestations Nephrotic syndrome  Complications include infection by gram-positive organism, thromboembolism, and protein malnutrition Treatment (Tx) & Prognosis (Px) Prednisone; cyclophosphamide or chlorambucil for steroid-resistant cases  Response is excellent Note: Minimal change disease is the prototype of nephrotic syndrome
  • 98. Marc Imhotep Cray, MD MCD (A) When viewed with a LM, silver methenamine–stained glomerulus appears normal, w a delicate basement membrane. (B) Schematic diagram illustrating diffuse effacement of foot processes of podocytes with no immune deposits. 98 Kumar V, Abbas AK, Aster JC. Robbins and Cotran Pathologic Basis of Disease, 9th ed. Philadelphia: Saunders-Elsevier, 2015.
  • 99. 99 MCD, Electron micrograph Klatt EC. Robbins and Cotran Atlas of Pathology, 3rd Ed. Philadelphia: Saunders, 2015.
  • 100. Marc Imhotep Cray, MD Trigger Case 2 100 A 40-year-old woman with a history of SLE presents to your office with a chief complaint of increased swelling in her legs. She had been referred by her primary care physician, who suspected a secondary illness to her lupus. Recent laboratory studies show proteinuria, hypoalbuminemia, hyperlipidemia, and hypercholesterolemia. You suspect that a renal biopsy would demonstrate immune complex deposition on electron microscopy as well as a “spike and dome” appearance on silver methenamine stain. What is the Diagnosis?
  • 101. 101 Membranous Glomerulonephritis (also called M. Glomerulopathy or M. Nephropathy) Etio. & Epidem.  An immune complex disease of unknown etiology  Secondary disease seen in 10% of SLE patients (type V Lupus Nephritis)  is sometimes assoc. w infections (eg, hepatitis B and C, syphilis, malaria), drugs (eg, gold salts, penicillamine, NSAIDs), or malignancy  Incidence is highest in adults Pathology  LM: Diffuse capillary wall thickening & basement membrane (BM) thickening  IF: Granular pattern of IgG or C3 deposits (lumpy-bumpy)  EM: Electron dense immune complex deposition in subepithelial locations within BM of glomerular capillary walls  Silver methenamine stain: A spike-and-dome appearance resulting from extension of BM betw. and around immune deposits (spikes = basement membrane, domes = immune complex deposits)
  • 102. Marc Imhotep Cray, MD Membranous Glomerulopathy cont’d. 102 Clinical Manifestations Nephrotic syndrome accompanied by azotemia  Complications include renal vein thrombosis and higher incidence of occult neoplasms of lung, stomach, and colon Tx Cyclophosphamide or steroids  ACE inhibitors (reduce urinary protein loss)  renal transplantation for severe cases Note: Membranous glomerulopathy is second most common cause of nephrotic syndrome in adults, w focal segmental glomerulosclerosis (FSGS) recently becoming most common
  • 103. 103 Membranous nephropathy A. Silver methenamine stain. Note marked diffuse thickening of capillary walls without an increase in number of cells. There are prominent “spikes” of silver-staining matrix (arrow) projecting from basement membrane lamina densa toward urinary space, which separate and surround deposited immune complexes that lack affinity for silver stain. B. EM showing electron-dense deposits(arrow) along epithelial side of basement membrane (B). Note effacement of foot processes overlying deposits. CL, Capillary lumen; End, endothelium; Ep, epithelium; US, urinary space. C. Characteristic granular immunofluorescent deposits of IgG along glomerular basement membrane. D. Diagrammatic representation of membranous nephropathy. Kumar V, Abbas AK, Aster JC. Robbins and Cotran Pathologic Basis of Disease, 9th ed. Philadelphia: Saunders-Elsevier, 2015.
  • 104. Marc Imhotep Cray, MD Trigger Case 3 104 A 40-year-old HIV-positive man is admitted to the hospital complaining of generalized edema and fatigue. A complete history reveals that he is a habitual IV drug user. Laboratory studies show hypoalbuminemia, hyperlipidemia, proteinuria, and microscopic hematuria. You suspect that his current presentation is related to his HIV and you prepare the patient for a renal biopsy to determine the exact diagnosis. What is the Diagnosis?
  • 105. Marc Imhotep Cray, MD Focal Segmental Glomerulosclerosis (FSGS) 105 Etio. & Epidem.  Often idiopathic; has been assoc. w heroin use, HTN, prior GN and HIV infection (=collapsing glomerulopathy)  Most often occurs in older patients Pathology  LM: Sclerosis within capillary tufts of deep juxtaglomerular glomeruli w focal and segmental distribution  hyalinosis (deposition of hyaline masses) also seen  IF: granular mesangial fluorescence for IgM and C3  EM: Fusion of epithelial foot processes
  • 106. Marc Imhotep Cray, MD FSGS cont’d. 106 Clinical Manifestations nephrotic syndrome  more severe disease in HIV and IV drug users  Lab findings: 80% have microscopic hematuria at presentation Tx & Px Prednisone  Most patients progress to ESRD in 5–10 years Notes:  FSGS most common cause of nephrotic syndrome in adults in U.S.  b/c of focal nature of FSGS, early cases can be difficult to distinguish from MCD (How might one differentiate the two clinically?)  FSGS, w no cellular proliferation, is different from focal segmental glomerulonephritis (FSGN), which involves cellular proliferation
  • 107. Marc Imhotep Cray, MD FSGS, microscopic (PAS stain) 107 A Low-power view showing segmental sclerosis in one of three glomeruli (at 3 o’clock) B High-power view showing hyaline insudation (arrow) and lipid (small vacuoles) in sclerotic area Kumar V, Abbas AK, Aster JC. Robbins and Cotran Pathologic Basis of Disease, 9th ed. Philadelphia: Saunders-Elsevier, 2015.
  • 108. Marc Imhotep Cray, MD Collapsing Glomerulopathy 108 Visible are retraction of glomerular tuft (arrows), narrowing of capillary lumens, proliferation and swelling of visceral epithelial cells (double arrows), and prominent accumulation of intracellular protein absorption droplets in the visceral epithelial cells (arrowheads). Kumar V, Abbas AK, Aster JC. Robbins and Cotran Pathologic Basis of Disease, 9th ed. Philadelphia: Saunders-Elsevier, 2015.  A morphologic variant of FSGS, characterized by retraction and/or collapse of entire glomerular tuft  May be idiopathic, but it also has been assoc. w some drug toxicities (e.g., pamidronate)  Most characteristic lesion of HIV-associated nephropathy  Typically assoc. w prominent tubular injury  It has a very poor prognosis
  • 109. Marc Imhotep Cray, MD Trigger Case 4 109 A 60-year-old African Americans man with a 20-year history of type II DM presents for a nephrology consult after his primary care physician found a progressive increase of proteinuria in recent laboratory studies. These laboratory studies also showed hyperlipidemia and hypercholesterolemia. On physical examination, the patient has bilateral diabetic retinopathy and 2+ edema in both legs. You start the patient on an ACE inhibitor and you suspect that a renal biopsy would show Kimmelstiel-Wilson nodules. What is the Diagnosis?
  • 110. Marc Imhotep Cray, MD Diabetic Nephropathy 110 Etio. & Epidem. Assoc. w long-standing diabetes  T1DM carries 30%–40% chance of diabetic nephropathy after 20 years  T2DM carries 15%–20% chance after 20 years however, b/c there are more pts w type II diabetes ESRD is more prevalent among type II diabetics  higher risk of developing diabetic nephropathy among men, African Americans and Native Americans w DM Pathology LM: Increase in mesangial matrix, resulting in either  diffuse glomerulosclerosis (diffusely distributed ↑ in mesangial matrix) or  nodular glomerulosclerosis (Kimmelstiel- Wilson nodules—nodular accumulations of mesangial matrix material)  EM: Striking ↑ in GBM thickening
  • 111. Marc Imhotep Cray, MD Diabetic Nephropathy cont’d. 111 Clinical Manifestations  Nephrotic syndrome  diabetic retinopathy is invariably present  Lab findings: Microalbuminuria (early sign), proteinuria (late sign) Tx & Px  Strict glycemic control  Treatment of hypertension and microalbuminuria w ACE inhibitors during early stages to slow progression  Often progresses to ESRD and dialysis Note: Diabetic nephropathy is most common cause of ESRD in U.S.
  • 112. 112 Nodular glomerulosclerosis, microscopic Klatt EC. Robbins and Cotran Atlas of Pathology, 3rd Ed. Philadelphia: Saunders, 2015.
  • 113. 113 Diffuse glomerulosclerosis, microscopic Klatt EC. Robbins and Cotran Atlas of Pathology, 3rd Ed. Philadelphia: Saunders, 2015.
  • 114. 114 A 60-year-old man complains of chronic back pain and fatigue, excessive urination, and increased thirst. X-ray examination reveals numerous lytic lesions in the lumbar vertebral bodies. Laboratory studies show hypoalbuminemia, mild anemia, and thrombocytopenia. Urinalysis displays 4+ proteinuria. A monoclonal immunoglobulin light-chain peak is demonstrated on serum electrophoresis. A bone marrow biopsy discloses foci of plasma cells, which account for 20% of all hematopoietic cells. A kidney biopsy is obtained (shown in image). Which of the following is the most likely cause of nephrotic syndrome in this patient? A. Amyloid nephropathy B. Crescentic glomerulonephritis C. IgA nephropathy (Berger disease) D. Membranous glomerulonephritis E. Nodular glomerulosclerosis (Kimmelstiel-Wilson disease) Question
  • 115. Marc Imhotep Cray, MD 115 Diagnosis (A) , Amyloid nephropathy, multiple myeloma Neoplastic plasma cells typically secrete a homogeneous immunoglobulin chain, which can be detected in serum or urine by electrophoresis. Amyloid nephropathy is caused by the deposition of secreted light chains in the glomerular basement membranes and mesangial matrix. Amorphous acellular material expands the mesangium and obstructs the glomerular capillaries. Deposits of AL amyloid may also appear in the tubular basement membranes and in the walls of renal vessels. Renal amyloidosis usually presents with nephrotic syndrome. The deposits of amyloid may take on a nodular appearance, reminiscent of Kimmelstiel-Wilson lesion of diabetic glomerulosclerosis (choice E). However, amyloid deposits are not PAS positive and are identifiable by Congo red staining with characteristic apple-green birefringence. IgA nephropathy (choice C) and membranous glomerulonephritis (choice D) are unrelated to light-chain disease.
  • 116. 116 Renal Amyloidosis  Etiology:  Amyloidosis is complication of chronic inflammatory disorders, such as RA, osteomyelitis or CA= multiple myeloma etc.  kidneys, liver, spleen, and adrenals most common organs involved  accumulation of proteins in form of abnormal, insoluble fibers=amyloid fibrils  Pathology:  Amyloidosis leads to nephrotic syndrome and renal failure  LM-Congo red stain shows apple-green birefringence under polarized light due to amyloid deposition in mesangium  Diagnosis: Congo red staining or tissue biopsy gold standard methods of Dx Amyloid nephropathy. In a section stained w Congo red and examined under polarized light, amyloid deposits in glomerulus and adjacent arteriole show a characteristic apple-green birefringence Rubin R, Strayer D (eds.) Rubin’s Pathology. Clinicopathologic Foundations of Medicine, 6th ed. Baltimore: Wolters Kluwer Health, 2012.
  • 117. 117 Amyloid nephropathy Disorder is initially assoc. w accumulation of characteristic fibrillar deposits in mesangium. These inert masses, which are fibrillar by EM, extend along inner surface of basement membrane (BM), frequently obstructing capillary lumen. Focal extension of amyloid through BM may elevate epithelial cell, in which case irregular spikes are seen along outer surface of BM Amyloid nephropathy, EM Deposits of fibrils (10 nm diameter) in a glomerulus adjacent to podocyte cytoplasm with effaced foot processes. Rubin R, Strayer D (eds.) Rubin’s Pathology. Clinicopathologic Foundations of Medicine, 6th ed. Baltimore: Wolters Kluwer Health, 2012.
  • 118. Marc Imhotep Cray, MD Trigger Case 5 118 A 45-year-old white woman with a 15-year history of SLE presents to the nephrologist after her primary care physician found hematuria and proteinuria on a routine urinalysis. The patient has edema of the ankles on physical examination and she is experiencing a SLE flare-up with an extensive malar rash visible over her face. A renal biopsy is obtained and examination by light microscopy reveals wire-loop abnormalities. You examine the patient’s current immunosuppressive therapy to see what additional therapies should be added. What is the Diagnosis?
  • 119. Marc Imhotep Cray, MD Lupus Nephropathy 119 Etiology Renal component of systemic lupus erythematosus Pathology Five distinct renal histologic patterns:  (1) Type I: normal  (2) Type II (mesangial form): focal & segmental glomerular involvement w ↑ in mesangial matrix  (3) Type III (focal proliferative form): involves less than half of glomeruli, causing extensive damage to individual glomeruli  (4) Type IV (diffuse proliferative form): most severe form involving all glomeruli w marked inflammation, mesangial proliferation, and scarring o LM: wire-loop abnormality caused immune complex deposition and gross thickening of GBM o EM: endothelial cell proliferation o IF: marked subendothelial immune complex deposition= granular pattern  (5) Type V (membranous form): similar to membranous glomerulonephritis
  • 120. Marc Imhotep Cray, MD Lupus Nephropathy cont’d. 120  Clinical Manifestations  Type I: No clinical findings  Types II and III: Mild to moderate proteinuria and hematuria  Type IV: Combination of nephrotic and nephritic syndromes  Type V: Nephrotic syndrome  Tx  Types I and II: No treatment required  Types III, IV, and V: Immunosuppression (corticosteroids, cyclophosphamide, and/or azathioprine) Note: Renal lesion severity often determines overall prognosis of SLE patients
  • 121. 121 A, Focal proliferative glomerulonephritis, with two focal necrotizing lesions at the 11 0'clock and 2 0'clock positions (H&E stain). Extracapillary proliferation is not prominent in this case. B, Diffuse proliferative glomerulonephritis. Note marked increase in cellularity throughout glomerulus (H&E stain). Lupus nephritis Kumar V, Abbas AK, Aster JC. Robbins and Cotran Pathologic Basis of Disease, 9th ed. Philadelphia: Saunders-Elsevier, 2015.
  • 122. 122Kumar V, Abbas AK, Aster JC. Robbins and Cotran Pathologic Basis of Disease, 9th ed. Philadelphia: Saunders-Elsevier, 2015. Lupus nephritis C, Lupus nephritis showing a glomerulus w several 'Wire loop" lesions representing extensive subendothelial deposits of immune complexes (PSA stain). D, EM of a renal glomerular capillary loop from a patient w SLE nephritis. Subendothelial dense deposits (arrowheads) correspond to "wire loops" seen by light microscopy. B (with arrow) refers to the basement membrane.
  • 123. 123 E, Deposition of lgG antibody in a granular pattern, detected by immunofluorescence. Lupus nephritis Kumar V, Abbas AK, Aster JC. Robbins and Cotran Pathologic Basis of Disease, 9th ed. Philadelphia: Saunders-Elsevier, 2015.
  • 124. Marc Imhotep Cray, MD Trigger Case 6 124 A 10-year-old girl presents to the clinic complaining of eye swelling. You note that the child was seen 3 weeks ago in clinic for a chief complaint of sore throat. Upon taking a history and performing a physical, you find that the patient has pronounced periorbital edema, has been urinating very little despite adequate fluid intake, and has a blood pressure of 150/90. Laboratory findings include azotemia, hematuria, red cell casts in the urine, and an elevated ASO antibody titer. You reassure the parents that their child’s condition will likely resolve on its own. What is the Diagnosis?
  • 125. Marc Imhotep Cray, MD Poststreptococcal Glomerulonephritis (Acute Proliferative GN/ Acute GN) 125 Etio. & Epidem. Most frequently seen in children (6-10 yrs.) following infection w nephritogenic strains of group A β-hemolytic streptococci  decreasing in frequency in U.S but, fairly common disorder worldwide Pathology  Gross : Characterized by intense inflammatory reaction involving all glomeruli in both kidneys resulting in punctate hemorrhages on kidney surfaces  LM: Enlarged, hypercellular, swollen glomeruli w proliferation of mesangial and endothelial cells; normal GBM thickness  EM: Electron-dense humps on the epithelial side of the basement membrane (subepithelial localization)  IF: Coarse granular immunofluorescence for IgG or C3 (lumpy-bumpy)
  • 126. Marc Imhotep Cray, MD Poststreptococcal GN cont’d. 126 Clinical Manifestations Nephritic syndrome & periorbital edema; pulmonary congestion not uncommon  Lab findings: UA RBCs and/or red cell casts, decreased serum C3, elevated ASO antibody titer (evidence of recent streptococcal infection) Tx & Px Resolves spontaneously  More than 95% of affected children eventually recover renal function w conservative therapy aimed at maintaining sodium and water balance  In adults disease is less benign in up to 40% affected in epidemics fail to resolve quickly Note: Poststreptococcal glomerulonephritis is an immune complex disease w antigen-antibody complex of streptococcal origin and is prototype of nephritic syndrome & prototypical glomerular disease of immune complex etiology
  • 127. 127 Nonstreptococcal Acute Glomerulonephritis (Postinfectious Glomerulonephritis) A similar form of glomerulonephritis occurs sporadically in assoc. w other infections, including those of bacterial (e.g., staphylococcal endocarditis, pneumococcal pneumonia, and meningococcemia), viral (e.g., hepatitis B, hepatitis C, mumps, HIV infection, varicella, and infectious mononucleosis), and parasitic (malaria, toxoplasmosis) origin  In these settings, granular immunofluorescent deposits and subepithelial humps characteristic of immune complex nephritis are present  Postinfectious glomerulonephritis due to staphylococcal infections differs by sometimes producing immune deposits containing IgA rather than IgG
  • 128. 128 A Normal glomerulus. B Glomerular hypercellularity is due to intracapillary leukocytes and proliferation of intrinsic glomerular cells. C Typical electron-dense subepithelial “hump” and a neutrophil in lumen. D Immunofluorescent stain demonstrates discrete, coarsely granular deposits of complement protein C3 (stain for IgG was similar), corresponding to “humps” illustrated in part C. Acute proliferative glomerulonephritis Kumar V, Abbas AK, Aster JC. Robbins and Cotran Pathologic Basis of Disease, 9th ed. Philadelphia: Saunders-Elsevier, 2015.
  • 129. Marc Imhotep Cray, MD Trigger Case 7 129 A 40-year-old man is admitted to the hospital with complaints of blood in his sputum and urine. A thorough history also reveals fever, malaise, and a 10-pound weight loss over the past month. On physical examination, you find that his blood pressure is 160/95 and that he has several abnormal lung sounds. A urine dipstick demonstrates hematuria. CXR reveals several nodular lesions and blood tests show the presence of C-ANCA and an elevated ESR. You start the patient on a high dose of corticosteroids and you suspect that a renal biopsy would demonstrate crescent moon shapes between the Bowman capsule and the glomerular tuft. What is the Diagnosis?
  • 130. 130 Rapidly Progressive (Crescentic) Glomerulonephritis  RPGN is a clinical syndrome assoc. w severe glomerular injury, but does not denote a specific etiologic form of GN  Type 1=20% (Anti-GBM antibody-mediated disease); Idiopathic or Goodpasture syndrome (=Ab against BM lung alveoli hemoptysis)  Type 2=25% (immune complexes): Idiopathic; postinfectious GN; SLE; IgA nephropathy; Henoch- Schönlein purpura  Type 3=55% (pauci-immune type): Idiopathic; ANCA-assoc. vasculitides= Wegener granulomatosis; microscopic polyangiitis etc. Pathology  LM: Formation of crescent-moon shape betw. Bowman capsule and glomerular tuft resulting from deposition of fibrin in Bowman space and proliferation of parietal epithelial cells of Bowman capsule  IF: Type 1, linear pattern deposits of IgG & C3; Type 2=granular deposits IgG &C3
  • 131. Marc Imhotep Cray, MD RPGN cont’d. 131 Clinical Manifestations  Nephritic syndrome progressing rapidly to renal failure within months  Sn & Sx specific to each etiology (eg, hemoptysis and anti- GBM antibodies in Goodpasture syndrome) Tx & Px  Treat w diuretics and often eventual dialysis  immunosuppression if appropriate for underlying cause  may require transplantation  Rapid course to renal failure Note: RPGN refers to a syndrome assoc. w severe and progressive glomerular injury It encompasses many different etiologies
  • 132. Marc Imhotep Cray, MD RPGN Schematic 132  Rapidly proliferating epithelial crescent crushes glomerular tuft, which may show a range of changes including o focal proliferative GN, o segmental tuft necrosis, o mesangiocapillary GN, or o acute proliferative GN Note: An epithelial crescent is result of breaks in GBM permitting leak of fibrin, blood proteins and WBCs (particularly monocyte/ macrophages) into urinary space Stevens A, Lowe J, Scott I. Core Pathology, 3rd Ed. St. Louis: Mosby- Elsevier, 2009.
  • 133. 133 Rapidly progressive glomerulonephritis, microscopic Klatt EC. Robbins and Cotran Atlas of Pathology, 3rd Ed. Philadelphia: Saunders, 2015.
  • 134. Marc Imhotep Cray, MD RPGN, Electron micrograph 134 EM showing characteristic wrinkling of GBM w focal disruptions (arrows). Kumar V, Abbas AK, Aster JC. Robbins and Cotran Pathologic Basis of Disease, 9th ed. Philadelphia: Saunders-Elsevier, 2015.
  • 135. 135 RPGN, immunofluorescence Klatt EC. Robbins and Cotran Atlas of Pathology, 3rd Ed. Philadelphia: Saunders, 2015.
  • 136. Marc Imhotep Cray, MD Trigger Case 8 136 A 10-year-old boy brought to the clinic by his mother complaining of a red tinge to his urine. A more detailed history reveals that he was diagnosed with mild nerve deafness 2 years earlier and also developed posterior cataracts 1 year ago. Laboratory studies confirm hematuria as well as the presence of erythrocyte casts. You begin to wonder if his conditions may be related to a genetic disorder. What is the Diagnosis?
  • 137. Marc Imhotep Cray, MD Alport Syndrome 137  Etiology: Genetic disorder w heterogenous inheritance (usually X-linked dominant) results in mutation of α-5 chain of type IV collagen  caused by mutations in COL4A3, COL4A4, and COL4A5, three of six genes involved in basement membrane (type IV) collagen biosynthesis  Pathology EM: Irregular foci of thickening or attenuation in GBM with longitudinal splitting of lamina densa  Clinical Manifestations: Triad of nephritis, nerve deafness, and various eye disorders (cataracts, lens dislocation, corneal dystrophy)  often initially presents w hematuria and RBC casts  Treatment ACE inhibitors; renal transplantation
  • 138. 138 Alport syndrome, microscopic Klatt EC. Robbins and Cotran Atlas of Pathology, 3rd Ed. Philadelphia: Saunders, 2015.
  • 139. Marc Imhotep Cray, MD Trigger Case 9 139 A 25-year-old woman with a history of SLE is admitted to the hospital with generalized edema, malaise, and fatigue. You take a thorough history, which reveals that she had cold symptoms 2 weeks earlier. Laboratory studies show hypoalbuminemia, hypercholesterolemia, proteinuria, and low complement levels. When a renal biopsy shows reduplication of the basement membrane on electron microscopy, you adjust the patient’s current corticosteroid dose and decide to add an antiplatelet drug to her regimen. What is the Diagnosis?
  • 140. Marc Imhotep Cray, MD Diffuse Membranoproliferative GN (Mesangiocapillary GN) 140 Etio. and Epidem. Assoc. w inherited complement component deficiency  Type I MPGN (90%) is seen in SLE, hepatitis B and C, and involves classic & alternative pathway activation  Type II MPGN (10%) (dense deposit disease) dysregulation of alternative complement pathway  Most patients are under the age of 30  Pathology Types I and II:  LM: reduplication of basement membrane (splitting) and lobular proliferation of mesangial matrix into capillary loops (tram track appearance)  Type I: EM: subendothelial electron-dense deposits  Type II: EM: dense deposit of homogeneous material within GBM
  • 141. Marc Imhotep Cray, MD Membranoproliferative GN cont’d. 141 Clinical Manifestations  Type I: Commonly presents w nephrotic syndrome  Type II: Commonly presents w hematuria and chronic renal failure  Lab Findings: Low complement levels (↓ C3) , ↑BUN and Cr, RBCs and/or RBC casts in urine  Tx & Px: Corticosteroids (poor response) and immunosuppression if appropriate for underlying cause  Type II, particularly, poor prognosis slowly progresses to CKD  50% develop CKD within 10 year  Both types have high incidence of recurrence of disease in transplanted kidneys
  • 142. Marc Imhotep Cray, MD Schematic representation of MPGN 142 Patterns in two types of MPGN  In type I there are subendothelial deposits  Type II is characterized by intramembranous dense deposits (=dense-deposit disease)  In both, BM appear split when viewed in light microscope Kumar V, Abbas AK, Aster JC. Robbins and Cotran Pathologic Basis of Disease, 9th ed. Philadelphia: Saunders-Elsevier, 2015.
  • 143. 143 MPGN, microscopic Klatt EC. Robbins and Cotran Atlas of Pathology, 3rd Ed. Philadelphia: Saunders, 2015.
  • 144. Marc Imhotep Cray, MD A MPGN, type I. Note discrete electron-dense deposits (arrows) incorporated into glomerular capillary wall betw. duplicated (split) basemen membranes (double arrows), and in mesangial regions (M); CL, Capillary lumen. B Dense-deposit disease (type II MPGN). There are dense homogeneous deposits within basement membrane. In both, mesangial interposition gives appearance of split BM when viewed in LM 144 Kumar V, Abbas AK, Aster JC. Robbins and Cotran Pathologic Basis of Disease, 9th ed. Philadelphia: Saunders-Elsevier, 2015.
  • 145. Marc Imhotep Cray, MD Question 145 A 26-year-old woman with a history of mitral valve prolapse comes in with 1 week of fever that started 3 days after a dental procedure. Her urine contains red cells and her rheumatoid factor is elevated. Which of the following serologic abnormalities is expected to be present? A. Anti-GBM antibody B. Low serum complement levels C. Antineutrophil cytoplasmic antibody D. Elevated IgA levels
  • 146. Marc Imhotep Cray, MD Answer 146 The answer is B. (Robbins, 9th/e, p 926.) This clinical description is classic for membranoproliferative glomerulonephritis, which is associated with bacterial endocarditis. Low complement levels are typically found in membranoproliferative glomerulonephritis, but not in the other disorders listed. Choice A. describes anti-GBM antibody disease or Goodpasture’s syndrome when the lung is involved. Choice C. describes pauci-immune glomerulonephritis, such as Wegener’s. Choice D. describes IgA nephropathy.
  • 147. Marc Imhotep Cray, MD Trigger Case 10 147 A 15-year-old Asian boy presents to the emergency room complaining of blood in his urine. Upon taking a complete history, you learn that he has also been suffering from fevers, myalgias, and arthralgias for the last 2 days. Serum studies reveal increased serum IgA levels and normal serum complement levels. You begin him on prednisone and you suspect that he is afflicted with the most common form of acute glomerulonephritis in the United States. What is the Diagnosis?
  • 148. Marc Imhotep Cray, MD IgA Nephropathy (Berger Disease) 148 Etio. & Epidem. Primary renal disease of IgA deposition in glomerular mesangium  can manifest after infection (viral URI, GI infection, flu-like syndrome) or can be a component of Henoch-Schönlein purpura  Most commonly seen in children and young adults w men affected more often than women Pathology  LM: mesangial widening and focal and segmental inflammation  EM: Mesangial deposits of IgA
  • 149. Marc Imhotep Cray, MD IgA Nephropathy (Berger Disease) cont’d. 149 Clinical Manifestations Presents w recurrent hematuria (red or cola-colored urine) 1–2 days after an infection  Lab findings: ↑ serum IgA level (50% of cases), nml serum complement levels Tx & Px Prednisone  Can progress to chronic renal failure 25-50% go on to develop CKD within 20 yrs. Note: Berger disease is most common form of acute GN in U.S. and worldwide
  • 150. 150 IgA nephropathy (Berger disease), microscopic Klatt EC. Robbins and Cotran Atlas of Pathology, 3rd Ed. Philadelphia: Saunders, 2015.
  • 151. 151 IgA nephropathy, immunofluorescence Klatt EC. Robbins and Cotran Atlas of Pathology, 3rd Ed. Philadelphia: Saunders, 2015.
  • 152. 152Rubin R, Strayer D (eds.) Rubin’s Pathology. Clinicopathologic Foundations of Medicine, 6th ed. Baltimore: Wolters Kluwer Health, 2012. Algorithm demonstrating integration of pathologic findings with clinical data to make a diagnosis of a specific form of primary or secondary glomerulonephritis.
  • 153. Nephritic syndrome—due to GBM disruption. Hypertension, ↑ BUN and creatinine, oliguria, hematuria, RBC casts in urine. Proteinuria often in the subnephrotic range (< 3.5 g/day) but in severe cases may be in nephrotic range. • Acute poststreptococcal glomerulonephritis • Rapidly progressive glomerulonephritis • IgA nephropathy (Berger disease) • Alport syndrome • Membranoproliferative glomerulonephritis Nephritic-nephrotic syndrome—severe nephritic syndrome with profound GBM damage that damages the glomerular filtration charge barrier → nephrotic-range proteinuria (> 3.5 g/day) and concomitant features of nephrotic syndrome. Can occur with any form of nephritic syndrome, but is most commonly seen with: • Diffuse proliferative glomerulonephritis • Membranoproliferative glomerulonephritis • Lupus Nephropathy Type IV Nephrotic syndrome—podocyte disruption → charge barrier impaired. Massive proteinuria (> 3.5 g/day) with hypoalbuminemia, hyperlipidemia, edema. May be 1° (eg, direct podocyte damage) or 2° (podocyte damage from systemic process [eg, diabetes]). • Focal segmental glomerulosclerosis (1° or 2°) • Minimal change disease (1° or 2°) • Membranous nephropathy (1° or 2°) • Amyloidosis (2°) • Diabetic glomerulonephropathy (2°) Glomerular diseases capsule 0.25 3.5 > 3.5 GRAMS OF PROTEIN EXCRETED PER DAY (g/day)
  • 154. Marc Imhotep Cray, MD Lecture 3: Urinary Tract Infections Tubulointerstitial diseases Obstructive uropathy, hydronephrosis & urolithiasis 154
  • 155. Marc Imhotep Cray, MD Urinary Tract Infections 155  A urinary tract infection (UTI) is an infection that affects part of urinary tract  When it affects lower urinary tract it is known as a bladder infection (cystitis)  When it affects upper urinary tract it is known as kidney infection (pyelonephritis [discussed under tubulointerstitial diseases] )  Cystitis  Clinical features: dysuria in absence of vaginal discharge and significant pain, frequency, urgency, suprapubic pain
  • 156. Marc Imhotep Cray, MD UTI, Cystitis cont’d. 156 Etio-pathogenesis:  Bacteria gain access to urinary tract via urethra  Cystitis most frequently involves normal colonic flora o Escherichia coli is most common cause (approx. 80%) o Proteus, Klebsiella, and Enterobacter are also implicated o Staphylococcus saprophyticus causes 10% to 15% of infections in young women o Nosocomial cystitis is frequently caused by Pseudomonas or Staphylococcus aureus
  • 157. Marc Imhotep Cray, MD UTI, Cystitis cont’d. 157  Epidemiology  Women have a higher incidence of infection b/c they have shorter urethras  Other risk factors include sexual activity, pregnancy, urinary obstruction, neurogenic bladder, and vesicoureteral reflux-8 Diagnostic findings  Characteristic clinical features are present  Pyuria (more than 8 leukocytes/high-power field)  Bacterial culture yields >105 organisms/mL Treatment  antibiotics  Recurrent cystitis may require prophylactic antibiotics
  • 158. 158 Acute cystitis Patient died 2 days after surgery, and cystitis was caused by an indwelling catheter A. Several foci of hemorrhage are seen on hyperemic bladder mucosa B. Foci of mucosal hemorrhage C. Acute cystitis. Polymorphonuclear leukocytes infiltrate mucosa Rubin R , Strayer DS Eds. Rubin’s Pathology: Clinicopathologic Foundations of Medicine, 6th Ed. Baltimore: Lippincott Williams & Wilkins, 2012.
  • 159. Marc Imhotep Cray, MD Tubulointerstitial Diseases 159 Most forms of tubular injury also involve interstitium so they are discussed together= tubulointerstitial diseases Diseases involving tubules and interstitium may have clinical manifestations of nephritic syndrome, or of specific defects in tubular function, or of acute or chronic kidney disease without more specific defining features In this section we will discuss diseases characterized by: 1. Inflammatory involvement of tubules and interstitium (tubulointerstitial nephritis)  Acute and chronic pyelonephritis (Upper UTI) 2. Ischemic and toxic tubular injury leading to acute tubular injury and clinical syndrome of acute kidney injury (ARF)
  • 160. Marc Imhotep Cray, MD Tubulointerstitial Diseases cont’d. 160  Acute tubular necrosis (ATN)-now called acute tubular injury (ATI)- is usually due to profound hypotension causing ischemic damage to tubular epithelial cells  Infections include pyelonephritis, renal abscesses & TB  Drug toxicity usually causes tubulointerstitial nephritis due to a hypersensitivity reaction, but  other patterns may occur, including direct toxicity to tubular epithelial cells giving an appearance similar to ATN  Mechanical obstruction of ureters or bladder may lead to hydronephrosis and recurrent infection
  • 161. Marc Imhotep Cray, MD Tubulointerstitial Nephritis 161 Tubulointerstitial nephritis (TIN) refers to a group of inflammatory kidney diseases that primarily involve interstitium & tubules  glomeruli may be spared altogether or affected only late in course In cases of TIN caused by bacterial infection renal pelvis is prominently involved hence a more descriptive term is pyelonephritis (from pyelo, “pelvis”)
  • 162. Marc Imhotep Cray, MD Tubulointerstitial Nephritis cont’d. 162 Term interstitial nephritis is reserved for cases of TIN that are nonbacterial in origin includes  tubular injury resulting from drugs  metabolic disorders such as hypokalemia  irradiation  viral infections, and  immune reactions o For example, acute drug-induced interstitial nephritis caused by penicillin derivatives (eg, methicillin), NSAIDs, and diuretics On basis of clinical manifestations & character of inflammatory exudate TIN can be divided into acute and chronic categories
  • 163. Marc Imhotep Cray, MD Trigger Case 11 163 A 25-year-old woman presents to the emergency room with fever, severe flank pain, and costovertebral angle tenderness. After taking a complete history, you find that she is sexually active and has had a 2-week history of burning pain while urinating and increased urinary frequency. Urinalysis reveals white cell casts in the urine and a urine sample is sent for culture. While you await the culture results, you start her on broad spectrum antibiotics. What is the Diagnosis?
  • 164. Marc Imhotep Cray, MD Pyelonephritis (Acute and Chronic) 164  Etio. & Epidem.  Acute: Caused by infection of renal parenchyma more frequent among women  Chronic: Results from chronic urinary tract obstruction & recurrent UTIs  Pathology Acute: Affects renal cortex w sparing of glomeruli  neutrophilic infiltration and abscess formation within renal interstitium o abscesses may rupture introducing WBCs into tubular lumen Chronic: Asymmetric corticomedullary scarring  tubules contain eosinophilic, proteinaceous casts resulting in gross appearance reminiscent of thyroid follicles (thyroidization of kidneys)  in later stages results in tubular atrophy and interstitial fibrosis
  • 165. Marc Imhotep Cray, MD Acute pyelonephritis, gross 165  Note in this plate that cortical surface is studded w focal pale abscesses, more numerous in upper pole and middle region of kidney, lower pole is relatively unaffected  Betw. abscesses, there is dark congestion of renal surface Kumar V, Abbas AK, Aster JC. Robbins and Cotran Pathologic Basis of Disease, 9th ed. Philadelphia: Saunders-Elsevier, 2015.
  • 166. Marc Imhotep Cray, MD Acute pyelonephritis, microscopic 166  An extensive infiltrate of neutrophils is present in collecting tubules and interstitial tissue Rubin R , Strayer DS Eds. Rubin’s Pathology: Clinicopathologic Foundations of Medicine, 6th Ed. Baltimore: Lippincott Williams & Wilkins, 2012.
  • 167. Marc Imhotep Cray, MD Pyelonephritis cont’d. 167 Pathogenesis Principal causative organisms in acute pyelonephritis are enteric gram-negative bacilli  Escherichia coli is most common  Other important organisms are Proteus, Klebsiella, Enterobacter, and Pseudomonas these usually are assoc. w recurrent infections, especially in individuals who undergo urinary tract manipulations or have congenital or acquired anomalies of lower urinary tract Bacteria can reach kidneys from lower urinary tract (ascending infection) or through bloodstream (hematogenous infection)  N.B. Ascending infection from lower urinary tract is most important & frequent route by which bacteria reach kidney
  • 168. Marc Imhotep Cray, MD Pathways of renal infection 168  Hematogenous infection results from bacteremic spread  ascending infection results from a combination of o urinary bladder infection, o vesicoureteral reflux, and o intrarenal reflux Kumar V, Abbas AK, Aster JC. Robbins and Cotran Pathologic Basis of Disease, 9th ed. Philadelphia: Saunders-Elsevier, 2015.
  • 169. Marc Imhotep Cray, MD 169 Buja LM, Krueger GR. Netter’s Illustrated Human Pathology, 2nd Ed. Philadelphia: Saunders-Elsevier, 2014.
  • 170. Marc Imhotep Cray, MD 170 Vesicoureteral reflux demonstrated by a voiding  Dye injected into bladder refluxes into both dilated ureters, filling pelvis and calyces  In absence of vesicoureteral reflux, infection remains localized in bladder o majority of individuals w repeated or persistent bacterial colonization of urinary tract suffer from cystitis and urethritis (lower UTI) rather than pyelonephritis Kumar V, Abbas AK, Aster JC. Robbins and Cotran Pathologic Basis of Disease, 9th ed. Philadelphia: Saunders-Elsevier, 2015.
  • 171. Marc Imhotep Cray, MD Pyelonephritis cont’d. 171  Clinical Manifestations  Acute: Fever; flank pain with CVA tenderness; polyuria and dysuria; nausea, vomiting, and diarrhea  Chronic: Recurrent episodes of acute pyelonephritis can lead to renal hypertension and ESRD  Lab findings: Leukocytosis, WBC and/or WBC casts in urine Treatment Acute: IV antibiotics Chronic: Renal transplantation if progresses to ESRD
  • 172. 172 Chronic pyelonephritis, gross Rubin R , Strayer DS Eds. Rubin’s Pathology: Clinicopathologic Foundations of Medicine, 6th Ed. Baltimore: Lippincott Williams & Wilkins, 2012.  A Cortical surface contains many irregular, depressed scars (reddish areas)  B marked dilation of calyces (caliectasis) caused by inflammatory destruction of papillae, w atrophy and scarring of overlying cortex
  • 173. Marc Imhotep Cray, MD Chronic pyelonephritis, microscopic 173 Rubin R , Strayer DS Eds. Rubin’s Pathology: Clinicopathologic Foundations of Medicine, 6th Ed. Baltimore: Lippincott Williams & Wilkins, 2012.  A LM shows tubular dilation & atrophy, w many tubules containing eosinophilic hyaline casts resembling colloid of thyroid follicles (so-called thyroidization)  interstitium is scarred and contains a chronic inflammatory cell infiltrate
  • 174. Marc Imhotep Cray, MD Two major types of chronic pyelonephritis: Left. Vesicoureteral reflux causes infection of peripheral papillae and, therefore, scars in poles of kidney Right. Obstruction of urinary tract leads to high pressure backflow of urine, causes infection of all papillae, diffuse scarring of kidney & thinning of cortex 174 Rubin R , Strayer DS Eds. Rubin’s Pathology: Clinicopathologic Foundations of Medicine, 6th Ed. Baltimore: Lippincott Williams & Wilkins, 2012.
  • 175. Marc Imhotep Cray, MD Reflux nephropathy [vesicoureteral reflux] 175 N.B. Most common mechanism in pathogenesis of chronic pyelonephritis is reflux nephropathy  Reflux nephropathy is kidney damage (nephropathy) due to urine flowing backward (reflux) from bladder toward kidneys also called vesicoureteral reflux (VUR)  Longstanding VUR can result in small and scarred kidneys during first five years of life in affected children  End results of reflux nephropathy can include high blood pressure, excessive protein loss in urine, and eventually kidney failure  When reflux nephropathy is suspected as a cause of kidney disease, other conditions to consider include chronic pyelonephritis, obstructive uropathy, and analgesic overuse
  • 176. Marc Imhotep Cray, MD Renal papillary necrosis & Diffuse cortical necrosis 176 Important notes:  Renal papillary necrosis is a complication of acute pyelonephritis in diabetics or analgesics particularly those ingested at least 2 kg in past (=analgesic nephropathy)  risk is higher for phenacetin (withdrawn from market in U.S.) and acetaminophen compared to aspirin and other NSAIDs  Diffuse cortical necrosis is an acute generalized infarction of renal cortices (medulla is spared)usually b/c of a combination of DIC and end-organ vasospasm in assoc. w obstetric catastrophes or septic shock
  • 177. Marc Imhotep Cray, MD Papillary necrosis 177  Bisected kidney shows a dilated renal pelvis and dilated calyces secondary to urinary tract obstruction  Papillae are all necrotic and appear as sharply demarcated, ragged, yellowish areas Rubin R , Strayer DS Eds. Rubin’s Pathology: Clinicopathologic Foundations of Medicine, 6th Ed. Baltimore: Lippincott Williams & Wilkins, 2012. Mnemonic for causes of renal papillary necrosis is POSTCARDS: pyelonephritis, obstruction of urogenital tract, sickle cell disease, tuberculosis, cirrhosis, analgesia/alcohol abuse, renal vein thrombosis, diabetes mellitus, and systemic vasculitis
  • 178. Marc Imhotep Cray, MD Renal cortical necrosis 178  Cortex of kidney is pale yellow and soft due to diffuse cortical necrosis Rubin R , Strayer DS Eds. Rubin’s Pathology: Clinicopathologic Foundations of Medicine, 6th Ed. Baltimore: Lippincott Williams & Wilkins, 2012.  Usually caused by significantly diminished arterial perfusion of kidneys due to spasms of feeding arteries, microvascular injury, or DIC  Renal cortical necrosis is the pathological progression of acute tubular necrosis
  • 179. Marc Imhotep Cray, MD Case Trigger 12 179 A 68-year-old man presents to the emergency room because he is unable to urinate. After taking a detailed history, you learn that he has had increasing urinary hesitancy and decreased force of his urine stream for several months. He also complains of a sensation of incomplete bladder emptying. On rectal examination, you find a smooth, firm, elastic enlargement of the prostate. Examination of the lower abdomen reveals signs of a distended bladder. You order laboratory studies that show increased urinary sodium excretion and an elevated BUN and creatinine. You determine that the patient requires prompt urethral catheterization to help reverse his renal failure. What is the Diagnosis?
  • 180. Marc Imhotep Cray, MD Acute Renal Failure (Prerenal, Intrarenal, and Postrenal Azotemia) 180  Etiology  Prerenal: Caused by decreased effective arterial volume (ie, CHF, hypovolemia, systemic vasodilation [sepsis]), or renal vasoconstriction (NSAIDs, ACE inhibitors, RAS)  Intrarenal: Caused by acute tubular necrosis, acute interstitial nephritis, glomerulonephritis, and thrombotic microangiopathy  Postrenal: Caused by kidney stones, BPH, neurogenic bladder, and neoplasia  Pathology and Pathophysiology  Prerenal: Renal hypoperfusion leads to decreased GFR resulting in sodium and water retention  Intrarenal: Characterized by patchy tubular necrosis leads to tubule obstruction and fluid backflow across necrotic tubule and a resulting decrease in GFR  Postrenal: Only develops w bilateral outflow obstruction
  • 181. Marc Imhotep Cray, MD Acute Kidney Injury (AKI) [previously ARF] 181  An acute rise in serum Cr has been called ARF  It is classified as: prerenal, intrarenal, postrenal azotemia Clinical Manifestations:  oliguria; azotemia; hyperkalemia  Intrarenal ARF is now called acute kidney injury (AKI) AKI is categorized by portion of kidney that is primarily injured:  glomeruli (e.g., acute glomerulonephritis)  vessels (e.g., vasculitis),  tubules (e.g., ischemic acute tubular injury (previously ATN) or  interstitium (acute interstitial nephritis)  most common cause intrarenal AKI is ischemic acute tubular injury  most common cause of AKI overall is therapeutic drugs